You are on page 1of 717

10,000 SERIES (20

QUESTIONS)

1. Dr. Brown, a dermatologist, opened and drained multiple


complicated acne pustules and comedones on a 19-year-old
patient with severe acne. For one of the more complicated acne
pustules, Dr. Brown incised, drained, and marsupial zed the acne
cyst by suturing the right and left sides, leaving the cyst open for
drainage. What is the correct code for the procedure?

A. 10060

B. 10061, 10040

C. 10040

D. 10061

2. Dr. Brown, a dermatologist, is seeing a patient for an acne


surgery follow-up that was performed eight months ago. The
surgery was successful and the patient’s skin condition has rapidly
improved due to a strict hygiene regimen and acne medications.
Today the patient stated that he would like to reduce acne scarring
by having dermabrasion on his entire face. Although the patient
requested the entire face, the dermatologist only performed the
dermabrasion on the areas on the patient’s face that were not
currently prone to acne breakouts (his forehead and brow area).
What is the correct code for this procedure?

A. 15781

B. 15780

C. 15786

D. 15788
3. A physician biopsied the skin on a 30-year-old woman’s
upper eyelid, with a possible fungal infection. Three sites were
biopsied. How should the physician code for this procedure?

A. 11100, 11101 (X2)

B. 11100 (X3)

C. 11100, 67810

D. 67810

4. The physician performed the excision of two 1.5 cm


malignant lesions on a patient’s upper back. During the surgery,
the physician noted four additional lesions, which looked to be pre-
malignant. These lesion excisions were 0.3, 0.7, 1.0, and 1.45 cm,
respectively. The suspect lesions were sent to pathology lab,
where they were determined to be benign. What are the
appropriate codes for the service?

A. 11400 (X4), 11602 (X2)

B. 11400, 11401 (X2), 11402

C. 11404, 11603
D. 11400, 11401 (X2), 11402, 11602 (X2)

5. A 22-year-old woman with excessive skin of her upper


eyelid presented to her physician’s office for the revision of her
right eyelid. Her physician decided that she would perform a
blepharoplasty of the right eyelid, during which she would remove
the excessive skin weighing down the lid. When the physician
performs this procedure, what code should she report?
A. 15822

B. 15823

C. 15823-50

D. 15820

6. Two weeks ago, a 32-year-old male suffered a thumb injury


at work. He was using a belt grinder on a small piece of steel when
the belt caught the tip of his left thumb in the grinder. His thumb
was ground down to the tip, his thumb nail was removed and a
small piece of his anterior phalange bone was broken off. The
patient has an appointment scheduled today to see if his nail bed
on his left thumb can be reconstructed using a graft from his left
great toe. What is code for this procedure?

A.

11760-

LT B.

11765
C. 11762-LT

D. 11730, 11732
7. A 10-year-old boy was running through his house and ran
into a sliding glass door, breaking the glass and suffering severe
lacerations on his trunk and arms and minor lacerations on his face
and legs. The emergency department physician performed the
simple closure of one 2 cm laceration on the boy’s cheek and two
2.3 cm lacerations on the boy’s left leg. The physician performed
the simple closure of one 4 cm laceration on the right arm and the
layered closure of two lacerations on the left arm, which were 1.5
and 3 cm, respectively. The physician treated the 5 cm laceration
on the boy’s chest, which required the removal of particulate glass
and a single layer closure. What are the correct codes for the
wound repair performed by the emergency department physician?
A. 12001, 12002 (X2), 12032 (X2)

B. 12005, 12011-51, 12032-51

C. 12004, 12011-51, 12032 (X2)

D. 12004, 12011-51, 12034-51

8. A 32-year old woman with severe gynecomastia and a


prevalent family history of breast cancer elected to have a
complete bilateral mastectomy with immediate insertion of
breast prosthesis implants. What is/are the correct code(s) for
the procedure?

A. 19300-50, 19340-50

B. 19300-50, 19340-50

C. 19303, 19357

D. 19300-50, 19357-50
9. A 56-year-old male presents to a podiatry office with
complaints of recurrent problems with an ingrown toenail on his
right great toe. He has been treated multiple times with antibiotics,
but the pain and swelling is causing him difficulty wearing his work
boots. He currently does not have an infection, although the
ingrown toenail is present and painful. The doctor performs a
wedge excision of the skin of the nail fold to his right great toe.
Which of the following codes should you use?

A. 10060

B. 11760
C. 1176511765

D. 11750

10. A 10-year-old girl comes to her pediatrician’s office after


smashing the nail of her right 2nd digit in the car door the previous
night. This morning, the tip of her finger is edematous, there is a
large amount of ecchymosis present, and the pain is worse than
when the injury initially occurred. The pediatrician uses a
disposable Bovie high-temperature cautery pen to cauterize a hole
in the nail to allow the subungual hematoma to drain. A small
amount of serosanguineous fluid is removed and pain rapidly
decreased. Which of the following codes should you use?

A. 11720

B. 11730

C. 11740

D. 11750
11. A 31-year-old male arrives at general surgery for a pilonidal
cyst that has been unresponsive to antibiotic therapy at his primary
care physician’s (PCP) office. There is a large amount of
tenderness and edema at the site and the area is erythematic and
warm to the touch. The patient’s pain has been increasing and, on
palpation, the surgeon notes that the cyst is larger than six cm in
diameter and appears to be deep. After discussion with the patient,
it is decided to perform an incision and drainage (I&D) at the
surgeon’s office, with \ no sedation other than local use of
Lidocaine. The area was prepped and numbed with Lidocaine and
an incision was made, allowing a large amount of purulent
drainage to be expressed. The area was packed with gauze and
the procedure was completed without any complications. The
patient will follow up in the office in one week for recheck and is to
continue on the antibiotic prescribed by his PCP. Which of the
following codes should you use?
A. 10060

B. 10061

C. 10080

D. 10081

12. A 35-year-old male returns to the general surgeon’s office


with recurrent episodes of infected pilonidal cysts. He has had the
cyst incised and drained (I&D) once at his primary care physician’s
office and once at the surgeon’s office. The last I&D was
approximately four months ago. He returns to the office today to
discuss further treatment. It is decided that he will undergo
outpatient surgery so he can be sedated and the cyst will be
excised with the base curetted. During surgery, it was noted that
there was an extensive sinus tract. The wound will be packed
open, requiring daily dressing changes and he will return to the
postop clinic for a recheck in one week. Which of the following
codes should you use?

A. 11771

B. 11772
C. 11770

D. 10080

13. A surgeon is doing a pilonidal cyst incision and drainage


with excision in the operating room. What he initially thought was
an uncomplicated but deep pilonidal cyst developed into an
extensively complicated cyst excision with a z-plasty required to
repair the incision. How would you code this procedure?
A. 11770

B. 11772

C. 10081

D. 11771

14. A 22-year-old male was chopping wood with his friends and
accidently slashed his right lower leg with the axe. Upon
assessment at the emergency room, it was determined that he had
sliced through all layers of skin and the wound was down to
muscle, but no arteries were involved. He required layered sutured
closure including the deeper layers of subcutaneous tissue, and
superficial fascia as well as the epidermis and dermis. His wound
was measured at approximately 25 cm. How would you code this
procedure?

A. 12035

B. 12036

C. 12045

D. 12046
15. A 6-year-old boy was bitten by a dog and sustained multiple
lacerations as a result. He has a 5 cm laceration on his right lower
arm, a 2 cm laceration on his left hand, and a 2.6 cm laceration to
the right side of his lower cheek. While in the ER, the wounds were
flushed, then the patient was sedated and the arm and hand
lacerations were repaired. Intermediate repair was necessary for
the facial laceration due to the depth of the wound and layered
closure was performed. The patient was also given a tetanus shot
and started on antibiotics. What CPT codes would you use for the
laceration repairs of this child and what order is correct?
A. 12002, 12013

B. 12002, 12013 -59

C. 12032, 12002 -59

D. 12032, 12002

16. A 78-year-old male arrives at a dermatology office with


multiple skin tags on his arms, neck, and torso. There are a total of
14 lesions that need to be removed and the dermatologist removes
all 14 tags within the office. What is the correct code for this
procedure?

A. 11101

B. 11200

C. 11201

D. 11300
17. A 60-year-old woman presents to her dermatologists office
with a reddish brown lesion on her right upper arm. It is
approximately 0.8 cm in diameter and is raised. The patient and
provider decide to proceed with a shave excision and will send this
to pathology for confirmation. The dermatologist suspects this is a
benign angiofibroma. The patient receives local anesthetics at the
site and the area is excised with a sharp razor. What is the correct
code for this procedure?

A. 11300

B. 11301
C. 11302

D. 11305

18. A 12-year-old boy presents to his pediatrician’s office with


multiple molluscum contagiosum on his face, neck and upper right
shoulder. They have been present for about 15 months and are
getting larger. His parents are concerned about his appearance
and that he keeps scratching them. He was treated once for a local
infection on his neck, about three months ago. His parents would
like to have these removed or be referred to dermatology. The
pediatrician discusses options and the parents opt to use
cryotherapy with Histofreeze that the pediatrician has in his office.
In total, there are six lesions on his right shoulder, eight on his neck
and five on his cheek. What CPT code would you use?

A. 17111

B. 17110

C. 17000

D. 17004
19. A 24-year-old woman found a lump in her right breast
during a routine exam. She had a mammogram, which
confirmed the lump, and then met with a surgeon for a needle
biopsy without imaging. What code would you use for this
procedure?

A. 19085

B. 19100

C. 19101
D. 19086

20. A 22-year-old female is concerned about her inverted nipples.


She would like them to be corrected and sees a plastic surgeon for
a consult. She is self-conscious about this issue and wants to fix it.
The plastic surgeon explains her options and they decide to correct
this complaint. What code would you use for this procedure?

A. 19350

B. 19357

C. 19355

D. 19499
20,000 SERIES (37
QUESTIONS)

1. What is the difference between biopsy codes located in the


integumentary section and those found in the musculoskeletal
section?

A. The biopsy codes found in the integumentary section


are only for codes related to malignant neoplasms
B. There are no biopsy codes found in the musculoskeletal
section

C. The codes in the musculoskeletal system include biopsies


for bone only, whereas the biopsy codes found in the
integumentary section include codes for biopsies of
subcutaneous structures including bone
D. The biopsy codes found in the integumentary section are
for biopsies of the skin and subcutaneous structures whereas the
biopsy codes found in the musculoskeletal section are for deeper
structures

2. A physician took an impression of a 47-year-old woman’s


left orbital socket and created a custom prosthesis. What is the
correct code for this service?
A. 21076

B. 21089 -LT

C. 21088 -LT

D. 21077 -LT

3. The physician performed an arthroscopy of TMJ with


biopsy of soft tissue. What is the appropriate CPT code?
A. 29800

B. 29804

C. 2970

D. 21010

4.

PROCEDURAL

NOTE PATIENT:

Trohoske,

Janine AGE: 62
DATE: 01/13/2017

PREOPERATIVE DIAGNOSIS:

Degenerative Disc Disease

POSTOPERATIVE DIAGNOSIS:

Degenerative Disc Disease


PROCEDURE: Arthrodesis of L4-L2

utilizing autogenous bone graft

An anesthetized patient was placed in the prone position on the


operating table and draped in the usual manner. An incision was
made along the spinal column, from the area of the L1 to L5, and
skin and subcutaneous tissues were pinned back to allow access
to the L4-L2 vertebral spaces. A separate fascial incision was
made to obtain morselized bone graft segments for arthrodesis
procedure. Posterior arthrodesis was then performed along the L4-
L2 vertebrae. No additional fixation or instrumentation was placed.
Incision was then closed, stapled together, and dressed with a
sterile dressing. What is the appropriate code for this procedure?

A. 22612, 20937

B. 22612, 22614
C. 22612, 22614 (X2),
20937

D. 22612, 22614 (X2),


20936

5. A physician excised the head of the humeral bone and


replaced it with the appropriate implant. What is the correct code
for this procedure?

A. 23195

B. 23470

C. 23195, 23470

D. 23472

6. A physician performed the

following trigger point injections: Two

injections into the flexor carpi muscle


One injection into the

extensor carpi muscle Three

injections into the triceps

brachii
One injection into the biceps
brachii

What is the correct code for these injections?

A. 20553 (X7)

B. 20553

C. 20552 (X2), 20553 (X5)


D. 20552, 77021

7. A patient suffered a comminuted fracture of the right arm


due to a crush injury. The physician implanted three pins in a
single plane above the fracture and two pins and one wire along
the same plane under the fracture location on the humerus of
the right arm. What is the correct code for this service?

A. 20690

B. 20962

C. 20690 (X6)

D. 20692 (X6)

8.

PROGRESS

NOTE

PATIENT:
FINKE,

ISABEL AGE:

15
DATE: 3/14/2015

NOTE: The patient states that she was riding her bicycle on the
side of the road when a large dog ran out in front of her and forced
her to crash into a drainage ditch. The patient hit her left arm on a
cement drain pipe, suffering an oblique fracture to her left radius.
The fracture was set with manipulation of the radial shaft. The arm
was then casted in the usual fashion. The patient tolerated the
procedure well and was asked to return to the office in one month
for follow up x-ray and removal of the cast. How should you code
for this fracture repair?
A. 25505

B. 25500

C. 25515

D. 25535

9. The patient returned to the office one month later for


removal of cast on her left lower arm. The original attending
physician removed the cast. The physician also examined the arm
and determined that no further casting or follow-up was
necessary. What is the appropriate code for this service?

A. 25250

B. 99214

C. No code would

be reported D.

29799
10. What code would you use to report the enlargement and
exploration of a penetrating 5 cm stab wound to the upper thigh,
with ligation of minor muscular blood vessel, and appropriate
intermediate closure?

A. 12032

B. 20103, 12032

C. 20103
D. 12031

11. A patient was suffering from severe hallux valgus of the right
foot. The physician excised extraneous bone from the lateral end of
the proximal phalanx and the medial eminence of the metatarsal
bone. The physician inserted the implant in the lateral end of the
proximal phalanx. What is the correct CPT code for this procedure?

A. 28290

B. 28292

C. 28293

D. 28296

12. A 15-year-old boy was pushed during a basketball game


and landed face down, hitting the bottom bleacher with his nose.
There was immediate swelling and bruising and the nose was
displaced to the left. In the ER, the left nasal passage was not
patent and the septum was deviated. The patient was taken to
the operating room and closed treatment of the nasal fracture
with stabilization was performed. Which code is correct for this
procedure?

A. 21310

B. 21315

C. 21320

D. 21325
13. A 67-year-old male has an intramuscular tumor of his right
scalp over the parietal bone that is approximately 3 cm in diameter.
It is getting larger and is causing him pain. He elects to have it
removed and will have this excised in the OR as an outpatient
procedure. Which procedure code is correct?

A. 21011

B. 21012

C. 21013

D. 21014

14. A 76-year-old woman has noticed a growth along the left


side of her neck that is increasing in size. Initially it looked like a
mosquito bite, but has increased over the last three months to a
dime size, approximately 1.8 cm. Her PCP believes it is a lipoma
and sends her to surgery for assessment. It was removed in the
office using local anesthesia. Which code is correct?

A. 21550
B. 21555

C. 21552

D. 21556

15. A 25-year-old obese male has had right-sided flank pain for
about two months and noticed a lump during work one day. He
initially thought it was a muscle spasm and ignored it, but it has
increased in size and has gotten more painful. He went through a
CT scan, which showed an intramuscular mass
consistent to the physical exam that is approximately 6 cm in
diameter. Due to the size and presentation, it was decided to
excise the tumor. What is the correct code to use for this
procedure?

A. 21931

B. 21932

C. 21933

D. 21935

16. A 35-year-old patient has a lipoma on her upper back over


her scapula, which causes increased discomfort when her bra
strap rubs against it. She has already met with her surgeon and
this procedure will be performed in his office because it is only 2.75
cm in diameter and only involves the subcutaneous layer. What
CPT code would you use for this procedure?

A. 21920

B. 21930

C. 21931
D. 21932

17. There is a large mass noted on a 56-year-old man’s right


flank. Due to the size and asymptomatic characteristics, a CT
scan was performed and biopsy was recommended. This is a
deeply rooted tumor and the patient will be sedated during this
procedure. What code should you use for a deep tissue biopsy?
A. 21920

B. 21925

C. 21935

D. 21936

18. A 54-year-old female with degenerative disc disease at L4-L5


undergoes surgical repair with anterior approach for discectomy
and fusion to try to prevent further discomfort. What CPT code
would you use for this procedure?

A. 22554

B. 22556

C. 22558

D. 22585

19. A 56-year-old man undergoes percutaneous vertebroplasty


with vertebral augmentation to L2-L3 with unilateral injection.
What is the correct CPT code for this procedure?

A. 22510

B. 22511

C. 22513

D. 22526
20. A 37-year-old woman notices a lump in her left lower
abdominal wall. A large, 6 cm tumor was found on CT scan along
the musculature of the abdominal wall. This was removed and
found to be a desmoid tumor. What is the code for this procedure?

A. 22900

B. 22901

C. 22902

D. 22903

21. A 27-year-old woman began complaining about right lower-


quadrant pain, was noted to have rebound discomfort, and positive
pain over McBurney’s point. A CT scan was ordered due to
suspicion of appendicitis. The CT ruled out appendicitis but noted a
2.5 cm subcutaneous soft tissue mass in the right upper pelvis.
She undergoes surgery to have this excised. What is the correct
code for this procedure?

A. 22900
B. 22901

C. 22902

D. 22903

22. A 52-year-old man presents to the ER with severe abdominal


pain. He has had abdominal pain for about six months, which has
been coming and going. The pain increased one week ago, and
today the patient was unable to bend without becoming short of
breath due to the pain. A CT scan was performed
and patient was noted to have a large, 7.5 cm soft tissue tumor
attached to his abdominal wall. The patient underwent surgery to
have it removed and it was found to be a sarcoma. During
surgery, the patient also had a large margin removed from around
the tumor site. What is the correct code for this procedure?

A. 22902

B. 22903

C. 22904

D. 22905

23. A 78-year-old woman fell going up her steps and landed on


her outstretched right hand. She fractured her proximal humerus.
Unfortunately, this was a displaced fracture requiring open
reduction and fixation. What is the correct code for this procedure?

A. 23600

B. 23605

C. 23615
D. 23616

24. A 48-year-old woman fell off her horse, tearing her rotator
cuff. This required open surgery to repair the right rotator cuff.
What is the CPT code for this procedure?

A. 23410
B. 23412

C. 23415

D. 23420

25. A 12-year-old boy arrives to his pediatrician’s office


complaining of right arm pain around the elbow. Two days prior, he
fell during football practice. An x-ray was obtained and it was noted
that he had a displaced fracture of the capitellum at the radial
head. This was a closed fracture requiring manipulation. Which
CPT code is correct?

A. 24600

B. 24620

C. 24640

D. 24655

26. An 85-year-old woman fell directly on her elbow resulting in a


complex fracture. It was recommended to have a total elbow
arthroplasty performed to give her range of motion and use of this
joint. Which code is correct?

A. 24360

B. 24361

C. 24362

D. 24363
27. A 54-year-old woman has noticed a pea sized lesion on the
top of her left wrist. Her PCP diagnosed it as a ganglion cyst,
approximately 1 cm in diameter. He referred her to surgery for
removal, as it was becoming painful. What is the correct code for
surgical removal of this cyst?

A. 25111

B. 25112

C. 25115

D. 25130

28. A 25-year-old construction worker has been diagnosed with


trigger finger of his right 2nd digit. He has received three steroid
injections over the last year, but it continues to flare. He is referred
to surgery for further evaluation. The surgeon recommends a
tendon sheath incision and this is scheduled as an outpatient.
What is the correct CPT code for this procedure?

A. 26055
B. 26060

C. 26160

D. 26170

29. A 3-year-old girl was born with two polydactylous digits on


her right hand. One digit was soft tissue only and was able to be
tied off at birth. The second digit included bone and skin and was
unable to be
treated at birth. Because she is now three years old, this extra
digit will be removed, leaving her with five digits total. What is the
correct code for this procedure?

A. 26560

B. 26561

C. 26587

D. 26590

30. A 27-year-old male was cutting wood, using a circular saw,


when he accidentally cut off the end of his 3rd digit. On x-ray it
showed that he cut the distal phalange and partially cut the middle
phalange of the 3rd digit. Due to bone splinters and need for
debridement, the 3rd digit middle phalange was amputated back to
the proximal phalange, leaving a wide border of healthy bone and
tissue. What is the correct CPT code for this procedure?

A. 26910

B. 26951

C. 26952
D. 26989

31. A 56-year-old man with severe osteoarthritis in his hips


follows up with orthopedics for worsening pain in his right hip. At
this time, he opts for a total hip replacement on the right hip and
begins the process for surgery and rehab afterwards. What code
would you use for a total hip replacement?
A. 27125

B. 27130

C. 27134

D. 27236

32. A 27-year-old female fell off the tailgate of a truck, landing on


her coccyx. She sustained a compound fracture and needs surgery
for it to be removed. Which code is correct?

A. 27070

B. 27200

C. 27080

D. 27202

33. A 24-year-old college football player injured his knee during


practice. The MRI shows a torn medial and lateral meniscus. The
orthopedist reviewed the films and decides to proceed with an
arthroscopy with meniscectomy to remove the torn pieces of the
meniscus from the knee. What is the best code for this procedure?

A. 29879

B. 29880

C. 29881

D. 29882
E. 29883

34. An 18-year-old soccer player falls while running and


dislocates his knee. He was taken to the ER with significant
edema. He is found to have a significant patellar dislocation, which
will be reduced in the ER with conscious sedation. Which code is
the best code for this procedure?

A. 27550

B. 27552

C. 27560

D. 27562

35. A ballet dancer completely ruptured her left Achilles tendon


during a production. She requires open repair without grafting.
What is the best code for this procedure?

A. 27650
B. 27652

C. 27654

D. 27658

36. A 56-year-old type 2 diabetic has been battling diabetic


ulcers and poor perfusion in his feet and toes. Due to an infection
that was not quickly treated he developed gangrene in the tip of
his right 3rd
digit and the metatarsal had to be amputated to leave healthy
tissue and promote healing. It was amputated up to the
proximal phalange. What is the correct code for this
procedure?

A. 28805

B. 28810

C. 28820

D. 28825

37. A 52-year-old woman who has rheumatoid arthritis is


suffering with her right second toe bending upward in the middle. It
is causing her to not be able to wear her shoes and she is having
pain when walking. Her rheumatologist referred her to orthopedics
to have the hammertoe looked at and possibly corrected. She is
going to have surgery to correct the hammertoe by doing an
interphalangeal fusion. What is the correct code for this procedure?

A. 28280

B. 28285
C. 28286

D. 28290
30,000 SERIES (36
QUESTIONS)

1. What is the difference between anterior nasal packing


versus posterior nasal packing procedure codes?

A. None, they are the same codes

B. Posterior packing is done in the larynx, whereas anterior


packing is done at the back of the throat

C. There are no nasal packing procedure codes in the CPT


manual

D. Anterior packing is applied pressure and gauze and


posterior packing is the insertion of a balloon into the back of the
nasal cavity

2. A 3-year-old male presented to the pediatrician’s office after


the patient’s mother discovered that the boy had inserted a jelly
bean into his nose. The mother, when trying to remove the jelly
bean, ended up pushing the bean further into the patient’s nose to
the point that it is no longer visible. Upon presentation, the patient
was extremely agitated and anxious, and was experiencing pain
and swelling in the left nasal sinus. The pediatrician determined
that due to the patient’s emotional state and the depth of the jelly
bean, the patient would have to be anesthetized in order to remove
it. The pediatrician admitted the patient into the hospital and
performed the removal of the foreign body under general
anesthesia. At one point, a lateral rhinotomy was considered but
ruled out, as the physician was finally able to grasp the foreign
body. How should the pediatrician code for this service?

A. 99214 -57, 30310

B. 99392 -57, 30310

C. 30310

D. 99214 -57, 30320


3. A patient with chronic sinus pain and swelling presented to
the office for a diagnostic endoscopy of the nasal sinuses. During
the procedure, the surgeon excised two polyps on the left
sphenoid sinus. What is the correct code for this procedure?

A. 31231, 31235

B. 31231, 31237

C. 31235

D. 31237

4. A 2-month-old male with diphtheria presented to the hospital


with severe respiratory distress. The emergency department
physician performed a pulse oximetry and discovered that the
patient’s blood oxygen level was in the 60’s. The on-call physician
ordered an emergency endotracheal intubation to restore oxygen
flow to the infant. Once the airflow was established, care was
transferred to the on-call pediatrician. What is the appropriate
code for the emergency department procedure(s)?

A. 31500, 94760-51
B. 31500, 94761

C. 31603, 94761

D. 31603, 94760 -51

5. A 47-year-old male patient with advanced cancer of the


lower left mandible presented to the hospital for surgical removal of
the lower left jawbone with secondary insertion of mandibular
prosthesis. In
order to perform surgery, the patient had to be intubated through a
tracheostomy. After anesthesia, the surgeon performed the
tracheostomy by incising the cricothyroid membrane horizontally
along the trachea and inserting the intubation device. The surgeon
completed the primary surgical procedure on the patient’s
mandible. What is the correct code for the intubation?

A. 31605

B. 31600

C. No code would be used

for the intubation D. 31603

6. You can find codes for cardiovascular services in the three


different chapters within the CPT manual. They are:

A. Surgery, Anesthesia, and Medicine

B. Surgery, Medicine, and Laboratory

C. Medicine, Radiology, and Evaluation and Management

D. Surgery, Radiology, and Medicine


7. What term is used for the two-layered protective membrane
that surrounds the heart and the roots of the great vessels?

A. Pleura
B. Pericardium

C. Atrium

D. Atrium

8. In the electrical conduction system of the heart, the sinoatrial


node is located in the right atrium near the superior vena cava. It
serves as the normal pacemaker of the heart. There is another
node, which is located in the right atrium on the septal wall. This
node slows the impulses of the heart so that the atria can fill with
blood before it contracts. What is the name of this node?

A. Sinoatrial Node

B. Bundle of His

C. Purkinje Fibers

D. Atrioventricular Node

9. A pacemaker or pacing cardioverter-defibrillator that has


pacing and sensing functions in three or more chambers of the
heart is considered a:

A. Multiple Lead

B. Dual Lead

C. Single Lead

D. Triple Lead
10. A cardiovascular surgeon implanted a temporary pacemaker
system in a patient with cardiac arrhythmia. The implanted system
contained transvenous dual chamber pacing electrodes. What is
the correct code for the procedure?

A. 33210

B. 33211

C. 33216

D. 33217

11. A physician removed a 54-year-old male’s old pulse


generator on a dual lead system cardioverter- defibrillator and
replaced it with a new one. During the procedure, the surgeon
determined that the electrodes needed to be replaced as well and
removed the old electrodes, replacing them with new ones. What
are the correct codes for this surgical procedure?

A. 33262, 33243 -51, 33249 -51

B. 33263, 33241 -51, 33244 -51


C. 33263, 33243 -51, 33249 -51

D. 33262, 33241 -51, 33249 -51

12. What is the difference between arteriosclerosis and


arteriostenosis?
A. -sclerosis is the enlargement of an artery and -stenosis is the
narrowing of an artery

B. . -sclerosis is the blocking of an artery and -stenosis is the


hardening of an artery

C. -sclerosis is the hardening of an artery and -stenosis is the


narrowing of an artery

D. -sclerosis is the enlargement of an artery and -stenosis is the


hardening of an artery

13. A physician performed a thromboendarterectomy with a


patch graft on the common femoral artery of a 63-year-old female
with advanced lower arterial plaque. What is the correct code for
this procedure?

A. 35301

B. 35302

C. 35355

D. 35371
14. A 14-year-old patient with an abscessed tooth presented to
the physician’s office with possible sepsis. The tooth had gone
untreated for two weeks, and now the patient is experiencing a
high fever, severe headaches and toothaches and malaise and
fatigue. The physician suspects that the bacteria from the tooth
has spread to the patient’s blood and is now a systemic infection.
As part of the office procedure, the physician orders a CBC in
order to examine the bacterial levels in the patient’s blood. After
the physician writes the orders, the nurse performs a venipuncture
on the patient in order to obtain a blood sample. What is the
correct code for the collection of the blood only?

A. 36416
B. 36415

C. 36410

D. 36406

15. A 48-year-old patient was brought to the Emergency


Department following a horseback riding injury. The patient was
riding in the woods when the horse spooked at the sight of a snake
and bucked the patient, who was subsequently kicked by the horse
in the abdomen. The patient was suffering severe internal pain, and
upon presentation to the ED, the abdomen was stiff and distended.
Suspecting an internal splenic injury, the physician ordered
emergency surgery. During the surgery, the physician performed a
partial lateral splenectomy with a splenorrhaphy on the anterior
face of the spleen. Upon gross inspection, the remaining organs
were all intact. The physician then sutured the abdominal cavity
and admitted the patient to the ICU for observation. What is the
correct procedure code for this service?

A. 38115

B. 38101, 38115

C. 38101
D. 38120

16. A physician performed an X-ray of the spleen and portal


system. Assign the appropriate CPT code(s).

A. 38200, 75810-26

B. 38200
C. 75810

D. 38200, 75810

17. After careful selection and testing of bone marrow donors, a


potential candidate was found for a patient with severe leukemia.
The physician collected a small sample of the potential donor’s
bone marrow via aspiration technique. This sample was then sent
to pathology to determine whether or not it would be a match for
the patient’s bone marrow. What is the correct code for the
procedure performed by the physician?

A. 38221

B. 38220

C. 38230

D. 38232

18. A physician performed the open excision of two deep axillary


nodes in a 45-year-old female patient. What is the correct code for
this procedure?

A. 38500

B. 38555

C. 38525

D. 38525 (X2)
19. A physician performed a lymphangiography with insertion of
radioactive tracer for identification of sentinel node. What is the
correct code for this procedure?

A. 38790 -50, 75803

B. 38790, 75801

C. 38792 -50, 75803

D. 38792, 75801

20. A physician performed a resection of the diaphragm utilizing


an autogenous free muscle flap. What is the correct CPT code?

A. 39561

B. 39560

C. 39540

D. 39541
21. How are mediastinotomy codes divided?

A. Based on the age of the patient

B. Based on the site of the mediastinotomy

C. Based on the approach to the mediastinotomy


D. Based on whether it was surgical or diagnostic

22. A physician performed a mediastinotomy on a 54-year-old


female via cervical approach. Once access was gained, the
physician removed the foreign body from a shrapnel injury ten
years prior and biopsied a suspicious polyp near the foreign body
site. What is the correct code for this mediastinotomy?

A. 39000

B. 39010

C. 39400

D. 39499

23. A physician performed a repair of the diaphragmatic hernia


on a 15-day-old female with the creation of a ventral hernia. What
is the correct code for this service?

A. 39501
B. 39503

C. 39540

D. 39541

24. PREOPERATIVE

DIAGNOSIS: Cyst of Mediastinal

Wall POSTOPERATIVE

DIAGNOSIS: Tumor of Mediastinal

Wall
A physician removed 1.5 cm mass from mediastinal wall, along
with appropriate margins. The tumor was sent to pathology to
determine the malignancy status.

What is the correct code for

this procedure? A. 39200


B. 39220

C. 39000

D. 39010

25. A physician performed an imbrication of diaphragm to treat


an abnormal elevation of the dome of the diaphragm. The
eventration was reduced significantly by the procedure and the
patient’s condition improved greatly. What is the correct code for
this procedure?

A. 39545

B. 39560

C. 39561
D. 39501

26. A 23-year-old male has been having recurrent epistaxis of


the right naris. The last nose bleed was prolonged, lasting more
than 10 minutes. He sees his otolaryngologist and is offered
cautery in the office to stop the nose bleeds from recurring. He
agrees to the procedure and cautery is performed in
the office of the anterior right nares. This was considered a simple
procedure. What is the correct CPT code for this procedure?

A. 30901

B. 30901, -50

C. 30903

D. 30903, -50

27. A 72-year-old woman began to have excessive tears and it


was found that she had a blockage of her nasolacrimal duct on the
left side. She elected to have a nasal endoscopic
dacryocystorhinostomy to create a passage from the nose directly
to the lacrimal sac, allowing easy drainage. What is the correct
code for this procedure?

A. 31237

B. 31238

C. 31239

D. 31240
28. A 26-year-old is undergoing transnasal endoscopy of the
frontal sinus with balloon dilation. This is performed as an
outpatient and he tolerated the procedure well. Which CPT code
should you use?

A. 31233
B. 31256

C. 31276

D. 31295

29. A 3-year-old boy swallowed a small rounded block, which


became lodged in the larynx causing difficulty breathing. The
foreign body has to be removed immediately and the patient is
prepped for a laryngoscopy with removal of the foreign body.
What is the code for this procedure?

A. 31505

B. 31510

C. 31511

D. 31512

30. A 4-month-old boy with recurrent episodes of apnea and


hoarseness was diagnosed with congenital laryngeal web. Due to
symptomatic presentation, surgery is proposed by performing a
laryngoplasty with silastic keel to correct the anomaly.

A. 31535

B. 31575

C. 31580

D. 31588
31. A patient in cardiac arrest had to be intubated with an
endotracheal tube (ET) during advanced cardiac life support
treatment. What procedure code would you use for an
emergency ET tube intubation?

A. 31420

B. 31500

C. 31502

D. 31505

32. A 66-year-old woman has been having a chronic cough for


three years, which is worsening. She was finally sent to
pulmonology where a chest x-ray indicated inflammation. A flexible
bronchoscopy was scheduled to be performed with multiple
bronchial biopsies performed. This was done under fluoroscopic
guidance in the OR and the patient tolerated the procedure well.

A. 31622

B. 31623

C. 31624
D. 31625

33. A patient has empyema. What CPT code would you use for a
thoracostomy with rib resection?

A. 32035
B. 32036

C. 32096

D. 32097

34. A long-time smoker is undergoing bilobectomy of his right


lung due to cancer. He will have the middle and lower lobes
removed during his procedure. What CPT code should be used to
indicate that not just one lung or one lobe of the lung was
removed?

A. 32440

B. 32480

C. 32482

D. 32484

35. A 78-year-old woman has to have biopsies done of her right


lower lung infiltrate. She is scheduled for a thoracoscopy where a
wedge biopsy will be taken of the right lower lobe. Which code
should you use for this procedure?

A. 32601

B. 32607

C. 32608

D. 32650
36. An 8-year-old boy has a massive over the handlebar accident
on his bicycle, lacerating his spleen. He requires a total
splenectomy to control the hemorrhaging. What CPT code would
be used?

A. 38100

B. 38101

C. 38102

D. 38115
40,000 SERIES (44
QUESTIONS)

1. Mr. Johnson, a 46-year-old male, has smoked cigars for the


last 20 years of his life. He has a suspicious lesion on his lower lip
and is being seen for treatment. The physician suspects that it may
be malignant. The physician performed a shave biopsy of the
patient’s lower lip. What is the correct code for this procedure?

A. 40490

B. 40500

C. 11100

D. 11600

2. What is the correct code for the plastic repair of cleft lip with
nasal deformity when utilizing an Abbe- Estlander cross-lip pedicle
flap?

A. 40527

B. 40761

C. 42200
D. 40700

3. The physician performed a needle biopsy of salivary gland


and sent the aspirate out to pathology for study. What is the
correct CPT code for the physician to report?
A. 42400

B. 10021

C. 42400, 88172

D. 42405, 88173

4.

PROCEDURA

L NOTE

PATIENT:

Ortiz, Pamela

AGE: 13
DATE: 01/19/2016

PREOPERATIVE DIAGNOSIS: Oropharyngeal

Hemorrhage s/p Tonsillectomy

POSTOPERATIVE DIAGNOSIS: Same


PROCEDURE: Control of oropharyngeal hemorrhage
The patient was seen in the emergency room one day after
tonsillectomy due to uncontrolled oropharyngeal hemorrhage. The
patient was admitted to the hospital for control of nasal
hemorrhage and observation. The suture site from tonsillectomy
was visualized and the pharynx was packed with gauze to control
any additional bleeding. The patient remained in hospital for an
additional 24 hours under observation and to control any additional
hemorrhaging. The physician recommended the patient schedule a
follow-up appointment in two days to check the sutures.

How should you code this procedure?

A. 42960
B. 42961

C. 42962

D. 42971

5. A physician performed a thoracoabdominal diaphragmatic


hernia repair on 45-year-old male patient with the implantation of
prosthetic mesh for support. What is the correct code for this
procedure?

A. 39531

B. 39530

C. 43336

D. 43337

6. The physician performed a partial Nissen Fundoplasty via a


lower thoracic incision. What is the correct code for this procedure?
A. 43327

B. 43328

C. 43325

D. 43331
7. A 5-year-old patient received a gastric intubation from
physician after ingesting approximately 5 tablespoons of carpet
cleaning detergent. The gastric tube was orally inserted through
the esophagus and into the stomach and lavage was performed to
evacuate the stomach contents. What is the correct CPT code?

A. 43753

B. 43752

C. 43754

D. 43755

8. A physician performed a revision of a gastrojejunostomy with


intestine resection and vagotomy. What is the correct code for this
procedure?

A. 43850

B. 43855

C. 43860

D. 43865
9. Where are codes for the pancreas located in the CPT manual?

A. Within the endocrine section (60000 to 60699)

B. Within the digestive system section (40490 to 49999)


C. Within the urinary system section (50010 to 53899)

D. There are no codes for the pancreas in the CPT manual

10. A 7-year-old boy fell from a treehouse, cutting open his upper
lip across the upper vermilion border. Due to the vermilion border
being involved, repair of the lip was performed by plastic surgery.
What CPT code should be used for repair of the lip over the
vermilion border?

A. 40650

B. 40652

C. 40654

D. 40700

11. A newborn needed a frenectomy, due to a tongue tie, to allow


her to breastfeed properly. What CPT code should be used for this
procedure?
A. 40806

B. 40819

C. 40820

D. 40899
12. A 10-year-old involved in a MVA lacerated his posterior
tongue. Laceration was 2.4 cm in length. What code should be
used for repair of this child’s tongue?

A. 41250

B. 41251

C. 41252

D. 41599

13. A 25-year-old smoker has a lesion on the right anterior


portion of his tongue. He needs to have a biopsy done of this
lesion. What CPT code should be used for this procedure?

A. 41100

B. 41105

C. 41108

D. 41112
14. A patient has been having recurrent ulcerations on his uvula.
He is scheduled for a biopsy of these ulcerations. What CPT code
should be used?

A. 42000

B. 42104

C. 42100
D. 42140

15. A newborn needs to have a cleft palate repaired. His cleft


involves the soft and hard palates only. What CPT code would
you use for a palatoplasty for this child?

A. 42200

B. 42205

C. 42210

D. 42215

16. A 70-year-old woman has an abscess on her soft palate that


needs to be drained. She is able to have this procedure done in the
ER by otolaryngology. What CPT code should be used?

A. 42000

B. 42100
C. 42104

D. 42140

17. What CPT code should be used for complicated abscess


drainage of the parotid gland?

A. 42300
B. 42305

C. 42310

D. 42320

18. A 64-year-old woman has a sublingual salivary cyst that


needs to be removed. What CPT code should be used for this
procedure?

A. 42400

B. 42405

C. 42408

D. 42409

19. An 8-year-old girl is going to have a tonsillectomy and


adenoidectomy for recurrent strep throats. What CPT code
should be used for this patient’s procedure?
A. 42825

B. 42830

C. 42820

D. 42821
20. A 39-year-old man has been having a recurrent issue, feeling
as though there is a lump in his throat. His ENT performed a
fiberoptic laryngoscopy on him in the office and it was noted that he
had enlarged lingual tonsils. Due to persistent problems and not
responding to medication, he is scheduled for lingual tonsil
excision. What CPT code should be used for this procedure?

A. 42825

B. 42826

C. 42860

D. 42870

21. A patient underwent esophagogastroduodenoscopy (EGD)


and had multiple biopsies taken of the esophagus, stomach and
duodenum. She also had one polyp removed with hot biopsy
forceps from the antrum of the stomach. What CPT code(s) should
be used for this procedure?

A. 43239

B. 43239, 43250
C. 43239, 43251

D. 43239, 43255

22. A patient is undergoing bariatric surgery for morbid obesity.


She has opted to have a gastric band placed, which will be
completed through laparoscopy procedure. She will have the
gastric band and subcutaneous port components placed during
this procedure. What CPT code should be used for this type of
procedure?
A. 43644

B. 43645

C. 43770

D. 43774

23. A patient with colon cancer has to undergo a partial


colectomy with colostomy and creation of mucofistula. What CPT
code should be used for this procedure?

A. 44140

B. 44141

C. 44143

D. 44144

24. A patient who had a colostomy due to colon cancer and


resection is going through their six-month checkup after surgery,
which involves a colonoscopy through the stoma of the colostomy.
During this procedure, the patient has multiple biopsies taken.
What CPT code should be used for this procedure?

A. 44388

B. 44389

C. 44391

D. 44392
25. A 12-year-old patient needs to have an appendectomy
performed for acute appendicitis with rupture and subsequent
generalized peritonitis. Unfortunately, he isn’t a candidate for
laparoscopy and the surgeon will perform an open appendectomy.
What CPT code should be used for this procedure?

A. 44950

B. 44955

C. 44960

D. 44970

26. A patient is scheduled for a laparoscopic appendectomy.


What CPT code should be used for this procedure?

A. 44950

B. 44960

C. 44970
D. 44979

27. A patient has an appendiceal abscess and needs to have


open incision and drainage of it. There will be no removal of the
appendix during this procedure. What CPT code should be used?
A. 44900

B. 44950

C. 44960

D. 44970

28. A patient is undergoing a routine colonoscopy. During his


procedure three polyps were removed by snare technique and sent
for biopsy. What CPT code should be used for this procedure?

A. 45378

B. 45380

C. 45384

D. 45385

29. A 60-year-old woman developed a rectovesical fistula and


needs to have it closed. Because of the extent of her fistula, she
also requires a colostomy. What CPT code is appropriate?
A. 45800

B. 45805

C. 45820

D. 45825
30. A recently diagnosed rectal cancer patient requires a partial
proctectomy with anastomosis, using both the abdominal and
transsacral approach. What CPT code should be used for this
patient?

A. 45113

B. 45114

C. 45116

D. 45116

31. A patient needs to have internal hemorrhoids removed. To


do this he opted to have an internal hemorrhoidectomy by
rubber band ligation. What CPT code should be used?

A. 46221

B. 46945

C. 46220

D. 46255
32. An 82-year-old woman is undergoing an anal
sphincteroplasty for anal incontinence. What CPT code should be
used for this procedure?

A. 46750

B. 46751

C. 46760
D. 46761

33. A young man has a superficial perianal abscess that needs to


be opened and drained. What CPT code should be used for this
procedure?

A. 46040

B. 46045

C. 46050

D. 46060

34. A percutaneous needle biopsy is performed on the liver of


a 45-year-old man who had a CT scan showing a large area of
inflammation. No imaging was performed for this procedure.
What CPT code should be used?

A. 47000

B. 47001
C. 47100

D. 47300

35. A daughter is going to donate the right lobe of her liver to


her mother, as a living donor. What CPT code should be used to
show this type of donor hepatectomy?
A. 47120

B. 47130

C. 47140

D. 47142

36. A patient had a MVA where he lacerated his liver causing a


hemorrhage. He required a complex suture of this liver injury
without hepatic artery ligation. What CPT code should be used?

A. 47350

B. 47360

C. 47361

D. 47362

37. A 32-year-old female is suffering from a poorly functioning


gallbladder. Due to significant symptoms she will have a
laparoscopic cholecystectomy. Because there were no gall stones
present on radiologic films, she will not need to have exploration of
the common duct performed or cholangiography. What CPT code
should be listed for this patient?

A. 47560

B. 47561

C. 47562
D. 47563

38. A patient has a choledochal cyst that needs to be removed.


What CPT code should be used for the removal of this type of
cyst?

A. 47630

B. 47711

C. 47715

D. 47999

39. A patient needs to have a percutaneous biliary endoscopy


with removal of calculi. What CPT code should be used for this
procedure?

A. 47552

B. 47553
C. 47554

D. 47555

40. A patient with pancreatic cancer needs to have peripancreatic


drains placed for acute pancreatitis. What CPT code should be
used for this procedure?
A. 48000

B. 48001

C. 48020

D. 48510

41. A 56-year-old patient has a pancreatic adenoma. She is


scheduled for surgery to have this removed. What is the proper
CPT code for excision of this type of lesion?

A. 48102

B. 48120

C. 48148

D. 48155

42. A 28-year-old male noticed a painful bulging around his belly


button that worsened during and after working out. He was seen by
a general surgeon who recommended laparoscopic surgery for a
reducible umbilical hernia with mesh insertion. What CPT code
should be used for this procedure?

A. 49650

B. 49652

C. 49653

D. 49654
43. A 9-month-old baby has an incarcerated right inguinal
hernia with hydrocele that needs to be repaired. What CPT code
should be used for this procedure?

A. 49492

B. 49496

C. 49501

D. 49507

44. An abdominal paracentesis is performed for a patient who


has ascites to obtain a sampling of the fluid for testing and to
provide relief for the patient’s discomfort. No imaging guidance was
necessary for this procedure. What CPT code should be used for
this procedure?

A. 49060

B. 49082

C. 49083
D. 49084
50,000 SERIES (80
QUESTIONS)

1. A physician endoscopically removed a 0.3 cm calculus from


patient’s right kidney under general anesthesia. Access was
gained through an established nephrostomy. What is the correct
code for this procedure?

A. 50060 -RT

B. 50075 -RT

C. 50561 -RT

D. 50580 -RT

2. A 25-year-old female patient with ESRD received a dual


renal transplant without nephrectomy. What is the correct code for
this transplant?

A. 50360 -50

B. 50360

C. 50365
D. 50365 -50

3. A physician performed a cystoscopy with ureteral stent


removal. What is the appropriate CPT code?

A. 52000, 52310

B. 52310
C. 52000

D. 52315

4. The patient presents to the gynecologist’s office complaining


of stress incontinence. The GYN decides to perform a sling
operation using synthetic materials to reinforce the patient’s
muscles and allow for additional urinary control. This procedure is
performed laparoscopically in order to be less invasive. What is the
correct code for this procedure?

A. 57288

B. 57287

C. 51990

D. 51992

5. A 38-year-old female patient with severe endometriosis


and urethral malignancy received pelvic exenteration with
removal of bladder and ureteral transplantations. The patient
also received a full hysterectomy. What is the correct code for
this service?

A. 58240

B. 51597, 58285

C. 51597

D. 58240, 58285
6. An 87-year-old male patient in a rehab hospital status-post
hip transplant is having trouble voiding. After his last voiding, the
physician measured his bladder capacity using a non-imaging
ultrasound device. How should you code for this procedure?

A. 51741

B. 51798

C. 51798 -50

D. 51792

7. A physician performed an endoscopic resection of a 2.3 cm


bladder tumor. What is the correct CPT code?

A. 52235

B. 52234

C. 52240

D. 52224
8.

PROGRESS

NOTE

PATIENT:

Davis, Levi

DATE:

09/08/2014

MEDICAL

NOTE:
The patient is actively being treated for bladder cancer. The patient
presented to the office today for bladder instillation treatment. The
patient was catheterized and his bladder was emptied of all
contents. An anticarcinogenic agent was introduced into the
bladder at 0100 hours. The patient was instructed to lie in supine
position for 45 minutes. The physician returned at 0145 hours and
emptied the bladder of the anticarcinogenic agent. The patient
tolerated the procedure well. The patient was instructed to return to
office in one week for the next treatment and a follow-up
examination. What is the correct code for the instillation treatment
only?

A. 51715

B. 51701

C. 51702

D. 51720

9. A physician performed a cystourethroscopy with an


ejaculatory duct catheterization and irrigation. Duct radiography
was also performed to visualize ejaculatory duct system. What
CPT codes should be reported?
A. 52000, 52010

B. 52000, 52010, 74440

C. 52010, 74440

D. 52010

10. A physician performed an incision and drainage on a


complicated deep penile abscess. What is the appropriate code
for this procedure?
A. 10061

B. 10081

C. 54015

D. 54105

11. A pediatrician performed a routine circumcision on a


newborn baby boy in a hospital setting. The physician
anesthetized the area with dorsal penile block, clamped the
foreskin away from the tip of the penis and excised the excess
foreskin. What is the correct code for the circumcision?

A. 54160

B. 54150 -47

C. 54150

D. 54160 -47
12. A physician performed an orchiopexy via an inguinal
approach for intra-abdominal testis. What is the correct code for
this procedure?

A. 54692

B. 54650

C. 54690

D. 54640
13. A physician performed a bilateral excision of the vas
deferens for the purpose of sterilization on a 45-year-male. What
is the correct code for this procedure?

A. 55250

B. 55250 -50

C. 55200

D. 55200 -50

14.

PROCEDURAL

NOTE

PATIENT: Ray,

Alexander AGE:

59
DATE: 02/15/2015
PREOPERATIVE DIAGNOSIS: Prostate

Cancer, Primary POSTOPERATIVE

DIAGNOSIS: Same
PROCEDURE:
TURP

A patient was placed in supine position on the operating table,


draped and anesthetized accordingly. Using a resectoscope with
light source, the physician located the prostate and resected the
malignant prostatic tissue with electrocautery knife, leaving the
appropriate margins. The physician removed the resectoscope and
the patient was catheterized for drainage of his bladder contents
and resected prostatic tissue. The patient tolerated the procedure
well and was transferred to postoperative recovery. What is the
correct code for this surgical procedure?
A. 52601 -58

B. 52601

C. 52500

D. 52630

15. A patient with Bartholin’s gland cyst presents to the clinic for
incision and drainage of the cyst. Due to the size and area of the
cyst, the physician decides to incise the cyst and suture the lateral
sides, leaving one side open for drainage. Which of the following
codes would represent the physician’s work?

A. 56420

B. 56740

C. 56440

D. 53060
16. A physician performed a colpopexy with suspension of the
vaginal apex via laparoscope. What is the correct code for the
procedure?

A. 57280

B. 57282

C. 57283

D. 57425
17. A 36-year-old woman with a history of multiple complicated
ectopic pregnancies presented to the clinic for abdominal
hysterectomy. Two days ago, an advanced interstitial uterine
ectopic pregnancy was discovered and after careful consideration
the patient elected to have a full hysterectomy. After the patient
was prepped and anesthetized, the obstetrician performed a full
incisional hysterectomy with removal of ectopic pregnancy, without
removal of ovaries. Patient tolerated the procedure well and was
sent to post-operational recovery.

How should the OB code for this surgical procedure?

A. 58150

B. 59120

C. 59135

D. 59136
18. A 23-year old woman, pregnant with her second child,
received antepartum care from her physician in Atlanta, GA. After 9
visits with her physician in Atlanta, the patient moved to
Albuquerque, NM where she continued her prenatal care with a
new doctor. The new physician saw the patient for the remaining
antepartum visits. The new physician also performed vaginal
delivery and postpartum care, which included the 6-week
postpartum checkup. How should the physician in Atlanta code for
his services?

A. 59426

B. 59425

C. 59410
D. 59430

19. A woman with polyhydramnios presented to the OB/GYN


clinic for amniocentesis with reduction of her amniotic fluid. After
prepping the patient with local anesthetic and with ultrasound
guidance, the physician inserted the needle with therapeutic
drainage system, and drained 18 cc of amniotic fluid from the
patient. The needle was then removed under ultrasound guidance
and surgical site was monitored for bleeding. The patient tolerated
the procedure well and was scheduled to return in one week for
another evaluation and possible treatment of her polyhydramnios.
What code should be reported for these services?

A. 59000

B. 59001, 76946

C. 59001

D. 59000, 76946
20. A 27-year-old pregnant woman presented to the hospital
maternity ward to deliver her third baby. She was 39 weeks
pregnant and has had no complications so far. The patient planned
on a vaginal delivery. Her oldest child was born vaginally and her
second was born via cesarean section. Her OB, who had provided
her antepartum care, was able to successfully complete a vaginal
delivery with no complications. Her OB also provided postpartum
care to the patient. What procedure code should be reported by
the OB?

A. 59400

B. 59510
C. 59620

D. 59610

21. A 27-year-old woman in labor presented to the emergency


room. Upon admission and evaluation, it was determined that the
fetus was in breech position. Due to the severity of the mother’s
contractions, the OB decided that tocolysis was necessary in order
to delay the contractions to allow for an external cephalic version
procedure. The OB and the delivery team of nurses carefully
performed the external cephalic version and successfully turned
the fetus into the appropriate delivery position, at which time the
tocolysis was ceased and the patient was given Pitocin to begin
contractions again. The baby was successfully delivered vaginally
after three additional hours of labor.

How should the OB code for the procedure?

A. 59400

B. 59412

C. 59400 -22
D. 59400, 59412 -51

22. A physician performed a needle aspiration of an infected


thyroid cyst using imaging guidance. The aspirate was sent to the
pathology lab to identify the bacterium within the cyst. What are
the correct procedure codes?

A. 60300, 10021
B. 60300, 76942

C. 60000, 76942

D. 10021, 76942

23. A 61-year-old patient with thyroid cancer received a right


thyroid lobectomy via a transverse cervical incision. Superior and
inferior thyroid vessels were severed in order to remove the thyroid
lobe and the isthmus was severed and resected. How should you
code for this service?

A. 60220

B. 60220 -50

C. 60225

D. 60240

24. PRE-

PROCEDURAL
NOTE PATIENT:

Carter, Clara
AGE: 46

DATE: 04/10/2014

PREOPERATIVE DIAGNOSIS:

Primary Malignancy, Thyroid

PROCEDURE PLANNED:

Thyroidectomy
Patient is being treated unsuccessfully for primary malignancy of
the thyroid and has elected to have a subtotal removal of the left
thyroid. The physician will perform a partial thyroid lobectomy
without isthmusectomy. How should the physician code for the
procedure?
A. 60212 -LT

B. 60210 -LT

C. 60220 -LT

D. 60210 -50

25. PRE-

PROCEDURAL

NOTE PATIENT:

Carter, Clara
AGE: 48

DATE: 04/18/2016

PREOPERATIVE DIAGNOSIS:

Primary Malignancy, Thyroid

PROCEDURE PLANNED:

Thyroidectomy
An female patient, who received a subtotal thyroidectomy two
years ago, presented to the operating room today for a complete
removal of all the remaining thyroid tissue.
The physician originally treated the patient for a primary
malignancy of the thyroid by removing the malignant section of the
thyroid. The malignancy has now returned and the patient has
elected to have all of the remaining thyroid tissue removed. What is
the correct code for this service?

A. 60254

B. 60270

C. 60252

D. 60260
26. A physician performed a biopsy of the thyroid with the use of
a percutaneous core needle, using fluoroscopic guidance. Aspirate
was sent to the laboratory so that the pathology of the disease
could be determined. What is the correct code for this procedure?

A. 60100, 76942, 88172

B. 60100, 77002, 88173

C. 60100, 77002

D. 60000, 88173

27. What is the correct code for a parathyroidectomy with


parathyroid autotransplantation performed on a 42-year-old male
patient?

A. 60512

B. 60500, 60512

C. 60500, 60512 -51

D. 60500
28. A physician performed a radical bilateral adrenalectomy with
excision of an adjacent retroperitoneal tumor. What is the correct
code this surgery?

A. 60540, 50323
B. 60540

C. 60545

D. 60545, 50323

29. What is the correct code for a surgical laparoscopy with partial
right adrenalectomy?

A. 60650 -RT

B. 60650 -50

C. 60540 -RT

D. 60540

30. A 25-year-old woman has five kidney stones in the right


kidney. Due to history of requiring surgery to remove stones when
they have become stuck, she undergoes lithotripsy to break up the
stones to easily pass them. What CPT code should be used for this
procedure?
A. 50580

B. 50590

C. 50592

D. 50593
31. A patient needs to have an indwelling ureteral stent placed.
To do so, she will have to have an ureterotomy performed. What
is the correct CPT code for this procedure?

A. 50600

B. 50605

C. 50610

D. 50620

32. A 6-month-old girl with recurrent urinary tract infections is


found to have a congenital anatomical anomaly. Her ureters are not
fully connected to the bladder. She is scheduled for bilateral
ureteroneocystostomy. What CPT code should be used for this
procedure?

A. 50780

B. 50785

C. 50780, -50

D. 50785, -50
33. A 68-year-old man has to have his urethral stricture dilated
with a urethral dilator. This will be his first time having this
procedure done. What CPT code should be used for this
procedure?

A. 53600

B. 53601
C. 53605

D. 53660

34. A 32-year-old woman has an abscess of the Skene’s


gland and this requires drainage. She will be scheduled for
outpatient surgery for this procedure. What CPT code would
you use?

A. 53040

B. 53060

C. 53080

D. 53085

35. A 25-year-old man decides that he wants to be circumcised


for religious reasons and for increasing discomfort. He sees
urology, who agrees to perform the procedure, and is set up for a
circumcision as an outpatient. What CPT code should be used for
this procedure?
A. 54150

B. 54160

C. 54161

D. 54162
36. A 21-year-old male has to have surgery to reduce a testicular
torsion with fixation of contralateral testis. What CPT code should
be used for this procedure?

A. 54550

B. 54600

C. 54620

D. 54640

37. A 36-year-old man has to have a radical orchiectomy due to a


tumor on the left side. This is done without abdominal exploration
through the inguinal approach. What CPT code should be used for
this procedure?

A. 54520

B. 54522

C. 54530

D. 54535
38. A 34-year-old man has a spermatocele that has been getting
larger and is now causing constant discomfort. He needs to have
this removed but fortunately will not need to have an
epididymectomy. What CPT code should be used for this
procedure?

A. 54800
B. 54830

C. 54840

D. 54860

39. Due to severe epididymitis a 38-year-old man has to have


a bilateral epididymectomy. What CPT code should be used for
this procedure?

A. 54830

B. 54860

C. 54861

D. 54865

40. A 28-year-old man who had a vasectomy wishes to have this


reversed. He is scheduled for a bilateral epididymovasostomy.
What CPT code should be used for this procedure?
A. 54861

B. 54865

C. 54900

D. 54901
41. A patient has a hydrocele that needs to be aspirated. He will
have a puncture aspiration performed without any medication
injected. What CPT code should be used for this procedure?

A. 55000

B. 55040

C. 55041

D. 55060

42. A 23-year-old male received a testicular injury and as a result


ended up with bilateral hydroceles. Over time these began to
become painful and he has elected to have both of these
hydroceles removed. What CPT code should be used for this
procedure?

A. 55000

B. 55040

C. 55041

D. 55060
43. A man is scheduled for repair of his tunica vaginalis
hydrocele using the bottle procedure. What CPT code should be
used for this procedure?

A. 55000

B. 55040
C. 55041

D. 55060

44. A 32-year-old man has an abscess in his scrotal wall that


needs to be drained. What CPT code should be used for this
procedure?

A. 55100

B. 55110

C. 55120

D. 55150

45. An 18-year-old male is scheduled for a simple scrotoplasty


due to significant scrotal injury during a motor vehicle accident.
What CPT code should be used for this procedure?

A. 55110
B. 55120

C. 55175

D. 55180

Answer: C - 55175 is the correct code for a simple scrotoplasty.


Code 55110 is incorrect because this is the code for scrotal
exploration. Code 55120 is incorrect because this code is for
foreign body removal within the scrotum. Code 55180 is incorrect
because this code is used for a complicated scrotoplasty.

47. A 35-year-old man decides to have a bilateral vasectomy.


What CPT code should be used for this procedure?

A. 55200

B. 55250

C. 55300

D. 55400

48. A man is getting ready to have bilateral vas deferens ligated


percutaneously. What CPT code should be used for this
procedure?

A. 55200
B. 55250

C. 55400

D. 55450
49. A young man needs to have a hydrocele of his spermatic
cord removed. What CPT code should be used for this procedure?

A. 55500

B. 55520

C. 55530

D. 55550

50. A young man is undergoing a surgical laparoscopy to ligate


the spermatic veins for a varicocele. What CPT code should be
used for this procedure?

A. 55520

B. 55530

C. 55550

D. 55559
51. A 42-year-old man is scheduled to have a varicocele excised
and have the spermatic veins ligated with hernia repair at the
same time. What CPT code should be used for this procedure?

A. 55520

B. 55530

C. 55535
D. 55540

52. A 42-year-old man is scheduled for a bilateral vesiculotomy


that is simple and is not complicated. What CPT code is used for
this procedure?

A. 55600

B. 55600, -50

C. 55559

D. 55559, -50

53. A young man is scheduled to have a Mullerian duct cyst


removed after it was visualized on a pelvic ultrasound and
abdominal MRI. What is the correct CPT code for this excision?

A. 55600

B. 55605
C. 55650

D. 55680

54. A 70-year-old man is going to have a direct vesiculectomy


performed on the right seminal vesicle. What is the correct CPT
code for this procedure?
A. 55600

B. 55605

C. 55650

D. 55680

55. A 65-year-old man has a growth on his prostate. He is


scheduled for a needle biopsy to identify the growth. What CPT
code should be used for this type of a biopsy?

A. 55700

B. 55705

C. 55706

D. 55720

56. A 67-year-old man is scheduled for a radical perineal


prostatectomy with bilateral pelvic lymphadenectomy including
external iliac, hypogastric and obturator nodes all occurring
during the same procedure.

A. 55801

B. 55810

C. 55812

D. 55815
57. A young woman has an abscess in her Bartholin’s gland.
She is able to have this incised and drained at her gynecologist’s
office. What CPT code can be used for this procedure?

A. 56405

B. 56420

C. 56440

D. 56441

58. A young female had to have a partial hymenectomy due to


an imperforate hymen. What CPT code should be used for this
procedure?

A. 56442

B. 56620

C. 56700

D. 56800
59. A 22 year old had a positive pap smear and must undergo
colposcopy within the gynecologist’s office. During this procedure,
multiple biopsies of the cervix will be taken. What is the correct
CPT code for this procedure?

A. 57200
B. 57282

C. 57420

D. 57421

60. A 55-year-old woman developed a rectovaginal fistula and


is scheduled to have this closed by a vaginal approach. What is
the correct CPT code for this procedure?

A. 57300

B. 57305

C. 57310

D. 57320

61. A postpartum mother is scheduled to have an intrauterine


device (IUD) inserted. What is the correct CPT code for this
procedure?
A. 58300

B. 58301

C. 58321

D. 58346
62. A 48-year-old woman is scheduled for a total abdominal
hysterectomy with removal of tubes and without removal of
ovaries. What CPT code is appropriate for this procedure?

A. 58150

B. 58200

C. 58260

D. 58285

63. A postpartum woman decides to have her tubes tied. She


opts to wait until six weeks postpartum to have this procedure
performed because she had a rough delivery. What is the correct
CPT code for a bilateral tubal ligation when not performed at the
same time as the postpartum hospitalization?

A. 58600

B. 58605

C. 58611

D. 58611
64. A 24-year-old woman is scheduled to have surgical
laparoscopy with lysis of fallopian tube adhesions. What is
the correct CPT code for this procedure?

A. 58660

B. 58661
C. 58671

D. 58673

65. A young woman is set to undergo a fimbrioplasty to help


repair the portion of the fallopian tube tissue near the ovary that
it meant to sweep the egg into the fallopian tube. What is the
correct CPT code for this procedure?

A. 58672

B. 58673

C. 58760

D. 58770

66. A young female has multiple ovarian cysts. There is a large


cyst approximately 10 cm in diameter on the right ovary. Due to
discomfort, it is recommended to drain this cyst using a vaginal
approach. What is the CPT code for this procedure?
A. 58800

B. 58805

C. 58820

D. 58822
67. A 56-year-old woman has been diagnosed with right ovarian
cancer and is scheduled for a total right oophorectomy with pelvic
lymph node biopsies with salpingectomy and omentectomy. What
is the correct CPT code for this procedure?

A. 58940

B. 58943

C. 58950

D. 58953

68. A 50-year-old woman is scheduled for a unilateral biopsy of


the right ovary. What is the correct CPT code for this procedure?

A. 58820

B. 58900

C. 58920

D. 58925
69. A 30-year-old woman discovered a lump on her thyroid,
which turned out to be a malignancy. Her surgeon recommends
total thyroidectomy with limited neck dissection to eliminate all
cancerous tissue. What code should be used for this procedure?

A. 60220
B. 60240

C. 60252

D. 60254

70. A middle-age woman is scheduled for a


parathyroidectomy. What is the correct code for this
procedure?

A. 60500

B. 60502

C. 60505

D. 60512

71. A patient who has already had one exploration of her


parathyroid(s) is scheduled for a re-exploration six months later.
What CPT code should be used for the second procedure?
A. 60500

B. 60502

C. 60505

D. 60512
72. A woman with myasthenia gravis is scheduled for a
total thymectomy using the transcervical approach. What CPT
code should be used for this procedure?

A. 60520

B. 60521

C. 60522

D. 60540

73. Thymoma are present in a woman’s thymus. She is


scheduled for a transthoracic total thymectomy with radical
mediastinal dissection. What is the correct CPT code for this
procedure?

A. 60520

B. 60521

C. 60522

D. 60650
74. A patient is scheduled for a thymectomy for myasthenia
gravis without radical mediastinal dissection and will be performed
through a sternal split. What is the CPT code for this procedure?

A. 60520

B. 60521

C. 60522
D. 60540

75. A young woman had a transabdominal surgical laparoscopy


with complete adrenalectomy. What is the CPT code for this
procedure?

A. 60540

B. 60545

C. 60650

D. 60659

76. A patient needs to have a dorsal exploration of the adrenal


gland with biopsy. What is the correct CPT code for this
procedure?

A. 60540

B. 60545
C. 60650

D. 60659

77. A middle-age man needs to have a complete adrenalectomy


with excision of an adjacent retroperitoneal tumor. His surgeon
plans to use a transabdominal approach to perform this procedure
and will take biopsies of the surrounding tissue.
A. 60540

B. 60545

C. 60650

D. 60659

78. A 52-year-old man has a chemodectoma (also known as a


carotid body tumor) that is aggressive. It is recommended to have
this removed. Because of how aggressive it is, the carotid artery is
also excised. What CPT code should be used for this procedure?

A. 60600

B. 60605

C. 61703

D. 61705
79. A young boy has a carotid body tumor that was just
discovered. It is a simple tumor at this time and is not very
aggressive. No excision of the carotid artery will have to be
performed with this tumor. What is the correct CPT code for this
procedure?

A. 60600

B. 60605

C. 61703
D. 61705

80. A patient with a carotid body tumor and inflammation around


the carotid body has had an ultrasound and MRI with imaging that
indicates a larger amount of inflammation than normal in this area.
His surgeon elects to undergo laparoscopy of this area to obtain
more information before scheduling surgery to remove the tumor,
to better prepare for whether the carotid artery will have to be
excised as well. The patient is scheduled for surgical laparoscopy
to explore the carotid body. What is the correct CPT code for this
procedure?

A. 60600

B. 60605

C. 60659

D. 60699
60,000 SERIES (20
QUESTIONS)

1. Surgeries of the skull base are often categorized


according to three things. Which of the following includes the
three things by which surgeries of the skull base are
categorized?

A. Approach procedure, biopsy, and definitive procedure

B. Gaining access to the lesion, approach procedure, and


definitive procedure

C. Definitive procedure, approach procedure, and repair or


reconstruction

D. Definitive procedure, repair, and reconstruction

2.

PROCEDUR

AL NOTE

PATIENT:

Lopez, Olga
AGE: 76

years
DATE: 11/05/2014

PATIENT DIAGNOSIS: Multiple Skull Lymphoma

PROCEDURE: Craniotomy converted to

craniectomy of left anterior cranial base

ANESTHESIA: General endotracheal

The patient was placed in supine position on operating table and


anesthesia was successfully administered. The patient was
prepped in the usual manner. An incision was made on the midline
of the patient’s anterior cranial base and the surgeon dissected the
epidermal layer to reveal the skull. Three .2 to .4 cm lymphomas
were located and the skull bone was excised in one piece to
remove the affected are as leaving a .1 cm margin. Halfway
through the procedure the patient’s blood pressure dropped. It was
difficult to control throughout the remaining procedure. Due to the
patient’s drop in blood pressure, the surgeon decided to convert
the procedure to a craniectomy, so that bone grafts were not
placed. A drain was placed beneath the remaining skull base and
the edges of the skin were then sutured back together using 4-0
vicryl sutures. A sterile dressing was placed on the excision site.
The patient was removed from endotracheal anesthesia and
remained under physician supervision until her blood pressure
stabilized. The patient was taken to the recovery room and
scheduled for a bone graft at a later date. What is the correct code
for this surgery?

A. 61500 -52

B. 61518

C. 61500 -53

D. 61500

3. A physician performed craniotomy on a patient with a severe


head trauma and intracerebral hematoma. Due to the patient’s
condition the procedure was extremely difficult, requiring a
significant amount of extra time and effort. What is the correct
code for this procedure?

A. 61315
B. 61313

C. 61313 -22

D. 61315 -23

4. A physician utilized a laser and neuroendoscopic


guidance to create a duct from the third lateral ventricle to the
cisterna magna as a treatment for hydrocephalus. What is the
correct code for this procedure?
A. 62200

B. 62201

C. 62220

D. 62180

5. A pediatrician performed a lumbar puncture on a 2-day-old


premature infant weighing 2.3 kg with possible meningitis. What
is the correct code for this service?

A. 62272

B. 62272

C. 62270 -63

D. 62272 -63

6. A physician performed a right lumbar hemilaminectomy with


decompression of nerve root, including the excision of two
herniated intervertebral discs for three vertebral interspaces. How
should the physician code for this service?

A. 63030, 63035 (X2)

B. 63020, 63035 (X2)

C. 63042, 63044 (X2)

D. 63042, 63044
7. A 59-year-old patient with continuous post-traumatic pain
received a tunneled epidural catheter for long-term pain medication
administration by use of an external pump. The patient was
previously implanted with an intrathecal pump, which was
ineffective for the patient’s pain management. Before the
implantation of the epidural pump, the physician removed the
intrathecal catheter. What are the correct CPT codes for this
service?

A. 62350

B. 62350, 62355 -51

C. 62351, 62355

D. 62350, 62355

8. A physician performed the excision of an intra spinal intra


dural neoplasm of the sacral area. What is the correct code for this
procedure?
A. 63275

B. 63275

C. 63278

D. 63283
9. A pediatric neurologist performed a repair of 3.4 cm
meningocele on 40-day-old infant with spina bifida weighing 3 kg.
Due to the location, size and depth of the meningocele, complex
closure of 7 cm wound was required. What are the correct codes
for this procedure?
A. 63700 -63, 13101 -51

B. 63700, 13120

C. 63700 -63, 13101

D. 63700, 13101 -51

10. A physician sutured 3 digital nerves in the left hand, the


common sensory nerve in the left foot and two common sensory
nerves in the right foot. What codes need to be reported?

A. 64831, 64832 (X2), 64834, 64837 (X2)

B. 64831, 64832, 64834, 64837

C. 64831, 64832 (X4), 64834

D. 64831, 64834, 64837 (X4)


11. A young man has to have a lumbar spinal tap performed to
obtain cerebrospinal fluid. This will be done with a needle for
diagnostic purposes so that the fluid can be tested. What is the
correct code for this procedure?

A. 62269

B. 62270
C. 62272

D. 62273

12. A patient needs to have a lumbar laminectomy without


discectomy. Which code is correct for this procedure?

A. 63001

B. 63003

C. 63005

D. 63011

13. A 40-year-old man needs to have a percutaneous


implantation of a neurostimulator electrode array to the trigeminal
nerve, which is a cranial nerve. What code is correct for this
procedure?

A. 64550
B. 64553

C. 64555

D. 64561

14. A 62-year-old man needs to have a trigeminal nerve block to


help with the pain caused by shingles. What CPT code should be
used for this procedure?
A. 64400

B. 64402

C. 64405

D. 64408

15. During oral surgery, the lingual nerve was transected.


What CPT code should be used for this transection?

A. 64732

B. 64734

C. 64740

D. 64742

16. A 53-year-old woman has been having significant sciatica.


She is going to have a neuroplasty of the sciatic nerve. What is the
correct code for this procedure?
A. 64702

B. 64704

C. 64708

D. 64712
17. A 26-year-old man who had right ACL reconstruction surgery
received a femoral nerve block. What CPT code should be used for
the femoral nerve block?

A. 64445

B. 64447

C. 64449

D. 64450

18. A patient needs to have an anesthetic agent injected into


the superior hypogastric plexus nerve. What CPT code should be
used for this procedure?

A. 64505

B. 64510

C. 64517

D. 64520
19. A 74-year-old man with pancreatic cancer is going to have
his celiac plexus nerve destroyed by a neurolytic agent without
radiologic monitoring, to help with the intense pain that is caused
by his cancer. What is the correct code for this procedure?

A. 64646
B. 64650

C. 64680

D. 64681

20. A middle-age man, who has significant peptic ulcers, is going


to have a vagotomy to help reduce stomach acid production. He is
going to have the vagus nerve transected using an abdominal
approach. What is the correct code for this procedure?

A. 64755

B. 64760

C. 64763

D. 64766
ANATOMY (32
QUESTIONS)

1. An umbilical hernia protrudes from which of the following body


areas?

A. The inguinal space in the groin area

B. The esophageal space in the abdomen

C. The belly button

D. The testes

2. A patient with an injured neck sustained an injury to which of


the following areas of the spine?

A. Cervical

B. Lumbar

C. Coccygeal

D. Thoracic
3. Which of the following is a type of connective tissue?

A. Adipose tissue

B. Epithelial tissue

C. Nerve tissue

D. Cardiac tissue
4. The pancreas is part of which two body systems?

A. The respiratory and gastrointestinal

B. The urinary and endocrine

C. The endocrine and gastrointestinal

D. The reproductive and lymphatic

5. Which of the following is the site of sperm maturation?

A. Seminal vesicles

B. Prostate

C. Sperm ducts

D. Epididymis
6. Which of the following answers below identifies all three
different types of blood cells?

A. Neutrophils, Eosinophils, and Basophils

B. Monocytes, Leukocytes, and Platelets

C. Erythrocytes, Leukocytes, and Thrombocytes


D. Erythrocytes, Monocytes, and Leukocytes

7. Spongy bone tissue:

A. Is found along the shafts of hard bones

B. Is found along the outer layer of bones

C. Makes up the top edges of long bones

D. Makes up the insides of bones

8. Inflammation of tendons or the connective tissue that


attaches tendons to muscles or bones is referred to as:

A. Tendonitis

B. Scoliosis

C. Gout

D. Bursitis
9.
The three types of muscles are: smooth,
skeletal, and .

A. Compact

B. Cardiac

C. Contractile
D. Flexion

10.
Jaundice is a condition caused by an excess of in
the body, which causes a yellowing of the skin.

A. Uric acid

B. Bilirubin

C. Melanin

D. Carotene

11. A patient is injured in a motor-cross accident and


shattered two of his vertebrae. Now he is paralyzed from the
chest down and can move his arms, but not his legs. At what
level is his injury?

A. Cervical

B. Lumbar
C. Thoracic

D. Sacral

12. The tympanic membrane is often referred to as:

A. A taste bud
B. The ear drum

C. The stirrup

D. The inner ear

13. What is a result of a nasal polyp?

A. It will cause nose cancer in the affected patient

B. It can change the shape of the nose and necessitate


rhinoplasty

C. It results in the release of histamine causing an allergic


reaction

D. It can obstruct the nasal passageway making it difficult to


breathe

14. Which of the following is a type of cyanosis that presents on


the fingers and toes and may indicate a need for oxygen therapy
for the patient?
A. Central cyanosis

B. Peripheral cyanosis

C. Medial cyanosis

D. Embolytic cyanosis

15. The term “alopecia” is commonly referred to as what?


A. Athlete’s foot

B. Hair loss

C. Heat rash

D. Head lice

16. When a doctor manipulates a dislocated joint back into place,


he:

A. Reduces the subluxation

B. Manipulates the fracture

C. Suspends the dislocation

D. Reduces the suspension

17. What is the difference between a craniotomy and a


craniectomy?
A. A craniectomy removes a part of the skull and allows the
surgeon to perform the craniotomy

B. A craniotomy replaces the part of the skull that was removed;


a craniectomy does not replace it

C. A craniotomy allows access into the patient’s spinal column;


a craniectomy is repairs the patient’s spinal column
D. There is no difference between the two procedures
18. If a doctor performs only the antepartum portion of a patient’s
obstetrical care, when did she take care of the patient?

A. Before the patient delivered the baby

B. During the patient’s delivery

C. After the patient’s delivery

D. Before, during, and after the patient’s delivery

19. If a patient is in the prone position, he is:

A. Lying flat on his back

B. Lying flat on his stomach

C. Sitting up straight

D. Lying flat on his back with his feet elevated


20. The superior eyelid is also referred to as the:

A. Lower eyelid

B. Edge of the eyelid that holds the lower lashes

C. The site of the tear duct


D. Upper eyelid

21. A doctor performed a partial pneumonectomy on a patient. This


procedure included:

A. Removing part of the patient’s lung

B. Cutting into the patient’s abdomen

C. Creating an opening in the patient’s chest

D. Removing a patient’s entire spleen

22. What does the suffix “-megaly” refer to, as in cardiomegaly?

A. Inflammation, as in inflammation of the heart

B. Pain in, as in heart pain

C. Enlargement of, as in enlargement of the heart

D. Softening of, as in softening of the heart


23. What is the medical term for a heart attack, which refers to the
death of heart muscle?

A. Cardiogenic shock

B. Pleural effusion

C. Tachycardia
D. Myocardial infarction

24. The nasal turbinates are part of which anatomical system?

A. Digestive

B. Respiratory

C. Cardiovascular

D. Integumentary

25. Which of the following is a bruise?

A. Contusion

B. Hematoma

C. Hemorrhage

D. All of the above


E. Only answers A and B

26. What is the difference between a laparoscopy and a


laparotomy?

A. One is an open procedure and the other is a closed procedure


B. One is a percutaneous procedure and the other is a closed
procedure

C. One involves the use of an endoscope and the other is a


cutting procedure

D. One involves the use of an endoscope and the other uses a


laparoscope

27. What are the three major bones found in the arm?

A. Ulna, Radius, Humerus

B. Tibia, Fibula, Humerus

C. Tibia, Fibula, Femur

D. Humerus, Tibia, Radius

28. Which of the following is a non-bony compartment between


two vertebral segments includes the intervertebral disc?
A. Vertebral Interspace

B. Paravertebral Facet Joint

C. Vertebral Corpus

D. Vertebral Foramen

29. The bones inside of the nose that are shaped like a spiral
shell serve to humidify, filter, and warm the air breathed in through
the nose and are referred to as:
A. Inferior palate

B. Septal shells

C. Nasal polyps

D. Nasal turbinates

30. The region that separates the lungs and holds the heart,
esophagus, trachea, and great vessels, is referred to as the what?

A. Diaphragm

B. Mediastinum

C. Visceral pleura

D. Parietal pleura

31. Diaphragmatic hernia repair codes are divided based upon


what?
A. The age of the patient and whether or not mesh was used

B. The age of the patient and whether or not the hernia is acute
or chronic

C. The stage of the hernia and the site of the hernia

D. The age of the patient and the site of the hernia


32. The diaphragm is the major muscle that controls breathing. Is
this muscle voluntary or involuntary?

A. Voluntary

B. Involuntary

C. Both

D. Neither
ANESTHESIA (15
QUESTIONS)

1.
The time reported for an anesthesia service begins ,
and ends
.

A. When the anesthesiologist administers the anesthetic agent;


when the patient leaves the operating table.
B. When the anesthesiologist begins prepping the patient;
when the anesthesiologist is no longer in personal post-operative
attendance.
C. When the physician begins the procedure; when the physician
ends the procedure.

D. When the anesthesiologist begins prepping the patient; when


the patient leaves the hospital.

2.

PROCEDURA

L NOTE

PATIENT:
Lopez, Olga

AGE: 76

years
DATE: 11/05/2014

PATIENT DIAGNOSIS: Multiple skull lymphomas

PROCEDURE: Craniotomy converted to

craniectomy of left anterior cranial base

ANESTHESIA: General endotracheal

The patient was placed in supine position on operating table and


anesthesia was successfully administered. The patient was then
prepped in the usual manner. An incision was made on the midline
of the patient’s anterior cranial base and the surgeon dissected the
epidermal layer to reveal the skull. Three .2 to .4cm lymphomas
were then located and the skull bone was excised in one piece to
remove the affected areas, leaving .1cm margins. Halfway through
the procedure the patient’s blood pressure
dropped, which was difficult to control for the remainder of the
procedure. Due to the patient’s drop in blood pressure, the
surgeon decided to convert the procedure to a craniectomy,
therefore bone grafts were not placed. A drain was placed beneath
the remaining skull base and the edges of the skin were then
sutured back together using 4-0 vicryl sutures. A sterile dressing
was placed on the excision site.
The patient was then removed from endotracheal anesthesia and
remained under physician supervision until her blood pressure
stabilized. The patient was then taken to the recovery room and
scheduled for a bone graft at a later date.

In the above procedural scenario, the anesthesiologist spent 30


minutes prepping and administering the anesthesia to the patient.
The surgeon spent 2 hours 15 minutes performing the procedure
and then another hour and a half was spent after the patient was
removed from anesthesia and returned to post- operative recovery.
The anesthesiologist supervised the patient in post-op recovery for
an additional 30 minutes.

How much time should the anesthesiologist report for his service?

A. 4 hours 45 minutes

B. 2 hours 15 minutes
C. 4 hours 15 minutes

D. 3 hours 45 minutes

3. Anesthesia that is administered intravenously, results in a


complete loss of consciousness and affects the entire body is
referred to as what type of anesthesia?

A. Regional
B. Local

C. General

D. Intrathecal

4. Ms. McClinton was administered anesthesia by an


anesthesiologist in her dentist’s office when she was seen for her
dental procedure. The dentist usually performs the procedure with
a local anesthetic, but anesthesia was administered in this
circumstance due to the patient’s extreme anxiety. What would be
the correct modifier for the procedure?

A. -23

B. -22

C. -47

D. -26

5. The patient, a 35-year-old female with well-controlled Type I


diabetes mellitus, received anesthesia for a regular vaginal
delivery with no complications. Code for the anesthesia services
only.

A.

01961 -

P3 B.

62310

C.

01960 -

P3 D.

01960 -

P2
6. A physician harvested a viable left cornea, liver, and heart
from a declared brain-dead patient. What anesthesia services
should have been provided?

A. 01990

B. No anesthesia services should have been

performed on a brain-dead patient C. 33930,

47133-51, 65110-51
D. 01990-P6

7. A physician performed a total hip arthroplasty and an


excision of a patellar bone spur. Code for the anesthesia services
only.

A. 01400, 01214

B. 01380, 01400

C. 01214

D. 01215, 01400
8. A physician performed burn debridement on a 75-year-old
male, who sustained third degree burns over 13% of his body,
while burning dead brush in his back yard. Code for the anesthesia
services only.

A. 01951, 01952, 01953

B. 01951, 01953 (X2), 99100


C. 01952, 01953, 99100

D. 01952, 01953 (X2)

9. A 38-year-old female patient presented to the office for an


extended ophthalmoscopy with retinal drawing. The physician also
performed interpretation and report of the findings. The physician
performing the procedure also performed the anesthesia service
for the patient, due to the fact that the anesthesiologist was not
available. What are the correct codes for the procedure?

A. 00148-47

B. 92225-47

C. 92225, 00148-47

D. 92225-47, 00148

10. A 48 year-old-man suffered internal injuries as well as


multiple lower body fractures in a multiple-car accident, and
presented in an extremely emergent situation. The patient’s liver
sustained damage and was hemorrhaging. Surgery began
immediately, as the patient was not expected to survive without
immediate surgery. During the surgery, the orthopedic surgeon
manipulated the patient’s tibia fracture and set the bone with a
percutaneous fixation. Code for the anesthesia services only.

A. 01462, 00794-P4

B. 01462,

00792 C.

00792-P5
D. 00792-P5, 01462
11. A 17 year-old patient presented to the orthopedist office after
having fallen off his skateboard. He suffered a closed fracture to
the left tibia and fibula as well as a severe sprain to the right wrist.
The patient was in severe pain so an anesthesiologist
administered anesthesia to him, while the physician manipulated
the fractures back into place and applied the cast to the lower left
leg. What are the appropriate anesthesia code(s)?

A. 01820, 01490

B. 01462

C. 01480, 01820

D. 01490

12.

PROCEDURA

L NOTE

PATIENT:
Lopez, Olga

AGE: 76 years
DATE: 11/05/2014

PATIENT DIAGNOSIS: Multiple skull lymphomas

PROCEDURE: Craniotomy converted to

craniectomy of left anterior cranial base

ANESTHESIA: General endotracheal

The patient was placed in supine position on operating table and


anesthesia was successfully administered. The patient was then
prepped in the usual manner. An incision was made on the midline
of the patient’s anterior cranial base and the surgeon dissected the
epidermal layer to reveal the skull.
Three .2 to .4cm lymphomas were then located and the skull bone
was excised in one piece to remove the affected areas, leaving
.1cm margins. Halfway through the procedure the patient’s blood
pressure dropped, which was difficult to control for the remainder
of the procedure. Due to the patient’s drop in blood pressure, the
surgeon decided to convert the procedure to a craniectomy,
therefore bone grafts were not placed. A drain was placed beneath
the remaining skull base and the edges of the skin were then
sutured back together using 4-0 vicryl sutures. A sterile dressing
was placed on the excision site.
The patient was then removed from endotracheal anesthesia and
remained under physician supervision until her blood pressure
stabilized. She was then taken to the recovery room and scheduled
for a bone graft at a later date.

A. 00210, 99135

B. 00211, 99100

C. 00192, 99135, 99100

D. 00192, 99100

13. When reporting the time involved for an anesthesia


procedure when do you start and stop the clock? What is the
correct answer?

A. Time starts when the patient is in the pre-operative


waiting area before the anesthesiologist is present and ends at
the end of the surgery time.
B. Time starts when the patient is in the operating room and the
anesthesiologist begins to prepare the patient for the induction of
the anesthesia and ends in the operating room at the end of the
surgery time.
C. Time starts when the patient is in the operating room and
the anesthesiologist begins to prepare the patient for the induction
of the anesthesia and ends when the anesthesiologist is no longer
in attendance after reporting to the nurses in the PACU (post-
anesthesia care unit).
D. Time starts when the patient is in the operating room
before the anesthesiologist is in attendance and ends in the
PACU after the anesthesiologist gives report to the post-
anesthesia care unit nurses.

14. It is possible to have more than one qualifying circumstances


for an anesthesia code. What qualifying circumstance codes
should be added onto the procedure code for lower abdominal
hernia repair in a child younger than one year of age who became
significantly hypotensive during the procedure, causing an
emergency condition?

A. 00834, 99100, 99135

B. 00834, 99100, 99140

C. 00834, 99140, 99135

D. 00834, 99140

15. A 54-year-old man is having a coronary artery bypass


grafting with pump oxygenator where the anesthesiologist is
also using total body hypothermia. What is the correct code for
this procedure?

A. 00566, 99116

B. 00566, 99135

C. 00567, 99116

D. 00567, 99135
CODING GUIDELINES (19 QUESTIONS)

1. In an outpatient setting, what is the primary diagnosis?

A. The reason the patient came in for the visit

B. The diagnosis code that the doctor lists first

C. The problem that causes the patient the most pain

D. The diagnosis that will result in the most reimbursement

2. Which of the following is NOT typically included in a global


package?

A. The surgical procedure

B. Follow-up appointments related to the procedure, within the


global period

C. Appointments for problems that are not related to the


procedure

D. Appointments for problems that are related to the procedure


3. Unbundling a code is:

A. Unwrapping a code to open a new code

B. Listing multiple procedures, services and supplies with their


own separate, distinct codes

C. Using one code to report a variety of services

D. Using a new code to report previous services


4. When a CPT code has the words “separate procedure” in
parenthesis after the code description, you:

A. Do not use this code if it is listed as a separate procedure

B. Code for all other elements of the procedure except this one

C. Only code for this procedure if it was the only thing performed

D. Code for this procedure, even if it was not performed

5. Which of the following is a surgery section CPT code?

A. 99212

B. 85025

C. 11000

D. 70020
6. When you see the symbol # in front of a CPT code, what does
it mean?

A. That the code is listed out of numerical order

B. That the code is listed in numerical order

C. That the code used to be listed with a different number


D. That the code description has changed

7. When you include an unlisted service procedure code on a


claim, what else must you also include on the claim?

A. A Category III code

B. A Category II code

C. A special report

D. All diagnostic studies

8. In the CPT manual, Appendix C lists clinical examples of:

A. Surgical procedures

B. Radiological procedures

C. Dermatology procedures

D. Evaluation and management procedures


9. Which of the following steps is NOT needed before an
unlisted services procedure code can be included on a claim?

A. Review the CPT manual to make sure a more appropriate


code does not exist
B. Review Category II codes to make sure an appropriate code
does not exist

C. Review Category III codes to make sure an appropriate code


does not exist

D. Check to see if a modifier is appropriate to include with your


code

10. The respiratory system subsection in the CPT manual


contains, but is not limited to procedure codes for the following
body areas:

A. Nose, mouth, and throat

B. Nose, accessory sinuses, and trachea/bronchi

C. Accessory sinuses, mouth, and stomach

D. Lungs, pleura, and heart

11. Mr. Jones was examined after a car accident as a


requirement of his car insurance claim. Which modifier is
appropriate to use on the claim?

A. -22

B. -51

C. -99

D. -32
12. A physician performed a complex repair of the scalp
measuring 7.2 cm and an intermediate repair of the arm measuring
3.4 cm. The repairs are different complexities, so they would be
reported with separate repair codes, but you would need to add a
modifier to indicate that they are two separate procedures
performed on the same date of service. Which modifier would you
use?

A. -59

B. -51

C. -25

D. -24

13. A 32-year-old woman with repeat urinary tract infections


presented to the office with painful urination. Suspecting another
UTI, the physician sent a urine sample to the lab to identify the
bacteria causing the infection. The physician’s office billed for the
laboratory service themselves, even though they sent the sample
to an outside lab for the test. What modifier is appropriate in this
case?
A. Modifier -99

B. Modifier -90

C. Modifier -52

D. No modifier would be necessary

14. What are the two most common modifiers used in radiological
services?

A. -51 and -26


B. - TC and -26

C. S&I and -26

D. - TC and -51

15. What organizations make up the “Cooperating Parties”


for the ICD-10-CM guidelines and who approved these
guidelines?

A. American Hospital Association (AHA), American Health


Information Management Association (AHIMA), National Center for
Health Statistics (NCHS), U.S. Federal Government’s Department
of Health and Human Services (DHHS)
B. American Hospital Association (AHA), National Center for
Health Statistics (NCHS), World Health Organization (WHO),
U.S. Federal Government’s Department of Health and Human
Services (DHHS)
C. American Hospital Association (AHA), American Health
Information Management Association (AHIMA), Centers for
Medicare & Medicaid Services (CMS), National Center for Health
Statistics (NCHS)
D. American Health Information Management Association
(AHIMA), Centers for Medicare & Medicaid Services (CMS),
National Center for Health Statistics (NCHS), World Health
Organization (WHO)

16. How do you locate a code in the ICD-10-CM?

A. Locate it in the Alphabetic Index then verify it in the Tabular


List and then submit it to insurance.

B. Locate it in the Alphabetic Index and then submit it to


insurance. There is no need to use the Tabular List.
C. Locate it in the Tabular List and submit to insurance. There is
no need to use the Alphabetic Index.
D. There is no need to use either the Alphabetic Index or the
Tabular List to identify the ICD-10-CM code.

17. Can you specify laterality in the ICD-10-CM codes?

A. No, you cannot specify laterality with the ICD-10-CM.

B. Yes, you can specify laterality for some codes. Some


codes do not have “bilateral” listed, so in this case the code for
“right” and “left” would be assigned.
C. Yes, you can specify laterality in the ICD-10-CM codes
when the side is not mentioned in the medical record.
D. No, you cannot specify laterality even if it is mentioned in
the medical record when using the ICD-10- CM codes.

18. Can you use signs, symptoms or unspecified codes in the ICD-
10-CM instead of an actual diagnosis?

A. Yes, symptom/sign codes can be used, but there are no


unspecified codes in the ICD-10-CM
B. No, in ICD-10-CM the purpose of added codes is to prevent
the use of signs, symptoms or unspecified codes
C. Yes, if a definitive diagnosis hasn’t been made by the end
of the patient encounter, then unspecified codes or sign/symptom
codes may be used to best describe the diagnosis at the end of
the encounter
D. No, ICD-10-CM doesn’t even have unspecified codes as an
option
19. What is sequela or “late effects” in the ICD-10-CM coding
guidelines?

A. It is the acute injury or illness right after it has happened

B. It is considered to be the one-month timeframe after an acute


injury or illness

C. Sequela is considered to be the effect after the acute


phase of the illness or injury within three months of the initial
episode
D. Sequela is considered to be the residual effect after the
acute phase of the injury or illness without a time limit imposed on
it
COMPLIANCE & REGULATORY (29
QUESTIONS)

1. A 76-year-old woman visited her dermatologist’s office in


response to a large suspicious nevus on her back. The
dermatologist excised the nevus and sent it to the pathology lab for
examination. The patient experienced pain excision site, due to its
large size and was prescribed pain medication. What two part of
Medicare insurance will the patient need to pay for the
dermatologist’s office visit and the prescription charges?

A. Medicare Parts A and B

B. Medicare Parts B and C

C. Medicare Parts B and D

D. Medicare Parts D and E

2. Tricare Prime patients typically must see physicians:

A. Whenever they feel like it

B. At their military treatment facility


C. Whenever their commander asks them to

D. At their typical private practice doctor’s office

3. Which of the following is always the payer of last resort?

A. Medicare
B. Medicaid

C. Worker’s Compensation Insurance

D. Commercial Insurance

4. Larry works at a cola bottling company and as one of his job


duties, he is required to lift heavy boxes of syrup onto pallets. He
is also required, by the safety department, to wear a back brace
and hernia belt. Larry was lifting a large box of syrup on to a pallet
when he suffered a severe abdominal hernia. Will worker’s
compensation insurance cover his medical costs?

A. Yes

B. No

C. Only if he was wearing the back brace and hernia belt while
he was lifting the box

D. Only if he was clocked in and lifting the box as one of his job
duties
5. A physician performs a rhinoplasty on a 2-year-old male with
nasal breathing problems. The child was born with a congenital
cleft lip and palate with nasal deformity. The cleft and partial
deformity was repaired when the boy was an infant, but the
breathing problems have persisted, and are a direct result of the
congenital nasal deformity. Now that the child can withstand a
more intensive procedure, the surgeon performs the rhinoplasty to
complete the correction of the deformity. What will likely happen
with the processing of this claim, as long as they receive
documentation of the procedure’s medical necessity?

A. Insurance will deny the claim as a cosmetic procedure

B. Insurance will pay for the claim as medically necessary


C. Insurance will initially deny the claim as medically necessary
and then pay for the cosmetic procedure

D. Insurance will not process the claim

6. Sheila took her 5-year-old daughter to the pediatrician’s


office for an annual well-child exam. She has a commercial Blue-
Cross Blue-Shield insurance plan, through her work, which covers
her daughter. Sheila also has Medicaid coverage on her daughter,
due to her low-income status. Which one of her insurances is billed
for the well-child exam?

A. You bill Blue-Cross Blue-Shield first and Medicaid second

B. You bill Medicaid only

C. You bill Blue-Cross Blue-Shield only

D. You bill Medicaid first and Blue-Cross Blue-Shield second

7. Medical billing fraud is:


A. Billing for services at a reduced level

B. Billing correctly for services performed

C. Billing for services that are not medically necessary

D. Billing for services that were not performed

8. What is the purpose of an internal audit?


A. It allows an outside agency to see your records to make sure
that the patients were billed correctly

B. It allows the coders and billers in your office to make sure your
claims were billed correctly

C. It allows Medicare to go through your charges to make sure


that they are reasonable

D. It allows patients to make sure they were not overcharged for


their office visit co pays

9. HIPAA was created to:

A. Protect patient privacy

B. Enact ways to uncover fraud and abuse

C. Create standards of electronic transactions

D. All of the above

E. Only options A and B


10. What is the purpose of a compliance plan?

A. It makes sure you are properly credentialed

B. It allows your office to defend itself in case of an audit

C. It helps your employees claim worker’s compensation

D. It helps your office follow the correct coding and billing


protocols
11. What does HIPAA stand for?

A. Health Insurance Portability and Accountability Act

B. Health Insurance Protection and Accountability Association

C. Health Insurance Post-Payment Auditing Association

D. Health Insurance Accountability and Auditing Act

12. What is the purpose of provider credentialing?

A. To make sure that your provider is performing the correct


procedures

B. To make sure that your provider is correctly licensed to


perform procedures

C. To make sure your provider pays all the necessary fees to


practice medicine

D. To allow your doctor to check the credentials of private


insurance companies
13. In the RBRVS calculation, the GPCI takes into account:

A. The geographic location of a practice or provider

B. The type of provider specialty

C. The malpractice risk of a procedure

D. The overhead cost of the practice


14. What are the correct steps to coding for the best payment
outcome?

A. Identify the reason for the encounter based on what the


physician documented as the diagnosis without reviewing the
medical record. Find the diagnosis in the Tabular List and choose
the code with the highest specificity. Assign the code and submit
to insurance.
B. Identify the reason for the encounter based on the
diagnosed reason and confirmation within the medical record. Find
the diagnosis in the Tabular List, confirm it in the Alphabetic Index,
assign the code without regard to the specificity and submit to
insurance.
C. Identify the reason for the encounter based on the diagnosed
reason and confirmation within the medical record. Find the
diagnosis in the Alphabetic Index, review entries for modifiers,
choose the best code and locate it in the Tabular List, then
determine whether the code is the highest level of specificity. If so,
assign that code to the encounter. Sequencing is very important, so
review this prior to final billing submission.
D. Identify the reason for the encounter based on what the
physician documented as the diagnosis after reviewing the medical
record. Find the diagnosis in the Alphabetic Index, find it in the
Tabular List, and use the first code available without regard to
modifiers or specificity.

15. What are Medically Unlikely Edits (MUEs)?

A. Units of service edits created by CMS to lower the Medicare


Fee-For-Service paid claims error rate

B. Codes that cannot be reported together because they are


mutually exclusive of each other

C. “Add-on” codes that describe a service that can only be


reported with another service listed as the primary code
D. Misuse of column two codes with column one codes

16. What does “place of service” reporting refer to?

A. It refers to the location of the billing department, whether


in a hospital or physician’s office or independent.
B. It refers to the primary location of the provider seeing the
patient. If the provider is an outpatient physician, but sees the
patient inside the hospital, then the place of service is the
outpatient physician’s office.
C. Only services in the hospital or in the providers’ office are
considered “place of service” for reporting purposes.
D. It refers to the location of the setting in which the patient is
treated.

17. What is special about services provided in a patient’s home?

A. When services are provided in the home by a physician or


provider who is not part of an agency, this is considered “non-
facility” services
B. When services are provided by a provider or physician who
is part of an agency, such as home health, then the service is
considered to be provided within a facility
C. Answers A and B are both correct about in home patient care
services

D. None of the above options are correct about patient care


within the home
18. True or false: Only physicians can use place of service
reporting in the CPT guidelines.

A. True, only physicians are considered to be able to provide


services to patients.

B. False, the words “physician,” “qualified healthcare


professional,” or “individual” can all be used and even other
entities may report the service.
C. False, the term “physician” encompasses all healthcare
providers whether registered nurses, physician assistants or nurse
practitioners. These terms leave out physical therapists, speech
therapists, occupational therapists and other entities that may
provide necessary services for the patient.
D. True, only physicians can provide facility and non-facility
services and code for them.

19. What is the Federal Anti-Kickback Law and Regulatory Safe


Harbors?

A. It protects patients and federal healthcare programs from


fraud and abuse by stopping the corrupting influence of money on
healthcare decisions. It states that anyone who knowingly and
willfully receives or pays anything of value to influence the referral
of federal healthcare program business can be held accountable
for a felony.
B. It allows physicians to accept money for referrals to certain
providers or services for patients within a federal healthcare
program.
C. It protects physicians from being charged with a felony if
they willfully receive or pay anything of value to influence their
medical decisions and best plan of care for the patient.
D. It protects patients, part of federal healthcare programs,
from being referred to other providers that aren’t in the best
interest of the patient.
20. The National Correct Coding Initiative (NCCI) promotes
correct coding of healthcare services and prevents payment for
incorrectly coded services. There are tables present for these edits
to the coding. How often are these edits updated at the Centers for
Medicare & Medicaid Services (CMS)?

A. Monthly

B. Quarterly

C. Semi-annually

D. Annually

21. What are mutually exclusive NCCI edits?

A. Code pairs that are unlikely to be performed on the same day


on the same patient

B. Code pairs that are likely to be performed on the same day on


the same patient

C. Code pairs that are unlikely to be performed on the same day


giving a different result

D. Code pairs that are billed together “bundled services”


22. Can you report component codes separately?

A. Yes, you can report component codes separately and may


receive better reimbursement if you do.

B. No, you cannot report component codes separately because


this type of code only reports a portion of the service described.
Look for comprehensive codes that are “bundled” to maximize
reimbursement.
C. Yes, you can report component codes separately. Even
though it is bundled with another service, you can report both the
component and the comprehensive codes.
D. Yes, you should report component codes separately. Many
times there is more than one component of a procedure and by
separating each you can use multiple codes.

23. What do NCD and LCD stand for?

A. National Category for Determinations (NCDs) and Legal


Coverage Determinations (LCDs)

B. National Coverage Determinations (NCDs) and Locally


Covered Detriments (LCDs)

C. National Coverage Determinations (NCDs) and Local


Coverage Determinations (LCDs)

D. National Coding Database (NCDs) and Labs Coverage


Database (LCDs)

24. What do NCD/LCDs specify?


A. They specify under which clinical circumstances a service can
be considered reasonable and necessary

B. They specify what codes are unreasonable or unnecessary


for every specialty and how much will be reimbursed for
something that isn’t reasonable or necessary
C. They specify benefit categories and exclusions for
reimbursement

D. They specify codes that have been decided to be too high


cost or too experimental and that will never be covered by the
insurance
25. What are the parts of the NCD?

A. LCD Information, Coding Information, and General


Information

B. Tracking Information, LCD Information, Coding Information,


and National Coverage Analyses

C. Tracking information, Description Information,


Transmittal Information, and National Coverage Analyses
D. Coding Information, Description Information, Transmittal
Information

26. What is an Advance Beneficiary Notice of Noncoverage (ABN)?

A. An ABN is a written notice that must be given to the


beneficiary that lists “fee-for-service” before items or services are
provided. It states that the services are usually covered but are not
guaranteed, allowing the beneficiary to make an informed decision
about whether to accept financial responsibility if Medicare doesn’t
pay.
B. It is a written form stipulating costs of care and promising that
Medicare will pay for the services
C. An ABN does not have to be given to the beneficiary and, if
not given, the patient/beneficiary is held responsible if Medicare
does not pay.
D. It is a fee-for-service form that does not list specific services
or items that patients have to sign before being seen at any office.

27. What are common reasons for Medicare to deny an item or


service as “a medical necessity”, thus requiring an ABN to be
issued?
A. The service or item is experimental or investigational

B. The service or item is not indicated for the particular


diagnosis or treatment that the patient is being seen or treated for
C. The service or item is not considered safe and effective

D. All of the above

28. What comprises the Total Relative Value Unit (RVUt)

A. Work (RVUw), Practice (PE-RVU), and Malpractice (RVUm)

B. Facility (RVUf), Work (RVUw) and Practice (PE-RVU)

C. Diagnosis (RVUd), Work (RVUw) and Malpractice (RVUm)

D. Work (RVUw), Malpractice (RVUm) and Diagnosis (RVUd)

29. What are the different types of Practice Relative Value Units
(PE-RVUs)?
A. Outpatient and interagency

B. Outpatient and inpatient

C. Facility and non-facility

D. Physician and non-physician


EVALUATION & MANAGEMENT (59
QUESTIONS)

1. A 36-year-old woman with a history of multiple complicated


ectopic pregnancies presented to her OB/GYN’s office. She took
an at-home pregnancy test two weeks ago, which was positive,
and experienced a heavy bleed in the middle of the night last night.
She presented this morning with complaints of excessive vaginal
bleeding and pain in the abdominal area. After confirmation of the
pregnancy via urinalysis, the physician performed a pelvic
examination. After examination, the OB suspected that the
pregnancy was ectopic and ordered an ultrasound confirmation.
The ultrasound confirmation, performed later that day, showed an
advanced interstitial uterine ectopic pregnancy. The estimated age
of the pregnancy was 12 weeks. The OB discussed the risks of the
ectopic pregnancy with the patient, who then decided to have an
excision of the ectopic pregnancy and a total hysterectomy. The
OB spent 45 minutes counseling the patient and the patient was
scheduled for an abdominal hysterectomy in two days.

How should the OB code for the procedure performed in the office?

A. 99214, 76801

B. 99215-57, 81025, 76801

C. 99214-57, 81025
D. 99215, 81025, 76805

2. Mr. Beck was recently seen in the hospital by Dr. Johnson,


who is his PCP. Dr. Johnson usually treats Mr. Beck for his
stomach issues, but he has not had any problems in the last four
years, so he has not scheduled any follow-up appointments. Mr.
Beck was recently weed-eating in his backyard when a rock flew
up and hit him in the eye. Dr. Johnson was the on-call physician,
who treated Mr. Beck for his eye injury. He recommended that Mr.
Beck schedule a follow-up visit the next week. Will Dr. Johnson bill
the follow-up visit as a new patient or as an established patient?
A. New

B. Established

C. Not enough information provided to determine patient status

3. A physician can base the level of an evaluation and


management service on the time spent counseling the patient and
nothing else. Is this statement true or false?

A. True

B. False

4. When selecting an evaluation and management code, what


is the first thing that the coder needs to determine?

A. The time the provider spent with the patient

B. The appropriate category of E&M service


C. Whether the patient was new or established

D. How long the discharge took

5. A new patient was seen in the office complaining of ear


pain, headache and a mild fever. The physician performed a
problem-focused history assessment and an expanded
problem-focused
examination on the patient. The physician diagnosed the patient
with an acute inner ear infection. This medical diagnosis was
considered to be of low complexity. What is the correct E&M code
for the service?

A. 99212

B. 99202

C. 99201

D. 99211

6. The physician on duty examined the patient and determined


that patient was ready for discharge. The physician spent 25
minutes on the hospital discharge of an inpatient discharge. What
is the correct procedure code?

A. 99238

B. 99239

C. 99217

D. 99315
7. HOSPITAL

CHARGE SHEET

PATIENT:

Carson, Cason

AGE: 10
DATE OF ADMISSION: 01/02/2014
DIAGNOSIS:

Coccidiomycosis

Meningitis DATE OF

DISCHARGE: 01/08/2014

The patient was admitted on 01/02/2014 complaining of a severe


headache and hyperpyrexia. An lumbar puncture determined on
01/02/2014 that the patient was suffering from coccidiomycosis
meningitis. The patient was administered antibiotics and admitted
to the hospital for observation and management of the condition.
The patient’s condition improved slowly with a regression on
01/05/2014, at which time the antibiotics were changed. After the
change in medication, the patient’s condition rapidly improved and
by 01/07/2014, the patient no longer developed a fever or
headache when not on pain medications. On 01/08/2012, the
physician spent 20 minutes managing patient discharge, with
instructions to continue antibiotics and pain medications, as
needed.

What E&M service codes would be reported for this service?

A. 99222 (X2), 99232 (X3), 99231 (X3), 99238 (X2)

B. 99232 (X7)
C. 99222, 99232, 99231 (X6)

D. 99222, 99232 (X3), 99231 (X2), 99238

8. In order for a physician to appropriately code for a


consultation service, three things must be documented. What
are those three things?

A. The referral or request from the PCP, the rendering of the


opinion by the specialist or consultant, and the written report or
findings sent from the specialist to the PCP.
B. The rendering of the specialty service to the patient, the
referral of the patient from the specialist to an additional specialist,
and the written report of the findings provided to the specialist.
C. The specialist request of a second opinion regarding the
patient, the PCP’s advice regarding which second specialist the
patient should see, and the second specialist’s report or findings.
D. The referral from the PCP to the specialist, an additional
referral from the specialist to another specialist, and the
written report or findings sent from the specialist to the PCP.

9. The patient was seen in the emergency department for a


severe laceration to the right forearm, following a work injury. The
on-call physician performed an expanded problem-focused history
assessment and examination, and then sutured the complicated
wound using 25-0 vicryl sutures in three subcutaneous layers. The
patient was prescribed prophylactic antibiotics and released from
the emergency department. The patient was instructed to return to
his PCP in one week for a follow-up appointment. The MDM was
moderate. What is the correct level of E&M service?

A. 99284

B. 99282
C. 99283

D. 99291

10. The physician saw the disabled patient during a home visit.
The patient recently moved to be closer to her family therefore she
is considered a new patient. The physician performs a detailed
history and an expanded problem-focused examination. Due to the
lack of risk and small amount of data to be analyzed, the MDM is
of low complexity. What is the correct evaluation and management
service code?
A. 99342

B. 99343

C. 99348

D. 99349

11. A pediatric patient with a history of asthma and pneumonia


presented to the office with severe respiratory distress. The
pediatrician performed a detailed history and comprehensive
examination, and diagnosed the patient with status asthmaticus. A
pulse oxygen level was taken and it was determined that the
patient’s blood oxygen level was at 88%. The patient was started
on a nebulizer treatment at 0950 hours, which lasted until 1015
hours. The physician then re-checked the patient and determined
that the patient’s breathing had only slightly improved. A pulse
oxygen level was taken again and it was determined that the
patient was at 92%. The physician then ordered another nebulizer
treatment, which was started at 1032 and continued until 1054.
After this second breathing treatment, an additional pulse oxygen
level was taken and the patient’s blood oxygen level had risen to
97%. The pediatrician then determined that the patient needed to
be sent for chest x-rays to determine whether or not pneumonia
was present in the lungs. Due to the resulting amount of data and
risk, the pediatrician considered the MDM of high complexity. The
total time spent with the patient was 1 hour 45 minutes.

What E&M codes would you use to code for the office visit?

A. 99214, 99354, 99355


(X2)

B. 99215, 99354, 99355

C. 99215, 99355 (X3)

D. 99215, 99354, 99355


(X2)
12. If a physician performed a female pelvic examination under
general anesthesia, you should code for the procedure using an
appropriate code from the female genital system subsection of the
surgery chapter of the CPT manual. If the physician performed a
pelvic examination without general anesthesia, from what chapter
or subsection of the CPT manual should you code?

A. Anesthesia

B. Maternity Care and Delivery

C. Evaluation and Management

D. Female Genital

13. A 32-year-old married female presents to her OB/GYN office


for diaphragm fitting. After performing a pelvic examination and
routine physical, the OB measures the cervix and adjusts the
diaphragm so that it fits neatly over the cervical opening. The OB
then instructs the patient in how to place the diaphragm for most
effective birth control, as well as how to remove, clean, and store
the diaphragm. Satisfied that the patient understands how to use
the device properly, she allows the patient to leave the office with a
follow-up appointment scheduled in one month. How should the
OB/GYN code for this visit?

A. 99395

B. 99395, 57170-59

C. 57170

D. 99395, 57170
14. The physician performed an annual examination on a 47-
year-old male new patient with a history of congenital heart
disease. What is the correct E&M code?

A. 99387

B. 99386

C. 99396

D. 99397

15. The nurse practitioner returned a phone call from a


concerned daughter regarding her ailing mother’s health. She
spent 20 minutes counseling the daughter on how to provide
hospice care during the last stages of her mother’s life. It had
been three weeks since the patient had been seen in the office,
and her next scheduled appointment was in two weeks. What
E&M service can be reported for this phone call?

A. You cannot report an E&M

service for this phone call B. 99442


C. 98967

D. 99443

16. A patient is admitted to the hospital for observation for


possible dehydration. A detailed history is performed with a
detailed examination of the patient on the unit. The medical
decision making was low
complexity and straightforward, so the physician was only on the
unit for about 20 minutes. What is the best code to be used for this
observation patient?

A. 99218

B. 99219

C. 99220

D. 99224

17. A patient is admitted to the telemetry unit for observation for


chest pain, directly from his physician’s office. The hospitalist
spent a little more than 50 minutes on the unit after the patient
arrived, performing a comprehensive examination and obtaining a
comprehensive history on the patient. The hospitalist performed
moderate complexity medical decision making when ordering labs,
telemetry, stress testing and echocardiogram. What is the
appropriate code for initial observation care of this patient?

A. 99218

B. 99219
C. 99220

D. 99224

18. A patient admitted in the afternoon for 24-hour observation,


after complaining about chest pain in his PCP’s office, has
worsened during the evening. He now has increasing chest pain,
requiring medication for relief; his EKG has changed from his
PCP’s office; and he is being prepped for a cardiac
catheterization because his labs are returning indicating an acute
MI. What is the appropriate code for a patient who is unstable after
initial observation?

A. 99220

B. 99224

C. 99225

D. 99226

19. A patient admitted to the hospital three days ago is getting


ready to be discharged this morning. The attending physician spent
more than 30 minutes arranging home health services and talking
with the patient and family regarding outpatient rehabilitation and
care after right hip replacement. What code should be used for the
discharge day management of this patient?

A. 99235

B. 99236

C. 99238

D. 99239
20. A 54-year-old woman is being set up for consultation with
general surgery for follow up after a large lump was identified on
mammography by the woman’s PCP. This office consultation will
include a comprehensive history, a comprehensive examination,
and moderate-complexity medical decision making because it will
involve discussing options, additional tests, and surgery. The
surgeon allowed for a 60-minute visit with this woman and her
family. What is the appropriate code for consultation?
A. 99242

B. 99243

C. 99244

D. 99245

21. A speech therapist is asked to consult on a stroke patient in


the hospital. She spends 45 minutes with the patient assessing and
performing an expanded problem-focused history and examination,
but that only required straightforward medical decision making.
What consultation code should be used for this patient?

A. 99251

B. 99252

C. 99253

D. 99254
22. An emergency room physician has to talk emergency
medical systems (EMS) personnel through advanced life
support, including telemetry of cardiac rhythm, endotracheal
intubation and electrical conversion of arrhythmia. This is done
by two-way voice communication with EMS outside of the
hospital. What code should be used for this service?

A. 99281

B. 99284
C. 99285

D. 99288

23. A 10-year-old boy lacerated his chin while chasing his baby
brother. He requires stitches. What would the evaluation and
management CPT code be for this patient, who is being seen in the
emergency department?

A. 99281

B. 99282

C. 99283

D. 99284

24. What CPT code should be used for a patient that requires 68
minutes of evaluation and management while in critical care?

A. 99285
B. 99288

C. 99291

D. 99292
25. In the evaluation and management of a critical care patient,
timing is most important. What code should be used for a patient
that requires two hours of evaluation and management?

A. 99291

B. 99292

C. 99291, 99292

D. 99291, 99292 x 2

26. A patient in critical care requires 190 minutes of


evaluation and management time. What CPT code(s) should
be used to indicate this evaluation and management of critical
care patients?

A. 99291

B. 99291, 99292

C. 99291, 99292 x 2

D. 99291, 99292 x 4
27. An 82-year-old woman is admitted to a nursing facility for
rehabilitation after having had a stroke. The admitting physician
spent 35 minutes at the nursing home completing a comprehensive
history and examination and moderate complexity medical decision
making. What evaluation and management CPT code should be
used for this patient?

A. 99304
B. 99305

C. 99306

D. 99307

28. A 76-year-old man has completed his rehabilitation at a


nursing facility after bilateral knee replacements. He is being
discharged from the nursing facility and the discharging physician
spends 35 minutes completing this process. What is the correct
evaluation and management code for this patient?

A. 99304

B. 99307

C. 99315

D. 99316

29. An 86-year-old woman at a nursing facility develops


confusion and becomes combative. It is discovered that she has a
urinary tract infection, which is causing her confusion. She has
been in the nursing facility for five days already and the attending
physician is called in to assess the patient and manage her care.
He comes and does a detailed interval history, a detailed
examination and moderate- complexity medical decision making,
spending about 25 minutes with the patient and writing orders.
What is the correct evaluation and management code for this
patient?

A. 99307

B. 99308

C. 99309
D. 99310

30. A down’s syndrome 40 year old is being admitted to a


domiciliary home. She does not have any known medical
conditions, but is being removed from her home due to inadequate
care with social services being the custodians of her care at this
time. The physician spends 20 minutes with the patient and facility
staff and the medical decision making is straightforward as he
completes a problem-focused history and examination. What
would the correct evaluation and management code be for this
new patient?

A. 99324

B. 99325

C. 99326

D. 99327

31. A patient in an assisted living facility is under the care of a


physician for atrial fibrillation requiring Coumadin, which requires
frequent laboratory checks. She is also a brittle diabetic requiring
frequent blood sugar checks and insulin adjustments, which
routinely involves telephone calls to the physician from the facility.
Because of COPD, she is on continuous oxygenation and routinely
needs medication adjustments and breathing machine adjustments
to maintain her oxygen levels. The physician is typically called 3-4
times a week, despite routine orders onsite for the nurses, and he
then has to spend time talking with his patient’s daughter about the
changes to her mother’s care. He spends more than 30 minutes a
month doing all of this. What is the correct evaluation and
management code for this type of encounter?

A. 99324
B. 99334

C. 99339

D. 99340

32. An established patient at an assisted living facility develops a


cough that is diagnosed as pneumonia after radiology was
performed. The physician was called in to assess her and ordered
the x-ray as an outpatient. Because of her history of congestive
heart failure, this acute diagnosis is more complex. The physician
does a detailed interval history, a detailed examination and
considers his medical decision making to be of moderate
complexity because he has spent 40 minutes with the patient, at
the assisted living facility and talking with the patient’s family when
they come for a visit. What is the correct evaluation and
management CPT code for this patient?

A. 99334

B. 99335

C. 99336

D. 99337
33. A rural doctor is called to the home of a friend of one of his
elderly patients, who is a new patient to him. She was having
difficulty breathing and has had a cough for more than three
weeks. He performed a detailed history and a detailed examination
and spent more than 45 minutes with the patient, causing his
encounter to be considered moderate complexity. What is the
correct CPT evaluation and management code for a new patient
receiving a home visit?

A. 99341
B. 99342

C. 99343

D. 99344

34. A physician visits one of his homebound patients, who has


been complaining about right hip pain after she fell three days ago.
She is now unable to bear weight and pain is worsening. He
performs a comprehensive examination and his medical decision
making is considered moderate complexity because he is
organizing transportation for the patient to the hospital for
suspected hip fracture and has spent 40 minutes with the patient
and the patient’s son discussing her care. What is the correct
evaluation and management code for this patient?

A. 99347

B. 99348

C. 99349

D. 99350
35. When is a prolonged service code able to be used?

A. When prolonged service involving direct patient care contact


is provided beyond the usual service in the outpatient setting or
inpatient setting
B. When face-to-face or non-face-to-face services are
performed on the patient’s unit during the same session
C. Reported as an “add-on” code in addition to the initial
evaluation and management services code for that patient
D. All of the above are correct regarding a prolonged service
code

36. A cancer patient who has had a relapse after moving states
is an inpatient at the cancer floor of the local hospital. Her current
oncologist has obtained her previous medical records from
oncology, pulmonology, general surgery, transplant team, and
cardiology and is spending a significant amount of time reviewing
her previous care and how the cancer was treated by her previous
doctors. He has spent an additional hour going over the charts
from her previous providers and has spent time on the phone with
her previous oncologist. This prolonged service did not involve
face-to-face time with the patient but was necessary for review of
her medical history and previous treatments and responses. What
evaluation and management code should be used for this
encounter?

A. 99356

B. 99357

C. 99358
D. 99359

37. A 54-year-old patient on Warfarin is being routinely followed


by having international normalized ratio (INR) testing done, with the
physician reviewing and adjusting medication dosing as needed.
This is done through the physician’s office on an outpatient basis.
The patient is in her first 90 days of therapy and has had nine INR
measurements done at this time. What is the correct evaluation
and management code for this encounter?
A. 99360

B. 99363

C. 99364

D. 99366

38. A patient who has a complicated medical history and


presentation of cancer relapse has met with cardiology,
pulmonology, oncology and general surgery. The main physician
from each of these specialties is going to have a medical team
conference with the patient and her family regarding her medical
plan of care. This conference is estimated to take two hours to
discuss all aspects of their recommendations. What is the correct
evaluation and management CPT code for this type of medical
conference?

A. 99363

B. 99366

C. 99367

D. 99368
39. To be considered a medical team conference, what is
the minimum number of healthcare professions from different
specialties or disciplines that must participate?

A. One

B. Two

C. Three
D. Four

40. A physician provides supervision of a 49-year-old patient, in a


domiciliary environment, requiring complex and interdisciplinary
care changes and revisions to care plans, due to having been
diagnosed with cancer and having uncontrolled hypothyroidism
and diabetes. This supervising physician spends more than 35
minutes per month talking with the legal guardian, the in-home
nurse, and reviewing laboratory results and making changes to her
medications. What evaluation and management CPT code should
be used for this patient’s care plan oversight?

A. 99374

B. 99375

C. 99377

D. 99378

41. A hospice patient’s supervising physician spends 28


minutes a month reviewing labs, making care plan changes, and
talking with hospice staff and family members of the patient.
What is the correct evaluation and management CPT code for
this patient?

A. 99374

B. 99375

C. 99377

D. 99378
42. An 87-year-old patient in a nursing home requires more
than 60 minutes of supervised care per month. What is the
correct evaluation and management CPT code for this patient’s
care?

A. 99374

B. 99377

C. 99379

D. 99380

43. A young college student uses email to communicate with her


family physician because she is living away from home. She has
developed acne while she has been away from home and various
over-the- counter medications are no longer helping. She sends
her physician an email requesting a prescription because she won’t
be able to come back to see him until the next school holiday,
which is in 2 ½ months. The physician copies her email into her
electronic medical record and sends a prescription for medication
to the pharmacy nearest to her. He also copies his response to her
through email into her electronic medical record as well,
documenting his care. What is the correct evaluation and
management code for this type of an encounter?

A. 99441

B. 99442

C. 99444

D. 99446
44. A patient’s mother calls, asking to speak with the physician,
because her three year old is having behavioral problems at school
and at home, including biting and sudden bedwetting. It was
recommended that she set up an appointment, but the mother
reports that, due to work schedule, she won’t be able to get her son
in until next week, but will set up an appointment to follow up in the
office. The physician spends 15 minutes on the phone with her
discussing the behavior, and that this may be related to life
changes because there have been some changes going on with
the patient’s dad no longer living in the home. He recommends
following up tomorrow due to the increased bedwetting, but due to
the mother’s schedule, she is unable to do this. What is the correct
evaluation and management code for this patient encounter?

A. 99441

B. 99442

C. 99443

D. 99444

45. A physician has a mother who is concerned about her


daughter having cancer. He does preliminary labs to help alleviate
the mother’s fears, but some of the labs are coming back
abnormal. He contacts the pediatric oncologist located more than
an hour away by phone, requesting a telephone consultation
regarding the abnormal labs and patient’s presentations. He
spends 15 minutes on the phone consulting and obtaining
recommendation for follow up labs and plan of care within his
office. The oncologist does not believe this is related to cancer and
has recommended additional work up. However, if subsequent labs
are abnormal, he recommends follow up in his office. The
subsequent labs are all within normal range and the primary
physician repeats the labs again within his office in one month.
What is the correct evaluation and management code for this
consultation encounter?

A. 99446

B. 99447
C. 99448

D. 99449

46. A 45 year old needs a basic life exam that includes height,
weight, blood pressure, medical history, urinalysis and/or blood
samples, and completion of necessary
documentations/certificates. What is the correct evaluation and
management code for this type of encounter?

A. 99449

B. 99450

C. 99455

D. 99456

47. A patient needs a work-related examination and it will be


performed by the treating physician. What is the correct evaluation
and management code for this type of encounter?
A. 99444

B. 99450

C. 99455

D. 99456
48. A patient needs a medical disability examination performed,
but it will be performed by a different physician other than his
treating physician. What is the correct evaluation and management
CPT code for this encounter?

A. 99449

B. 99450

C. 99455

D. 99456

49. What is the CPT code for the initial evaluation and
management of a normal newborn infant in a birthing center?

A. 99460

B. 99461

C. 99462

D. 99463
50. A home birth newborn is seen for the first time in her
pediatrician’s office on the same day as her delivery at home.
What is the correct CPT code for this initial encounter of the
newborn?

A. 99460

B. 99461
C. 99462

D. 99463

51. What is the correct evaluation and management code for a


normal newborn infant who was born at 1 am and then discharged
by 6 pm that same day?

A. 99460

B. 99461

C. 99462

D. 99463

52. What is the correct evaluation and management code for a


2-day old infant who began breathing rapidly and had elevated
heart rate, without any known cause? He is admitted to the
neonatal critical care after interfacility transport is provided. What
is the correct code for the interfacility transport that lasted 35
minutes?
A. 99466

B. 99467

C. 99485

D. 99486
53. An 18 month old is rushed to critical care after being critically
injured during a motor vehicle accident. What is the correct
evaluation and management code for initial inpatient pediatric
critical care evaluation?

A. 99468

B. 99469

C. 99471

D. 99472

54. A three year old who was critically injured during a firecracker
accident has been in intensive care for two days now. What is the
correct evaluation and management CPT code for this patient?

A. 99468

B. 99471

C. 99475

D. 99476
55. A patient who requires complex chronic care management
services must meet certain criteria. What degree of medical
decision making and how many minutes of care management must
be provided for these patients?

A. Straightforward medical decision making, at least 60 minutes


of clinical care management.
B. Low-complexity medical decision making, at least 45 minutes
of clinical care management.

C. Moderate- or high-complexity medical decision


making, at least 45 minutes of clinical care management.
D. Moderate- or high-complexity medical decision
making, at least 60 minutes of clinical care management.

56. A leukemia patient who is on 10 medications suffered a heart


attack. He is receiving physical therapy and occupational therapy
as well as chemotherapy and radiation therapy. He meets the
requirements for complex chronic care management services and
his attending physician has spent one hour and 45 minutes in the
direct care of this patient’s management. What is the correct
evaluation and management code for this complex patient?

A. 99487

B. 99489

C. 99487, 99489

D. 99487, 99489 x 2
57. How many chronic continuous or episodic health
conditions must a patient have to qualify for complex chronic
care management services?

A. One

B. Two

C. Three
D. Four

58. What is the timeframe for transitional care management


services?

A. Begins when the patient enters the hospital and ends 30 days
after this date

B. Begins when the patient enters the hospital and ends 29 days
after their discharge from the hospital

C. Begins on the day of discharge and ends 60 days after this


date

D. Begins on the day of discharge and ends 29 days after this


date

59. A congestive heart failure patient is discharged from the


hospital and has a follow up appointment with his primary care
physician five days after discharge. The primary care physician’s
office calls him within 36 hours after discharge to follow up by
phone to see how he is doing. Because he has had weight gain
since leaving the hospital, the primary care office requests to see
him that same day to see if this gain is from fluid retention and
related to his congestive heart failure. He is seen in their office and
the medical decision making is considered moderate complexity.
What is the correct evaluation and management code for this
patient?

A. 99495

B. 99496

C. 99497

D. 99498
HCPCS LEVEL II (14 QUESTIONS)

1. HCPCS Level II ambulance service modifiers represent:

A. The type of ambulance transport vehicle

B. The type of emergency service

C. Whether or not the patient received life support during


transport

D. Where the patient was picked up and where the patient was
dropped off

2. When listing both CPT and HCPCS modifiers on a claim, you:

A. List the HCPCS modifier first

B. Do not list the HCPCS modifier at all

C. Only list the CPT modifier

D. List the CPT modifier first


3. When you append the HCPCS modifier -TC to a service, what
are you indicating?

A. That the provider performed the professional component of the


service

B. That the provider performed the technical component of the


service

C. That the provider performed the entire service

D. That the service was performed by an anesthesiologist


4. Fill in the
blanks: HCPCS modifiers can be used for but CPT modifiers
cannot be used for .

A. CPT codes, CPT codes

B. CPT codes, HCPS codes

C. HCPCS Codes, procedure codes

D. HCPCS Codes, CPT Codes

5. The HCPCS Level II modifier -E1 stands for:

A. Lower Right, Eyelid

B. Upper Right, Eyelid

C. Upper Left, Eyelid

D. Right Hand, Thumb


6. The HCPCS manual includes codes for:

A. Procedures that are also found in the CPT coding manual

B. Supplies, services, and procedures that are not found in the


CPT manual
C. Only supplies that you cannot find in the CPT manual

D. All services performed in the office, except for procedures

7. What types of codes are located in the V0000 through V5999


section of the HCPCS manual?

A. Pathology and Laboratory Services

B. Vision and Hearing Services

C. Dental Procedures

D. Transportation Services

8. Which of the following is the best, most effective way to


locate the correct code for a service or supply in the HCPCS
manual?

A. Look in the Index for the name of the service or supply, and
it will direct you to the correct code or range of codes
B. Go directly to Appendix 1 to check the name of the
supply, and then find out the route of administration
C. Flip through the sections of the book until you find the correct
service or supply and then assign the best code
D. Go directly to the code in the book, and assign the code that
looks right
9. HCPCS J-Codes are used to represent:

A. Drugs administered by methods other than the oral method

B. Durable medical equipment

C. Dental procedures not found in the CPT manual

D. Temporary national codes for Medicare

10. When using the table of drugs in the HCPCS Level II


manual, you must know the drug’s administration route. The
abbreviation “VAR” stands for which of the following?

A. Various Routes

B. Variable Routes

C. A Variety of Routes

D. None of the Above


11. HCPCS Level II codes are updated every quarter by:

A. CMS

B. Medicaid

C. Tricare

D. Commercial payers
12. Appendix 1 in the HCPCS Level II manual contains:

A. An alphabetized list of HCPCS modifiers

B. A table of drugs

C. A list of changes, additions, and deletions

D. A short list of CPT codes to use with HCPCS codes

13. What is the purpose of temporary national codes in the HCPCS


Level II manual?

A. They are for procedures that are considered temporary

B. There are no temporary codes, only permanent codes

C. They allow the establishment of codes prior to the January 1st


annual update

D. They allow the deletion of codes prior to the January 1st


annual update
14. A preoperative patient needs to have an antibiotic started
before his operation. He is given 500 mg Ceftriaxone sodium
through his IV. What is the correct HCPCS code for this
medication?

A.

J071

5 B.

J069

6
C.

J0696

x2 D.

J0697
ICD-10-CM VOLUMES 1 & 2 (32
QUESTIONS)

1. When is the placeholder “X” used in the ICD-10-CM?

A. It is used to stipulate that no further expansion is needed

B. It is used to stipulate that further expansion is needed for


certain codes

C. It is used to define a particular code and indicates that no


further expansion is needed

D. It is used to indicate a subdivision of a code with eight


characters

2. When can “other” codes be used in the ICD-10-CM?

A. “Other” and “other specified” are used when the medical


record provides enough details that a specific code can be
identified
B. “Other” and “other specified” codes are used when the
medical record provides enough details but a specific code is not
able to be identified because it does not exist
C. “Other and “other specified” codes are used when it is too
difficult to identify another code that exists
D. “Other and “other specified” codes can be used for any
diagnosis and any patient about which the provider and coder
differ on the codes they would choose and decide to use this
delineation instead.

3. What is a combination code and when can it be used?


A. A combination code is a single code that is used to classify
two diagnoses. It is used when the coder has reviewed the
inclusion and exclusion criteria and when the code fully identifies
the conditions listed in the medical record.
B. A combination code is a single code that is used to identify
three diagnoses. It is used when the coder has looked at the
exclusion criteria and when it fully all of the elements listed in the
medical record
C. A combination code consists of two codes used to classify
two diagnoses. It can be used to identify all of the elements
involved in the diagnoses, as long as there are detailed records of
these elements.
D. A combination code consists of two codes used to classify
at least two diagnoses. It can be used to identify all of the
elements of the detailed medical record.

4. Do “borderline” codes exist in the ICD-10-CM?

A. Yes, “borderline” codes exist for all diagnosis in the ICD-10-


CM

B. No, “borderline” codes do not exist for any diagnosis in the


ICD-10-CM
C. Yes, “borderline” codes exist, but only for some diagnoses in
the ICD-10-CM when the documentation clearly states a
“borderline” diagnosis
D. Yes, “borderline” codes do exist in the ICD-10-CM and are
only used when a condition isn’t clarified in the providers’
documentation at the coder’s discretion.

5. What is the correct sequencing for severe sepsis in the ICD-


10-CM?

A. Severe sepsis can be coded as the primary code and then


the systemic infection is coded because it is the underlying
infection.
B. Severe sepsis is coded as the secondary code and never as
the primary code.

C. Severe sepsis is coded as the primary code and does not


require additional codes.

D. Severe sepsis is coded as the primary code because there


are subcategories of it. The secondary code is the subcategory
designated only after it has been confirmed by further testing.

6. When can pain codes be used when using ICD-10-CM?

A. Pain codes can be used independently or in conjunction with


other codes

B. Pain codes can be used as primary or secondary codes.

C. If the site of pain is identified in the medical record it should


be indicated with the appropriate pain code too.
D. All of the above

E. None of the above


7. How do you report bilateral glaucoma where each eye is
documented as having a different type or stage?

A. Assign the bilateral glaucoma code for whatever type of


glaucoma is the more severe.

B. Assign individual codes when the glaucoma classification


distinguishes laterality. Use each code individually for each eye to
assign the type of glaucoma, and then use the appropriate seventh
character for the stage.
C. Assign individual codes for each eye, each type, and each
stage.
D. Assign individual codes when glaucoma classification
distinguishes laterality, but do not distinguish between the types
or the stages.

8. Is there a national requirement for external causes of morbidity


codes to be used?

A. Yes, it is a national requirement for mandatory ICD-10-CM


external cause reporting.

B. Yes, there is a national and state requirement for reporting


ICD-10-CM external causes of morbidity.

C. No, there is no national requirement for mandatory ICD-10-


CM external cause reporting. However, there may be a state-
specific requirement for reporting or individual payer
requirements may request this information.
D. No, there is no national requirement and there are no
state or individual payer requirements for mandatory ICD-10-
CM external cause reporting.

9. What are “Z” codes in the ICD-10-CM?


A. Z codes should always be listed as the primary code and
never as a secondary code.

B. Z codes are used to identify factors influencing health status


and contact with health services and may be used as either
primary or secondary codes. They are not procedure codes, so
they must always have a corresponding procedure code listed to
describe any procedure performed.
C. Z codes are used to identify factors influencing health status
and contact with health services and may only be used as
secondary codes. They may also be used as procedure codes, so
they may be used independently.
D. Z codes should always be listed as the primary code and are
used to identify factors influencing health status and contact with
health services.
10. How are coexisting conditions that the patient has but are not
the primary reason for the encounter coded? Which answer is
incorrect regarding coexisting conditions?

A. All conditions in the patient’s medical history should be coded

B. All conditions that coexist at the time of the encounter


should be coded, if they affect or require a plan of care in the
patient’s treatment plan
C. Conditions that previously existed and were treated that no
longer exist should not be coded

D. History codes may be used as secondary codes for


conditions that previously existed but that have been treated and
no longer exist

11. A patient with HIV is admitted for enterocolitis due to


Clostridium difficile. What is the correct ICD- 10-CM code(s) and
what is the correct sequence?
A. B20,

A04.7 B.

B20,

A09 C.

A04.7,

B20 D.

A04.7,

R75

12. A patient with acute lymphoblastic leukemia is currently


undergoing chemotherapy and radiation therapy and develops
anemia as a result. He requires admission and a blood transfusion
because of this
complication caused by the chemotherapy. What ICD-10-CM
codes should be used and what is the correct sequence?

A. C91.10,

D64.81,

T45.1X5A B.

C91.00,

T45.1X5A,

D64.81 C.

D64.81, C91.00,

T45.1X5A D.

T45.1X5A,

D64.81, C91.00

13. A 35-year-old male with type 1 diabetes, who has an insulin


pump, also has diabetic retinopathy without macular edema. He
is admitted to the hospital with ketoacidosis without coma. What
are the correct ICD-10-CM codes and what is the correct
sequence?
A. E10.10,

E10.319, Z79.4

B. E10.11,

E10.319

C. E10.10,

E10.311, Z79.4

D. E10.11,

E10.311

14. A 33-year-old woman follows up with her psychiatrist for


ongoing major depressive disorder that is considered moderate.
She is on a daily medication for this disorder. She also follows up
with him for generalized anxiety disorder. She smokes 1.5 packs of
cigarettes per day and she reports that this helps her anxiety and,
when she doesn’t smoke, her anxiety increases. What ICD-10-CM
codes are correct and what sequence should they be listed?
A. F33.0,

F41.9, F17.200

B. F33.1,

F41.1, F17.210

C. F33.0,

F41.1, F17.200

D. F33.1,

F41.9, F17.210

15. A 21-year-old patient with chronic tension-type headaches


that are not intractable relies on daily prophylactic medications to
help relieve the headaches. What is the correct ICD-10-CM code
for this type of headache?

A.

G43.10

B.

G44.01

9 C.
G44.20

1 D.

G44.22

16. A 45-year-old patient has a chalazion of her right upper


eyelid. What is the correct ICD-10-CM code indicating laterality for
this patient?

A.

H00.1

1 B.

H00.1

2 C.

H00.1

3 D.

H00.1

4
17. A 2-year-old patient is here for initial examination and is
found to have a right acute serous otitis media and impacted
cerumen of the left ear. What are the correct ICD-10-CM codes for
this patient in the correct sequence?

A. H65.02,

H65.21 B.

H61.22,

H65.01 C.

H65.01,

H61.22 D.

H65.03,

H61.21

18. A patient with a history of asthma is seen in the office for an


acute nasopharyngitis (common cold) and increasing difficulty
breathing, even using her albuterol inhaler at home 2-3 times a
day. She is diagnosed with a common cold and mild intermittent
asthma with acute exacerbation. What is the correct sequence of
ICD-10-CM codes for this patient?
A.

J45.21,

J22 B.

J45.20,

J00 C.

J45.31,

J00 D.

J45.21,

J00

19. A 26-year-old man has been diagnosed with Crohn’s disease


of the large and small intestine without complications. What is the
correct ICD-10-CM code for this diagnosis?
A.

K50.

0 B.

K50.

1 C.

K50.

8 D.

K50.

20. An elementary school student who wrestles has been


diagnosed with non-bullous impetigo. What is the correct ICD-10-
CM code for this type of infectious skin disease?

A.

L01.0

0 B.

L01.0

1 C.
L01.0

2 D.

L01.0

21. A 2-month-old infant is diagnosed with torticollis due to not


having enough tummy time and always preferring to lay on one
side of her head. What is the correct ICD-10-CM code for this
patient?

A.

M43.

2 B.

M43.

5 C.

M43.

6 D.

M43.

8
22. A 28-year-old woman will be undergoing a lithotripsy for
bilateral kidney stones. The stones are secure within the kidney
and have not begun to migrate. What is the correct ICD-10-CM
code for this patient’s kidney stones?

A.

N20.

0 B.

N20.

1 C.

N20.

2 D.

N21.

23. A 12-week-pregnant woman develops gestational diabetes,


which she is able to control through diet and hasn’t yet had to use
insulin to control. What is the correct ICD-10-CM code for this
patient?
A.

O24.4

10 B.

O24.4

14 C.

O24.4

19 D.

O24.4

24. A little girl was born to a mother who used cocaine up to the
day before delivery. The drug screen is pending, but the patient is
suspected to be affected by the maternal use of cocaine. The
patient is small for gestational age, at 2450 grams. What are the
correct ICD-10-CM codes for this patient?

A. P04.41, P05.18
B. P04.49,

P05.17 C.

P04.41,

P05.17 D.

P04.49,

P05.18

25. A patient was born with a ventricular septal defect. What is the
correct code for this diagnosis?

A.

Q20.

2 B.

Q21.

0 C.

Q21.

1 D.
Q22.

26. A patient was diagnosed with pneumonia based on chest x-


ray findings and has significant shortness of breath. An unknown
organism is causing the pneumonia. What is the correct coding for
ICD-10-CM?

A. J13,

R06.02 B.

J13,

R06.00 C.

J18.9,

R06.00 D.

J18.9,

R06.02

27. What codes are correct for impaired fasting glucose and for
elevated blood alcohol level?
A. R73.0,

R78.0 B.

R73.01,

R78.0 C.

R73.02,

R78.1 D.

R73.0,

R78.1

28. A patient fell off of a horse and broke her right wrist. She had
it x-rayed and it was noted that it was a non-displaced fracture of
the scaphoid bone of the right wrist and it was casted in the
emergency department. What ICD-10-CM codes should be used
for this incident?

A. S62.04A,

V80.010D, Y93.52

B. S62.04D,

V80.010S, Y93.52
C. S62.04S,

V80.010A
D. S62.04A, V80.010A, Y93.52

29. A 33-year-old woman is seen by obstetrics for a normal


second pregnancy and she is at 16-weeks’ gestation. What are
the correct ICD-10-CM codes for this patient?

A. Z34.02,

Z3A.16 B.

Z34.82,

Z3A.16 C.

Z34.01,

Z3A.15 D.

Z34.82,

Z3A.15
30. A five year old accidentally slammed his right 4th digit in a car
door. He is diagnosed with a contusion of the right 4th finger with
damage to the nail. What is the correct code for this initial
encounter?

A.

S60.11

1A B.

S60.12

1D C.

S60.14

1A D.

S60.14

1D

31. A patient receives a prophylactic antibiotic that is started


preoperatively through his IV. This antibiotic (ceftriaxone sodium,
250 mg) is infused over 30 minutes. What is the correct CPT code
for this procedure?

A. 96365, J0696

B. 96366, J0696

C. 96367, J0696

D. 96368, J0696

32. A patient receives normal saline as an intravenous infusion


that is given over 45 minutes within the physician office. What is
the correct CPT code for this procedure?
A. 96360

B. 96361

C. 96365

D. 96366
LABORATORY & PATHOLOGY (54
QUESTIONS)

1. A pediatric patient with hyperpyrexia, headache and nausea


presented to the office for a sick visit. Upon examination, the
pediatrician determined that the patient needed a Complete Blood
Count (CBC) to check for the presence and the amount of bacteria
in the blood. The first CBC came back with a highly elevated
number of white blood cells indicating infection. The pediatrician
then ordered a Dexamethasone antibiotic injection in the office and
waited three hours and then repeated CBC to recheck the patient’s
white blood cell count. The repeat CBC showed a decrease in the
number of white blood cells indicating that the antibiotic injection
was effective. The pediatrician then released the patient. What is
the correct code for the CBC only?

A. 85025, 85025 -91

B. 85025 -22

C. 85025

D. 85025 (X2)
2. The patient presented to the physician’s office for a
comprehensive metabolic panel, but the physician was unable to
obtain a sample of the patient’s total calcium. What is the correct
code for this laboratory service?

A. 82040, 82247, 82310, 82374, 82435, 82565, 82947, 84075,


84132, 84155, 84295, 84460, 84450,
84520

B. 80053

C. 82040, 82247, 82374, 82435, 82565, 82947, 84075, 84132,


84155, 84295, 84460, 84450, 84520

D. 82374, 82435, 84132, 84295


3.
A patient was brought to the Emergency
Department following a car accident. Suspecting that the patient
had been drinking and driving, the physician ordered a
alcohol pathology test. After determining that the patient did indeed
have alcohol in their system, the physician then ordered a
pathology test to check the amount of alcohol in the
bloodstream.

A. Qualitative; quantitative

B. Quantitative; qualitative

C. Qualitative; cytopathological

D. Pathological; quantitative

4. An adolescent comes to the lab for drug testing. The lab


tests the patient for three drug classes: alcohols, cocaine and
metabolites and tetrahydrocannabinols. The laboratory uses the
chromatographic method to test all three drug classes with one
screening. The patient comes up positive for
tetrahydrocannabinols, but negative for the remaining drug
classes. A confirmation test is performed to confirm the positive
test results. What is the correct code for this laboratory service?

A. 80101 (X3), 80102

B. 80100

C. 80101, 80102

D. 80100, 80102
5. A 28-year-old woman presented to the OB/GYN stating that
she tested positive on a home pregnancy test. The OB decides to
perform a urine pregnancy test in the office for confirmation. What
is the OB code for the pregnancy test only?

A. 81003

B. 81003, 81025

C. 81002, 81025

D. 81025

6. What is the name of the regulations, which were passed in


1988, to ensure quality standards for all laboratory testing?

A. Pathology and Laboratory Standards Regulations (PLSR)

B. Clinical Laboratory Improvement Amendments (CLIA)

C. Clinical Pathology Regulations (CPR)

D. Molecular Diagnostic Measurement Amendments (MDMA)


7. The patient presents to the laboratory for suspected heavy
metal poisoning. The actual amounts of metals detected were not
indicated. What laboratory code(s) are appropriate?

A. 83018

B. 83015
C. 82175, 83825, 83885

D. 80050

8. During the surgical session, the pathologist received and


performed analyses on three separate biopsies from the vaginal
wall and one from the cervix. How should the surgical pathologist
code for this service?

A. 88305, 88307

B. 88305 (X3)

C. 88305 (X4)

D. 88305 (X3), 88307

9. Level 1 studies in the surgical pathology section of the


Laboratory and Pathology chapter of the CPT manual describe:

A. Gross and Microscopic Examination


B. Quantitative Examination

C. Qualitative Examination

D. Gross Examination Only


10. An forensic pathologist performed a gross post=mortem
examination on a stillborn infant. The exam included the infant’s
brain, but did not include the infant’s spinal cord. How should the
pathologist code for this service?

A. 88005

B. 88012

C. 88025

D. 88014

11. The surgical pathologist obtained three skin tags samples


from in the patient’s right axilla, as well as four samples of breast
tissue from the surgical session. The surgical procedure was a
radical bilateral mastectomy, including excision of regional lymph
nodes and surrounding tissue. How should the surgical pathologist
code for this service?

A. 88304 (X7)

B. 88304 (X3), 88307 (X4)

C. 88305 (X3), 88307 (X4)


D. 88304 (X3), 88309 (X4)

12. A comprehensive metabolic panel and a general health panel


is ordered and performed on a patient. What is the correct CPT
coding for these laboratory orders?

A. 80053, 80050
B. 80050

C. 80050, 80051

D. 80050, 80048

13. A 26 year old must have a drug screen performed in order to


obtain a job at a local nursing home. The nursing home orders a 7-
panel drug screen for amphetamines, benzodiazepines, cocaine,
heroin, methadone, opiates, and oxycodone. If any of these come
up positive by direct optical observation, then definitive drug testing
is ordered as reflex testing. This patient did not have any positives
on optical observation so no additional testing was performed.
What is the correct CPT code for this drug screen?

A. 80300

B. 80301

C. 80302

D. 80303
14. A patient who has been on Digoxin needs to have
labs drawn to verify his response to the medication. A total
Digoxin level is ordered. What is the correct CPT code for
this test?

A. 80162

B. 80163

C. 80159

D. 80178
15. A patient is on two medications needing monitoring. She
needs to have a free and total phenytoin drug assay performed.
She also needs to have a Topiramate drug assay performed. What
are the correct CPT codes for these tests?

A. 80184, 80185, 80201

B. 80183, 80184, 80185

C. 80185, 80186, 80201

D. 80185, 80186, 80200

16. A patient is on Quinidine and needs to have therapeutic


drug assay testing completed to verify that he is on the optimal
dosage. What is the correct CPT code for this test?

A. 80190

B. 80194

C. 80195
D. 80197

17. A patient needs to have an ACTH stimulation panel


performed to assess adrenal insufficiency. What is the code for
reporting the laboratory component of the testing protocol that
should be used for this test?
A. 80400, 82533

B. 80400, 82533 x 2

C. 80400, 82533 x 3

D. 80400, 84143 x 2

18. A patient needs to have a TRH (thyrotropin releasing


hormone) stimulation panel performed over one-hour duration.
What is the correct reporting code for the laboratory component of
this testing panel?

A. 80438, 84443 x 3

B. 80438, 84443 x 2

C. 80439, 84443 x 3

D. 80439, 84443 x 4
19. A patient needs to have an insulin tolerance panel done to
assess ACTH insufficiency. What is the appropriate reporting
code(s) for the laboratory component of this testing?

A. 80432, 84681 x 5, 82947 x 5

B. 80432, 84681 x 4, 82947 x 2

C. 80434, 82533 x 5, 82947 x 5

D. 80434, 82533 x 4, 82947 x 4


20. A physician requests a clinical pathology consultation on one
of his cancer patients on whom he would like to have an in-depth
analysis done. He would like the pathology findings to be further
defined in conjunction with that patient’s medical history and
records. The pathologist complies with an in- depth review and
written report of his medical interpretive judgment. What is the
correct code for this type of consultation?

A. 80439

B. 80500

C. 80502

D. 81000

21. A pathologist has been requested to consult with general


surgery about lymph node findings. He doesn’t know the patient’s
medical history or have access to the medical records so this
consultation is considered to be limited. What is the correct CPT
code for this consultation?
A. 80500

B. 80502

C. 81000

D. 81005
22. A primary care physician contacts the pathologist at the local
hospital because of an anomaly on his patient’s pathology report.
He asks the pathologist to write a medical interpretive judgment,
without ever seeing the patient or knowing more about the patient’s
history. The pathologist is happy to do this, reexamines the
pathology sample and writes a report that he shares with the
primary care physician for further clarification. What is the correct
code for this type of a consultation?

A. 80400

B. 80402

C. 80500

D. 80502

23. A patient is in his primary care doctor’s office and needs to


have a urinalysis done. They do a dip stick in the office that is not
automated and doesn’t require microscopy. What is the CPT code
for this procedure?

A. 81000
B. 81001

C. 81002

D. 81003

24. A young woman goes to her doctor’s office to have a urine


pregnancy test performed. They can do this test by visual color
comparison only at the office. What is the correct CPT code for
this test?
A. 81000

B. 81005

C. 81015

D. 81025

25. A patient with a family history of breast cancer has a BRCA1


gene analysis ordered. The physician wants the full sequence
analysis and common duplication/deletion variants to be
assessed. What is the correct CPT code for this test?

A. 81211

B. 81212

C. 81214

D. 81216

26. A patient’s physician has had a hard time finding the correct
drug to put the patient on. Thinking that it has to do with the way
the patient’s body metabolizes the drug, she orders a CYP2C19
gene analysis. What is the correct CPT code for this test?

A. 81220

B. 81225

C. 81226

D. 81227
27. Due to a previous child having some fetal congenital
abnormalities, the OB/GYN ordered a biochemical assay of four
analytes (AFP, uE3, hCG, DIA) to assess for fetal congenital
abnormalities of pregnancy #3. This is done by using the mother’s
blood and is reported with a risk score for the provider to interpret.
What is the correct CPT code for this test?

A. 81508

B. 81509

C. 81510

D. 81511

28. A 19 year old was given a lab slip to have a biochemical


assay performed of seven analytes to assess for type 2 diabetes.
What code should be used for this test?

A. 81504

B. 81506
C. 81507

D. 81508

29. A patient needs to have an oncology biochemical assay of


five proteins performed to assess for ovarian cancer. These
proteins include CA-125, apolipoprotein A1, beta-2 microglobulin,
transferring, and pre-albumin.
A. 81500

B. 81502

C. 81504

D. 81506

30. A mother is going to have alpha-fetoprotein (AFP) testing


done, using the amniotic fluid. What is the correct CPT code for this
specific test?

A. 82103

B. 82104

C. 82105

D. 82106

31. A patient with a history of low B-12 needs to have this lab
test repeated. What is the correct CPT code for Vitamin B-12
testing, which is also known as cyanocobalamin?

A. 82607

B. 82608

C. 82610

D. 82615
32. A patient needs to have a specific clotting factor tested for
von Willebrand disease. He needs factor VIII, multimetric analysis
testing performed. What is the correct CPT code for this laboratory
test?

A. 85210

B. 85245

C. 85246

D. 85247

33. A patient’s physician ordered a clot lysis time, involving


whole blood dilution. What is the correct CPT code for this test?

A. 85170

B. 85175

C. 85210

D. 85300
34. A patient’s physician ordered an erythrocyte
sedimentation rate (ESR) and requested that it be automated.
What is the correct CPT code for this laboratory test?

A. 85635

B. 85651
C. 85652

D. 85660

35. A patient needs to have a high sensitivity C-reactive protein


test performed. What is the correct CPT code for this laboratory
test?

A. 86140

B. 86141

C. 86146

D. 86147

36. A patient has an order for an Islet cell antibody laboratory


test. What is the correct CPT code for this particular test?

A. 86336

B. 86337
C. 86340

D. 86341

37. A patient has an order for an absolute CD4 T-cell count. What
is the correct CPT code for this test?
A. 86359

B. 86360

C. 86361

D. 86367

38. A patient needs to have serologic blood typing performed for


ABO. What is the correct laboratory code for this test?

A. 86890

B. 86900

C. 86901

D. 86902

39. A patient needs to have an antihuman globulin test (Coombs


test) performed, which needs to be done as a qualitative indirect
test on each reagent red cell. What is the correct CPT laboratory
code for this specific test?
A. 86880

B. 86885

C. 86886

D. 86890
40. What is the correct CPT code for splitting of blood or blood
products per unit?

A. 86965

B. 86975

C. 86985

D. 86999

41. A patient needs to have a culture done of his stool looking


for Salmonella and Shigella. This will be performed as an aerobic
stool sample with isolation and preliminary examination. What is
the correct CPT code for this laboratory test?

A. 87040

B. 87045

C. 87046

D. 87070
42. A patient needs to have a stool sample done, looking for ova
and parasites. What is the correct CPT code for this laboratory test
involving direct smears for concentration and identification?

A. 87169

B. 87172
C. 87177

D. 87181

43. A patient needs to have a culture done for chlamydia. What


is the correct code for this laboratory testing?

A. 87109

B. 87110

C. 87116

D. 87118

44. A physician who is performing an autopsy performed a gross


examination with the brain and spinal cord. What is the correct CPT
code for this necropsy procedure?

A. 88000

B. 88005
C. 88007

D. 88012

45. A gross examination and microscopic examination was


performed during an autopsy (necropsy) of an infant with the brain
also examined. What is the correct CPT code for this procedure?
A. 88012

B. 88020

C. 88028

D. 88029

46. The brushings from a bronchoscopy need to be tested. The


ordering physician requests a cytopathology smear of the
brushings with interpretation to be performed. What is the correct
CPT code for this laboratory test?

A. 88104

B. 88106

C. 88108

D. 88112
47. A patient needs to have forensic cytopathology run on his
sperm. What is the correct laboratory CPT code for this test?

A. 88125

B. 88130

C. 88140

D. 88141
48. A physician orders a flow cytometry to be performed
for DNA analysis. What is the correct laboratory CPT code
for this test?

A. 88182

B. 88184

C. 88185

D. 88187

49. A patient is given a lab slip ordering chromosome analysis for


breakage syndromes specifically for baseline Sister Chromatid
Exchange (SCE) of 20-25 cells. What is the correct CPT code for
this laboratory test?

A. 88245

B. 88248

C. 88249
D. 88261

50. A patient has a laboratory order for interpretation and


report of cytogenetics and molecular cytogenetics. What is
the correct CPT code for this laboratory test?
A. 88240

B. 88241

C. 88291

D. 88299

51. A physician needs to order a test for tissue culture for non-
neoplastic disorders using amniotic fluid or chorionic villus cells.
What is the correct CPT code for this laboratory test?

A. 88230

B. 88233

C. 88235

D. 88237

52. A couple is trying to conceive. The man is given a laboratory


order for semen analysis for volume, count, motility and
differential using strict morphologic criteria (e.g., Kruger). What is
the correct CPT code for this test?

A. 89300

B. 89320

C. 89321

D. 89322
53. A woman wants to have her eggs cryopreserved. This is
going to be done using mature oocytes. What is the correct CPT
code for this procedure?

A. 89335

B. 89337

C. 89352

D. 89398

54. What is the correct CPT code for the insemination of oocytes?

A. 89264

B. 89268

C. 89272

D. 89280
MEDICAL TERMINOLOGY (33 QUESTIONS)

1. The fallopian tubes are two tubes on each side of the uterus
that carry the ovum from the ovaries to the uterus. There are three
other terms for these tubes other than “fallopian.” Which of the
following terms does NOT refer to the fallopian tubes?

A. Oviducts

B. Uterine Tubes

C. Salpinges

D. Cervix Uteri

2. During delivery, an episiotomy may be performed in order to


allow a larger opening for the baby to pass through. In this
procedure, the obstetrician will incise which area?

A. Vagina

B. Labia Minora

C. Labia Majora

D. Perineum
3. A procedure which takes a cone of cervical tissue for biopsy is
referred to as what?

A. Cervical Hysterectomy

B. Cervical Conization
C. Loop-Electrode Excision

D. Papanicolaou Smear

4. What are the three major components of an intestinal


transplant?

A. Cadaver Enterectomy, Backbench Work, and Biopsy of Lesion

B. Living Donor Enterectomy, Preparation of the Intestine, and


Transplantation with Enterectomy

C. Cadaver Enterectomy, Biopsy of Intestinal Wall, and


Allotransplantation

D. Cadaver Anastomosis, Backbench Work, and Recipient


Allotransplantation

5. Which gland not only produces thymosin, but also stimulates


T-cells to produce the hormones needed to regulate immune
system function? This gland is composed of two lobes that are
identical. This gland usually shrinks after puberty when it is
replaced by fat, but still continues to produce T-cells
A. Thymus

B. Thyroid

C. Parathyroid

D. Adrenal

6. The anterior segment of the eyeball, directly behind the


cornea, is filled with a clear, salty fluid called:
A. Aqueous humor

B. Scleral fluid

C. Viscous jelly

D. Vitreous humor

7. The system that produces tears is called the:

A. Tear Duct System

B. Lacrimal System

C. Lacrimal Punta

D. Tear Glands

8. The procedure known as blepharoplasty is performed to:

A. Correct the muscle misalignment caused by strabismus


B. Correct vision loss due to glaucoma

C. Plastic repair a droopy eyelid

D. Repair the lens of the eye caused by cataracts

9. The external ear contains the:


A. Auricle and Auditory Canal, leading up to the Tympanic
Membrane

B. Incus, Stapes, and Malleus

C. Cochlea, Semicircular Canals, and Eustachian Tube

D. Branches of the Vestibulocochlear Nerve and Tympanic


Membrane

10. Mastoiditis is inflammation of the middle ear, a condition that


can occur when a chronic ear infection goes untreated. If a
cholesteatoma develops as a result of a chronic ear infection, the
surgeon may elect to remove the inflamed area. This procedure is
referred to as which of the following:

A. Cochlear Implantation

B. Ossicular Surgery

C. Mastoidectomy

D. Retrobulbar Injection
11. This organ is a five-inch tube located behind the mouth that
helps close the nasopharynx and larynx when swallowing food.
This organ keeps your food out of your respiratory tract and in your
digestive tract. What is it called?

A. Esophagus

B. Pharynx

C. Nasopharynx
D. Trachea

12. This condition occurs when the lining of the esophagus


becomes inflamed. It is generally caused by an infection or
irritation of the esophagus. What is the name of this condition?

A. Esophagitis

B. Barrett’s Esophagus

C. Esophageal Varices

D. Mallory-Weiss Tear

13. A dilated and enlarged varicose vein that developed inside


the rectum and slipped outside of the anus is called a what?

A. External Hemorrhoid

B. Internal Hemorrhoid

C. Prolapsed Hemorrhoid
D. Varicose Hemorrhoid

14. Within the male genital system, the pair of tubular glands
located above the prostate and behind the bladder that lubricate
the duct system, nourish the sperm, and contribute fluid to the
ejaculate are called:
A. Seminal Vesicles

B. Testes

C. Vas Deferens

D. Epididymis

15.
One of the most common prostatic disorders is ,
which is an enlargement of the prostate gland. This disorder may
require a transurethral resection of the prostate (TURP).

A. Lower Urinary Tract Symptoms (LUTS)

B. Benign Prostatic Hyperplasia (BPH)

C. Elevated Prostate Specific Antigen (PSA)

D. Prostatic Intraepithelial Neoplasia III (PIN III)


16. There are four different classifications of a vulvectomy (the
removal of the vulva). What classification should be used for a
vulvectomy with the removal of skin and deep subcutaneous
tissues?

A. Simple Vulvectomy

B. Radical Vulvectomy

C. Partial Vulvectomy

D. Complete Vulvectomy
17. What is the muscular tube that carries urine from the kidneys to
the bladder?

A. Urethra

B. Ureter

C. Urinary bladder

D. Kidney

18. Which of the following is the most common voiding disorder?

A. Fecal Incontinence

B. Cholelithiasis

C. Kidney Stones

D. Urinary Incontinence
19. The spleen, which is part of the hemic system, does NOT
perform the following functions:

A. Creates new red blood cells

B. Recycles iron

C. Holds a reserve of blood in case of hemorrhage

D. Creates stem-cells for use throughout the body


20. A blood disease that is characterized by an abnormal
increase in the amount of white blood cells in the body is normally
referred to as:

A. Lymphadenitis

B. Thrombophlebitis

C. Leukemia

D. Lymphoma

21. Which of the following terms means “red cell,” (a red blood
cell) that carries oxygen and carbon dioxide throughout the
bloodstream?

A. Leukocyte

B. Erythrocyte

C. Granulocyte

D. Thrombocyte
22. The lymphatic system contains four organs:

A. Spleen, tonsils, bone marrow, and thymus

B. Spleen, Peyer’s patches, and tonsils


C. Spleen, tonsils, Peyer’s patches, and thymus

D. Tonsils, Peyer’s patches, thymus, and bone marrow

23. What is the primary function of the lymph nodes?

A. To filter the blood and excrete it from the body in the form of
urine

B. To circulate red blood cells throughout the body

C. To create and store red and white blood cells

D. To filter lymph fluid to remove bacteria and viruses

24. The small muscles under the dermis that serve to help the
hair follicles stand on end when the body is chilled are called:

A. Hair follicle Muscles

B. Arrector pili Muscles


C. Sebaceous Muscles

D. Hair shaft Muscles

25. What does the Rule of Nines calculate?


A. The Depth of the Burn

B. The Extent of the Burn

C. The Site of the Burn

D. The Degree of the Burn

26. The tibialis anterior muscle controls the:

A. Finger Flexor Tendons in the Wrist

B. Frontal Flexion of the Upper Abdomen

C. Frontal Flexion of the Lower Leg

D. Forehead Furrow Lines

27. A nerve plexus is an intersection of spinal nerves serving the


same body area. There are many nerve plexuses in the body.
Which of the following is NOT a nerve plexus?
A. Cervical plexus

B. Lumbar plexus

C. Sacral Plexus

D. Thoracic Plexus
28. What is the difference between meningitis and encephalitis?

A. Meningitis is the inflammation of the spinal cord, and


encephalitis is the inflammation of the lining of the brain
B. Meningitis is the inflammation of the lining of the brain and
encephalitis is the inflammation of the brain
C. Meningitis is the inflammation of the lining of the brain and
encephalitis is the swelling of the spinal cord
D. Meningitis is a respiratory infection and encephalitis is the
inflammation of the nervous system

29. If a patient receives a chest x-ray with a posteroanterior


projection, in what direction are the x-ray beams traveling?

A. From the front of the patient to the back of the patient

B. From the back of the patient to the front of the patient

C. From the left side of the patient to the right side of the patient

D. From the right side of the patient to the left side of the patient
30. Contrast material is introduced into a patient in order to
better visualize the area being imaged. What are the three main
ways that contrast material can be introduced?

A. Intravascularly, Intrathecally, and with an Angiogram


B. Intrathecally, Intravascularly, or Injected into a Vein

C. Intra-Articulary, Intravascularly, or Injected

D. Intravascularly, Intra-Articulary, or Intrathecally

31. A radiographic image of the colon’s interior is referred to as


which of the following?

A. Colonography

B. Colonoscopy

C. Duodenoscopy

D. Cholangiography

32. A is a procedure in which an


endoscope is passed through the oropharynx and vocal chords and
extended beyond the trachea into the right bronchus. This
procedure can be performed for either diagnostic or therapeutic
procedures.
A. Bronchoscopy

B. Laryngoscopy

C. Tracheoscopy

D. Oropharyngoscopy

33. What is the purpose of a mediastinotomy?


A. To remove part of the wall of the mediastinum

B. To remove part of the pericardial sac

C. To provide an opening for access to the mediastinum

D. To repair a rupture of the mediastinal wall


MEDICINE (85
QUESTIONS)

1.

PROGRESS

NOTE PATIENT:

TROHOSKE,

RUBY DATE:

12/14/2016

MEDICAL NOTE:
The patient was seen in the clinic today for a 5-year well-child
exam. The physical exam was unremarkable; the patient was at
normal growth and development for her age group. The following
vaccines were recommended for the patient at this visit: DTaP, IPV,
MMR, Varicella, and annual Influenza shot. What are the correct
codes for this visit?
A. 99393, 90471, 90700, 90471 (X4), 90713, 90707, 90716, 90658

B. 99392, 90471, 90700, 90472 (X4), 90713, 90707, 90716, 90657

C. 99393, 90471, 90700, 90472 (X4), 90713, 90707, 90716, 90658

D. 99393, 90471, 90700, 90472, 90713, 90707, 90716, 90660


2.

PROGRESS

NOTE PATIENT:

TROHOSKE,

RUBY DATE:

12/14/2016

MEDICAL NOTE:
The patient was seen in the clinic today for a 5-year well-child
exam. The physical exam was unremarkable; the patient was at
normal growth and development for her age group. The following
vaccines are recommended for patient at this visit: DTaP, IPV,
MMR, Varicella, and annual Influenza shot. The pediatrician
counseled the patient and her parents on the health risks and side-
effects associated with each recommended vaccination. What is
the correct code for these vaccines?
A. 90460, 90461 (X2), 90700, 90460, 90713, 90460, 90461 (X2),
90707, 90460, 90716, 90460, 90658

B. 90460, 90461 (X8), 90700, 90713, 90707, 90716, 90658

C. 90460 (X5), 90461 (X5), 90700, 90713, 90707, 90716, 90658

D. 90461, 90460 (X2), 90700, 90461, 90713, 90461, 90460 (X2),


90707, 90461, 90716, 90461, 90658

3. The patient is a 20-month-old with end-stage renal disease


receiving dialysis twice a week, awaiting a kidney transplant. The
patient’s parents receive weekly face-to-face counseling services
regarding the patient’s care and ESRD status. What is the correct
code for the patient’s dialysis care?

A. 90951 (X4)

B. 90951

C. 90952

D. 90954 (X4)
4. A 20-month-old with end-stage renal disease was receiving
dialysis twice a week, awaiting a kidney transplant. During the last
month of life, the patient received dialysis once per day. Dialysis
was administered between June 1 and June 14. The patient’s
parents received daily face-to-face counseling regarding the
patient’s care and ESRD status. What is the correct code for the
patient’s dialysis care?

A. 90968 (X14)

B. 90960

C. 90967 (X14)
D. 90964

5. An adolescent patient’s hard contact lenses were shattering


when a bouncy ball that he was playing with at school bounced up
at hit him in the eye. The patient was transported to the emergency
department where the on-call physician removed the contact lens
fragments from his cornea, using a slit lamp. What is the correct
code for the procedure?

A. 65205

B. 65210

C. 65220

D. 65222

6. The physician removed the lens of a patient with severe


cataracts via the pars plana approach, without a vitrectomy. During
the operative session, the physician used viscoelastic agents and
enzymatic zonulysis to aid in the procedure. How should the
physician code for the procedure?
A. 66852

B. 66840

C. 66983

D. 66852, 66982
7. The patient with severe strabismus visited the
ophthalmologist clinic for a strabismus correction surgery. The
ophthalmologist performed the surgery by correcting two
horizontal muscles and one vertical muscle in the patient’s right
eye. The physician then used a transposition procedure on the
superior oblique muscle in order to further correct the
misalignment. What is the correct way to code for the procedure?

A. 67311 (X2), 67320

B. 67312, 67314, 67320

C. 67312, 67314, 67318

D. 67312, 67314, 67320-51

8. The physician performed trichiasis on a patient with


severely ingrown lower eyelashes. Utilizing forceps and
biomicroscope, the physician removed four offending lower
lashes and one misdirected upper eyelash. What is the correct
code for the procedure?

A. 67825
B. 67830

C.

67820-

RT D.

67820-

50

9. Mr. Longoria, a patient with an established cataract, was


displeased with his current ophthalmologist so today he was seen
by a new physician. The ophthalmologist performed an
ophthalmoscopy and tonometry on Mr. Longoria, as well as an
external examination of both eyes and a review of the patient’s
interval history. What is the correct code for Mr. Longoria’s
ophthalmology service today?
A. 92002

B. 92004

C. 92012

D. 92014

10. The physician performed a bilateral tympanostomy with


placement of drainage tubes, in a 4 year old male, who required
general anesthesia. During the procedure a large abscess was
noted on the right external auditory canal. The physician incised
and drained the abscess, and placed ventilating tubes in each
tympanum. How would you code for the procedure?

A. 69436-50, 69020

B. 69433, 69020

C. 69436, 11000

D. 69436-50, 69020-50
11. A 58-year-old patient with decreased hearing decided to
undergo bilateral cochlear device implantation in order to restore
the gradual decline of his hearing, and prevent total deafness.
Due to the position of the device and the size of the patient’s
mastoid bone that was partially occluding the patient’s inner ear,
the physician performed a modified mastoidectomy. A
mastoidectomy was necessary for the completion of the cochlear
device implantation procedure. The physician used an operating
microscope throughout the cochlear implantation. How would you
code for the procedure?
A. 69930, 69505, 69990

B. 69930, 69501, 69900

C. 69930

D. 69930-50, 69900

12. A patient with a suspected cardiac arrhythmia was given a


routine rhythm EKG with three leads in the cardiologist’s office.
The results of the EKG were sent to the patient’s primary care
physician, who interpreted the results and determined that the
patient’s arrhythmia was mild. What is the correct code for the
cardiologist’s office service only?

A. 93040

B. 93042

C. 93041

D. 93000
13. Mr. Johnson was seen in his primary care physician’s office
today for an evaluation of chest pains. He has been experiencing
shortness of breath as well as intermittent chest pains for the past
week. He has also been experiencing tingling and numbness in his
left hand and fingers. His primary care physician suspects a
cardiac rhythm abnormality and performs a rhythm
electrocardiogram in the office. After reading the ECG report, Mr.
Johnson’s physician determines that his heart rhythm needs more
evaluation and refers him to a cardiac specialist. How should the
electrocardiogram code be reported?

A. 93000
B. 93010

C. 93042

D. 93040

14. The patient has a dual-chamber pacemaker and was


recently seen by his physician for analysis and report of his
implanted device. During the visit, the physician connected the
system, evaluated its performance, and then disconnected it.
What is the correct code for this service?

A. 33213

B. 93289

C. 33208

D. 93288

15. The physician performed a right and left heart catheterization


with a left ventriculography on 58- year-old male patient. During
catheterization, the patient participated in a physiologic exercise
study in the form of a bicycle ergometry. How should you code for
this service?

A. 93531, 93464

B. 93451, 93452, 93464

C. 93453, 93464

D. 93453
16. A pediatric patient presented to the office with a severe
asthma attack. The pediatrician ordered a pulse oximetry to check
the patient’s blood oxygen saturation level and a spirometry to
evaluate her lung capacity. The physician interpreted the results
and ordered an albuterol nebulizer treatment and a post-spirometry
to check the patient’s responsiveness to the albuterol treatment.
What are the correct codes for this office visit?

A. 99214, 94060, 94760, 94640,


A7015, A4616, J7630

B. 99214, 94010 (X2), 94760, 94640, A7015, A4616, J7630

C. 99214, 94060, 94640, A7015,


A4616, J7630

D. 99214, 94010, 94760, 94640,


A7015, A4616, J7630

17. The endocrine section in the CPT manual (60000 to 60699)


does not include all of the glands in the endocrine system. Which
of the following glands are missing from this section?

A. Testes and Ovaries


B. Pineal Gland

C. Pituitary Gland

D. All of the Above

E. Only Options B and C


18. What is the correct code for the endocrinology service
located in the medicine section that allows for the ambulatory
continuous glucose monitoring of interstitial tissue fluid (includes
the physician’s reading and interpretation of the report’s findings
over a 72-hour period)?

A. 95250

B. 95251, 99091

C. 95251

D. 95250, 99091

19. The patient was seen in the ER clinic for severe dehydration
after a stomach virus. The patient received an intravenous infusion
of fluids and electrolytes in the ER. Infusion was began at 3:00 PM
and ended at 6:15 PM. What is the correct code for this infusion
service?

A. 96360, 96361 (X3)

B. 96360, 96361 (X2)

C. 96360 (X3)
D. 96360, 96361 (X4)

20. The physician ordered tests to evaluate a three-year-old


patient for cystic fibrosis. The test results came back as suspicious
so the physician ordered additional tests to confirm the diagnosis.
The physician ordered his nurse to call the patient’s parents to
discuss the patient’s results and any other recommended tests.
The phone conversation lasted 20 minutes during which time the
nurse scheduled a follow-up appointment for the next morning at
10:00 AM. How should you report the telephone services for this
phone call?
A. No telephone consultation service

code should be reported B. 98967


C. 98968

D. 99442

21. The patient was seen in the after-hours clinic on a Monday


evening at 5:30 PM due to a severe cough and trouble breathing.
This is his normal clinic, which usually stays open late to
accommodate patients so they do not have to go to the Emergency
Room. What additional code should you append to the E&M
service or any other services provided during the visit?

A. 99050

B. 99051

C. 99053

D. 99058
22. A 6-year-old boy needs to have his influenza vaccine for the
2016 flu season. What is the correct CPT code(s) for nasal
administration of the quadrivalent influenza vaccine, where the
physician counsels the parents about the administration route?

A. 90460, 90660

B. 90471, 90672
C. 90473, 90660

D. 90460, 90672

23. A psychiatrist is evaluating a patient in his office for an


initial visit. The physician billed for a psychiatric diagnostic
evaluation and he is using medical records and family reports to
evaluate this patient, including ordering diagnostic studies. What
is the CPT code for this visit?

A. 90785

B. 90791

C. 90792

D. 90832

24. A patient is being admitted to a psychiatric facility for suicidal


ideation. The patient undergoes psychotherapy for crisis and the
doctor spends 70 minutes with this patient. What is the correct
code(s) for this visit?
A. 90839

B. 90839, 90840

C. 90839, 90840 x 2

D. 90839, 90840 x 3
25. A patient is undergoing hypnotherapy at the psychiatry office.
What is the correct CPT code for this procedure?

A. 90870

B. 90875

C. 90880

D. 90885

26. A patient undergoes biofeedback to help with chronic pain.


What is the correct CPT code for this procedure?

A. 90875

B. 90876

C. 90901

D. 90911
27. A patient with bowel incontinence undergoes anorectal
biofeedback training. What is the correct CPT code for this
procedure?

A. 90875

B. 90876

C. 90901
D. 90911

28. A patient is undergoing heart variability biofeedback.


What is the correct code for this type of biofeedback therapy?

A. 90901

B. 90911

C. 90875

D. 90876

29. A patient who has dialysis three times a week as an


outpatient goes in for a routine hemodialysis appointment. What is
the correct code for this procedure where he only requires a single
evaluation by a physician?

A. 90935

B. 90937
C. 90940

D. 90945

30. A patient needs to have a gastric manometric (motility)


study performed. What is the correct CPT code for this
procedure?
A. 91010

B. 91013

C. 91020

D. 91022

31. A patient who has been having chronic reflux is scheduled to


have an acid perfusion test performed on the esophagus. This test
is also known as the Bernstein test. What is the correct CPT code
for this procedure?

A. 91022

B. 91030

C. 91034

D. 91035
32. A patient needs to have a capsule endoscopy of the
esophagus through the ileum. What is the correct CPT code for
this test?

A. 91040

B. 91110

C. 91111

D. 91112
33. A patient has been having difficulties swallowing. He is set
up for a flexible fiberoptic endoscopy evaluation of swallowing
by video recording. What is the correct CPT code for this
procedure?

A. 92610

B. 92611

C. 92612

D. 92614

34. A patient who undergoes hemodialysis three times a week


needs to have his access scanned to make sure the blood flow is
patent. What is the correct code for a duplex scan of hemodialysis
access?

A. 93970

B. 93975
C. 93990

D. 93998

35. A patient needs a duplex scan of extracranial arteries. The


vascular surgeon orders a complete bilateral study instead of a
unilateral or limited study. What is the correct CPT code for this
procedure?

A. 93880
B. 93882

C. 93886

D. 93888

36. A patient with history of a small aortic aneurysm needs to


have a complete duplex scan of the aorta, inferior vena cava, and
iliac vasculature. What is the correct CPT code for this procedure?

A. 93975

B. 93978

C. 93979

D. 93980

37. A patient who has been having shortness of breath on


exertion is scheduled for a simple pulmonary stress test. What is
the correct CPT code for this procedure?
A. 94610

B. 94620

C. 94621

D. 94640
38. A patient is being set up with a continuous positive airway
pressure (CPAP) machine upon arriving to the unit from the
emergency department. What is the correct code for CPAP
ventilation initiation and management?

A. 94644

B. 94645

C. 94660

D. 94662

39. A pediatric patient with suspected food allergies is sent for


allergy testing that involves scratch tests over his back. He is
tested for eight different foods and there are two control tests
performed. What is the correct code for percutaneous (scratch)
tests with allergenic extracts?

A. 95004

B. 95017

C. 95018

D. 95024
40. A three year old is going to have a peanut challenge
performed in the allergist’s office under direct supervision. The
child is scheduled for three hours of testing, what is the correct
code(s) for this testing?

A. 95076
B. 95076, 95079

C. 95076, 95079 x 2

D. 95076, 95079 x 3

41. A patient is undergoing de-sensitivity for bee stings, which


involves allergen immunotherapy with a single stinging insect
venom. What is the correct code for this procedure?

A. 95120

B. 95125

C. 95130

D. 95131

42. A patient needs to have continuous glucose monitoring for


four days. What is the correct code for placement, hook up,
calibration of the monitor, patient training and removal of sensor
and printout of the findings? Patient will set up an additional
appointment to have endocrinologist interpret and report the
findings.

A. 95180

B. 95199

C. 95250

D. 95251
43. A patient needs to have a sleep study performed to assess
for sleep apnea. He will be spending the night at the center and
have his ventilations, respiratory effort, ECG and oxygen saturation
monitored. A technologist will be in attendance for this. What is the
correct CPT code for this procedure?

A. 95806

B. 95800

C. 95801

D. 95807

44. A patient is scheduled for an electromyography (EMG)


of all four extremities to assess for myasthenia gravis. What
is the correct code for this procedure?

A. 95860

B. 95861

C. 95863

D. 95864
45. A patient needs to have an electroencephalography (EEG)
performed while awake and asleep. What is the correct code for
this procedure?

A. 95816

B. 95819
C. 95822

D. 95824

46. A newborn with congenital malformations is undergoing


genetic testing to try to identify the specific genes involved. The
parents and infant have a 90-minute appointment scheduled with a
geneticist to review the family genetic history and assessment for
genetic concerns. What is the correct CPT code for this visit?

A. 96040

B. 96040 x 2

C. 96040 x 3

D. 96040 x 4

47. A patient has a quick follow-up appointment with genetics for


counseling services. His appointment only takes 20 minutes. What
is the correct CPT code for this service?
A. 96040

B. 96040 x 2

C. 96040 x 3

D. 96040 x 4
48. Parents of a two year old, who has been undergoing genetic
testing, have a follow-up appointment scheduled to go over the
newest labs and have genetic counseling. They spent 46 minutes
for this appointment as they make a plan of action and begin
getting referrals to child development services. What is the correct
code for this visit?

A. 96040

B. 96040 x 2

C. 96040 x 3

D. 96040 x 4

49. A patient has to have a neurobehavioral status examination.


What is the correct code for this test?

A. 96110

B. 96111

C. 96116

D. 96118
50. A pediatric patient, who is having difficulties in school and
showing signs of being on the autism spectrum, needs to have
developmental testing performed. What is the correct CPT code
for this testing?

A. 96110
B. 96111

C. 96116

D. 96118

51. A patient is undergoing neuropsychological testing


performed by a computer with qualified healthcare professional
interpretation and report. What is the correct CPT code for this
test?

A. 96119

B. 96120

C. 96125

D. 96127

52. A patient has an initial 15-minute health and behavior


assessment with the provider. What is the correct code for this
initial assessment?
A. 96150

B. 96151

C. 96152

D. 96153
53. A provider is meeting with a patient’s family, without the
patient present, to plan a health and behavior intervention.
What is the correct code for this?

A. 96152

B. 96153

C. 96154

D. 96155

54. Health and behavior interventions are planned and discussed


with a group of four patients. What is the correct CPT code for this?

A. 96152

B. 96153

C. 96154

D. 96155
55. A patient needs to have D5-1/2 normal saline + 30mEq
KCl/liter given. However, it needs to be infused slowly and is
given over 90 minutes. What is the correct code for this
hydration procedure?

A. 96360

B. 96361

C. 96360, 96361
D. 96360, 96361 x 2

56. A patient is given an intramuscular injection of Ancef prior to


his procedure. What is the correct CPT code for the subcutaneous
or intramuscular therapeutic, prophylactic or diagnostic injection?

A. 96365, J0690

B. 96369, J0690

C. 96372, J0690

D. 96374, J0690

57. A patient needs to have an intravenous push of an initial


dose of Versed. Five minutes later, he needs a second
intravenous push of the same drug. What is the correct code for
this procedure?

A. 96372, J2250 x 2

B. 96374, 96376, J2250 x 2


C. 96374, 96375, J2250 x 2

D. 96379, J2250 x 2

58. A patient is receiving chemotherapy through her intra-arterial


access port. She is scheduled for an infusion that will last about 50
minutes. This infusion will be for the drug methotrexate sodium
and she will receive 50 mg. What is the correct code for this
procedure?
A. 96420, J9260

B. 96422, J9260

C. 96423, J9260

D. 96425, J9260

59. A patient with breast cancer is undergoing chemotherapy


treatment with a drug called Fluorouracil and she receives 500 mg
through her PICC line over a 4-hour timeframe. What is the correct
code for this type of administration?

A. 96420, J9190

B. 96422, J9190

C. 96423, J9190

D. 96422, 96423 x 3, J9190


60. A patient with acute lymphoblastic leukemia is being
given a drug called Asparaginase. This is administered 1000
units intramuscularly. What is the correct code for this
chemotherapy administration?

A. 96401, J9019

B. 96402, J9019

C. 96405, J9019
D. 96409, J9019

61. A patient has a malignant lesion on his face with adjacent


mucosa (lip) involvement. He is set to undergo photodynamic
therapy by external application of light to destroy this malignancy.
What is the correct code for this procedure?

A. 96567

B. 96570

C. 96571

D. 96900

62. A patient needs to have a transoral flexible esophagoscopy


to treat abnormal tissue in the esophagus. This will be done by
providing photodynamic therapy applying light to ablate the
abnormality by activating photosensitive drugs. This patient’s
procedure takes 22 minutes. What are the correct CPT codes for
this procedure?
A. 96567

B. 43200, 96567, -52

C. 43200, 96567

D. 43200, 96571
63. A patient has a bronchoscopy scheduled to ablate abnormal
tissue by using photodynamic therapy by endoscopic application
of light to activate photosensitive drugs. This patient’s procedure
takes 50 minutes to complete. What is the correct code(s) for this
procedure?

A. 31622, 96570

B. 31622, 96570, 96571

C. 31623, 96570

D. 31623, 96570, 96571

64. A patient has been losing a large amount of hair. The


provider plucks a few hairs from the patient for microscopic
evaluation to determine telogen and anagen counts. What is the
correct code for this procedure?

A. 96900

B. 96902

C. 96904

D. 96910
65. A patient has been receiving laser treatments for psoriasis
for a total area of 350 sq. cm. What is the correct code for this
procedure?

A. 96913
B. 96920

C. 96921

D. 96922

66. A patient with skin cancer is undergoing actinotherapy. What is


the correct code for this procedure?

A. 96900

B. 96902

C. 96910

D. 96913

67. A patient needs to have an initial evaluation for physical


therapy. What is the correct CPT code for this evaluation?

A. 97001
B. 97002

C. 97003

D. 97004

68. A patient is referred to an athletic trainer and began a


specific plan of care. About one month into the care, he needs to
be re-evaluated. What is the correct code for athletic training re-
evaluation?
A. 97001

B. 97002

C. 97005

D. 97006

69. A patient needs to have an initial assessment by


occupational therapy after suffering from a stroke. What is the
correct code for this evaluation?

A.

Z51.5

B.

Z51.8

C.

Z51.8

1 D.
Z51.8

70. A patient receives 15 minutes of face-to-face medical


nutrition therapy. What is the correct CPT code for this?

A. 97802

B. 97803

C. 97804

D. 97810
71. A provider is doing a 30-minute group session for medical
nutrition therapy to cover the basis of nutrition needs in patients
with congestive heart failure. What is the correct CPT code for
this type of therapy?

A. 97802

B. 97803

C. 97804

D. 97810

72. A patient who has had medical nutrition therapy in the past
and is struggling to retain the information, is scheduled for re-
assessment and intervention. The appointment is only scheduled
for 15 minutes. What is the correct code?

A. 97802

B. 97803

C. 97804
D. 97810

73. A patient is receiving osteopathic manipulative treatment


(OMT) of the thoracic and lumbar regions. What is the correct CPT
code for this procedure?
A. 98925

B. 98926

C. 98927

D. 98928

74. A patient is undergoing osteopathic manipulative treatment


(OMT) of her head, cervical, thoracic, lumbar and sacral regions.
What is the correct CPT code for this treatment?

A. 98925

B. 98926

C. 98927

D. 98928

75. A patient has been having pain in his legs and arms and is
undergoing osteopathic manipulative treatment (OMT). He is
having this focused on his lower extremities, upper extremities,
cervical region, thoracic region, lumbar region, sacral region and
pelvic regions. What is the correct CPT code for this procedure?

A. 98926

B. 98927

C. 98928

D. 98929
76. A patient went to a chiropractor to have specific regions of
his back manipulated. He needed to have his cervical, thoracic and
lumbar region manipulated. What is the correct CPT code for this
procedure?

A. 98940

B. 98941

C. 98942

D. 98943

77. A patient needs to have his head manipulated by a


chiropractor. What is the correct code for this area of
manipulation?

A. 98940

B. 98941

C. 98942
D. 98943

78. A pregnant patient is having problems with her round


ligaments and increasing pain in her pelvic region. She visits a
chiropractor who performed chiropractic manipulation of the pelvic
region. What is the correct CPT code for this procedure?
A. 98940

B. 98941

C. 98942

D. 98943

79. A nurse practitioner is doing a chronic disease self-


management course for a group of six patients. This is a
standardized course and occurs over the course of six weeks for
30 minutes each week. Diabetes, congestive heart failure,
hypertension, hyperlipidemia, and chronic obstructive pulmonary
disease are some of the topics that are covered in this curriculum.
What is the correct code for education for the patient to promote
self-management?

A. 98960

B. 98961

C. 98962

D. 98966
80. A patient with diabetes is given a 30-minute course on initial
diabetes self-management. Following a specific curriculum, the
physician sees this patient in his office with the patient’s daughter
and husband to make sure everyone has been given the same
information to assist the patient at home. What is the correct CPT
code for this type of training/education?

A. 98960

B. 98961
C. 98962

D. 98966

81. A group of patients on the rehabilitation unit are given a self-


management course about congestive heart failure because all of
these patients have had exacerbations of this disease. A total of
three patients are able to attend these classes. The classes take
30 minutes and are given twice weekly. What is the correct code
for this type of self-management training?

A. 98960

B. 98961

C. 98962

D. 98966

82. A patient calls his primary nurse practitioner because he is


having abdominal pain and diarrhea. He speaks with his her for five
minutes and after obtaining more signs and symptoms, the nurse
practitioner suspects the patient is suffering from a gastrointestinal
virus. The patient is recommended to proceed with supportive care,
and if not improving in 48-72 hours, to follow up in the physician’s
office. The patient improved, so did not have to follow up in the
office. What is the correct CPT code for this type of telephone
service?

A. 98966

B. 98967

C. 98968
D. 98969

83. A 4-year-old patient is seen in the emergency department for


a facial laceration. Due to the age and the need for laceration
closure, the patient is given moderate sedation for 20 minutes
while the laceration is closed. The physician performing the
laceration repair is the same one giving the sedation and a nurse
is monitoring the patient’s vitals during this process. What is the
correct code only for the moderate sedation service?

A. 99143

B. 99144

C. 99145

D. 99148

84. A 20-year-old man fractures his arm in two places. He is seen


in the urgent care setting and the bone is reset. To do this, the
physician orders moderate sedation and repeat x-rays after the
procedure is done to ensure proper placement before casting. The
patient is given moderate sedation with one physician and one
nurse present and the procedure lasts 35 minutes. What is the
correct code for the moderate sedation service?

A. 99143

B. 99144

C. 99145

D. 99148
85. A 52-year-old patient requires moderate sedation after motor
vehicle accident to repair multiple lacerations and set multiple
broken bones. The sedation is performed in the emergency
department by the attending physician and monitored by a nurse.
The patient is under sedation for a total of 50 minutes. What is the
correct CPT code just for the moderate sedation services
performed?

A. 99143

B. 99144

C. 99145

D. 99144, 99145
RADIOLOGY (23
QUESTIONS)

1. The patient with possible brain damage received a functional


MRI of the brain with administration of repetitive body part
movements and visual stimulation. Due to the type of brain injury,
the physician was required to administer all of the neurofunctional
tests. The physician reviewed the test results and reported his
findings to the patient’s primary care physician. What is the correct
code for this visit?

A. 96020, 70554

B. 96020, 70555

C. 96020

D. 70555

2. What is the correct code for a complete chest x-ray?

A. 71020

B. 71034
C. 71030

D. 71020, 71021, 71022, 71023

3. The patient is being evaluated for spinal curvature problems


of the lower back. She has been sent to the radiologist for a set of
spinal x-rays. The radiologist takes x-rays from 4 different views of
her spine (standing straight, bending forward and from each side)
along with three additional views. These films are sent to the
patient’s PCP for interpretation and report. What code should the
radiologist report?
A. 72114 -TC

B. 72110 -TC, 72120 -TC

C. 72114

D. 72110, 72120

4. The patient was given an MRA of the pelvis with and without
contrast material. What is the correct code for the procedure?

A. 72195, 72196

B. 72196

C. 72198

D. 72197

5. A 38-year-old female patient with severe endometriosis


received a CT scan of the pelvis and the abdomen with and
without contrast. What radiology code should be reported?
A. 74176, 74177

B. 74178

C. 74170, 72194

D. 74177
6. A 65-year-old female patient with atherosclerosis receives
an abdominal aortography via serialography. The patient’s
cardiologist reviews and interprets the findings. What is the
appropriate code for this service?

A. 75625

B. 75625 -26

C. 75605

D. 75630

7. Radiation oncology codes consist of a specialized portion of


radiology codes that are used to report which of the following?

A. Radiation services provided by oncologists

B. Radiology services that are used to treat benign neoplasms

C. Radiological services that are used to deliver radiation


treatments to tumors

D. Radiation services that are provided in hospitals


8. ONCOLOGY

CLINIC NOTE

PATIENT NAME:

Watts, Douglas

AGE: 15
DATE: 07/08/2014

DIAGNOSIS: Mantle Cell Lymphoma in the Neck


TREATMENT: Clinical Treatment Planning stage I including
simulation of treatment

The patient was recently diagnosed with mantle cell lymphoma in


the neck. He was seen in the clinic today for the first stage of
clinical treatment planning. The patient’s first step is to start
radiation treatments. Due to the mild nature of the lymphoma,
planning will focus on a single treatment area, with a single
radiation port and no blocking. The primary care oncologist agrees
that this focus is the best way to manage the malignancy before it
progresses. No further aggressive treatments are necessary at this
time. The patient and his family also agree that this will be a
sufficient means of treatment at this time. Simple simulation of
single treatment area was completed and the treatment plan
seems sufficient.
The patient is scheduled to return to the clinic in two days, after
dosimetry calculations have been completed and the total length
of time for the radiation treatment has been calculated.

What is the correct code for this service?

A. 77263, 77280

B. 77262, 77280
C. 77261, 77280

D. 99215, 77280

9. ONCOLOGY

CLINIC NOTE

PATIENT NAME:

Watts, Douglas

AGE: 15
DATE: 07/10/2014
DIAGNOSIS: Mantle Cell

Lymphoma in the Neck

TREATMENT: Dosimetry

A physician recommends clinical treatment planning, simple


simulation of treatment, basic radiation dosimetry calculations
(including central axis depth calculation, TDF, NSD, gap
calculation, off-axis factor, and calculation of non-ionizing radiation
surface) and depth dosing during clinical treatment planning. The
amount of radiation is calculated and blocked to protect the
remaining healthy tissues in the neck, shield the rest of the body
from radiation, determine the dosage delivery and fractionate the
radiation into treatments. The patient is scheduled to receive three
treatments (once per week for the first two weeks) and once per
month for the next three months. An simple treatment device
delivers the required dosage to the port.
What is the correct code for this procedure?

A. 77300, 77332 -51

B. 77301, 77332 -51

C. 77300

D. 77301, 77333
10. A young woman with right lower abdominal pain and history
of ovarian cysts is ordered to have a transvaginal ultrasound to
rule out ovarian cyst as the cause of her pain. What is the correct
CPT code for this diagnostic ultrasound?

A. 76801

B. 76830

C. 76856
D. 76857

11. A pregnant woman needs to have a fetal echocardiogram


performed. This will be done in real time with image documentation
(2D) with M-mode recording. What is the correct code for this
procedure?

A. 76820

B. 76825

C. 76826

D. 76827

12. A patient needs to have an epidural and, due to


compressed discs, this procedure is done under fluoroscopic
guidance. What is the correct code for this type of fluoroscopic
procedure?

A. 77002
B. 77003

C. 77011

D. 77012

13. A patient undergoes magnetic resonance guidance for


needle placement for placement of a localization device. What
is the correct code for this procedure?
A. 77011

B. 77012

C. 77021

D. 77022

14. A physician orders a fluoroscopic guidance for needle


placement to aspirate a small, possibly malignant lesion on a
patient’s liver. What is the code for this type of radiologic
guidance procedure?

A. 77001

B. 77002

C. 77003

D. 77011

15. A 56-year-old woman is undergoing a bilateral screening


mammogram. What is the correct code for this procedure where
there is computer-aided detection?

A. 77054

B. 77056

C. 77057

D. 77057, 77052
16. A patient has a breast lump that has been assessed by her
gynecologist, who scheduled her for a bilateral mammogram.
What is the correct code for this procedure, given that it is not a
screening mammogram, because there is a known problem?

A. 77055

B. 77056

C. 77057

D. 77058

17. A patient has to have a right-sided digital breast


tomosynthesis. What is the correct code for this procedure?

A. 77058

B. 77061

C. 77062

D. 77063
18. A patient is given an order for bone age studies. What is the
correct CPT code for this procedure?

A. 77072

B. 77073

C. 77074
D. 77076

19. What is the correct code for computed tomography for a


bone mineral density study of the axial skeleton?

A. 77077

B. 77078

C. 77080

D. 77086

20. A patient is scheduled for a vertebral fracture assessment


via dual-energy X-ray absorptiometry (DXA). What is the correct
code for this procedure?

A. 77078

B. 77080

C. 77086
D. 77084

21. A patient needs to have static liver imaging performed with


vascular flow. What is the correct CPT code for this procedure?
A. 78195

B. 78199

C. 78201

D. 78202

22. A young patient with recurrent urinary tract infections is


having a radiopharmaceutical voiding cystogram performed, also
known as a ureteral reflux study. What is the correct CPT code
for this procedure?

A. 78700

B. 78725

C. 78740

D. 78799

23. An athlete was seen in the emergency department for sudden


onset of testicular pain and swelling after playing basketball. A
testicular imaging with vascular flow study is ordered. What is the
correct code for this particular nuclear medicine procedure?

A. 78700

B. 78740

C. 78761

D. 78799
10,000 SERIES (20 QUESTIONS) – ANSWER KEY &
RATIONALE

1. Answer: C - The correct code for the procedure is 10040 (Acne


Surgery). This is the only code necessary, as it includes the
opening of acne comedones, cysts, and pustules. It also
includes the marsupialization of the acne cysts. Codes: 10060
and 10061 are used for the incision and drainage of unspecified
abscesses or cysts, rather than acne cysts.

2. Answer: A - The correct code for this procedure is


15781(Dermabrasion; Segmental, Face). Code 15780 is not
appropriate because it refers to the total face, which in this
case, did not apply because the dermatologist only performed
the procedure on certain parts of the patient’s face. Code 15786
is inappropriate because it refers to an abrasion of a lesion,
rather than dermabrasion. Code 15788 is also inappropriate
because it refers to a chemical peel, not dermabrasion.

3. Answer: A - The physician should code for this procedure using


codes: 11000 (Biopsy of Skin, Subcutaneous Tissue and/or
Mucous Membrane) and 11001 (X2) (Each Additional
Lesion).The physician biopsied three sites, therefore three
biopsy codes should be listed on the claim. Code 67810
(Incisional Biopsy of the Eyelid Skin including Lid Margin) is not
the correct code because the biopsy is for the skin of the eyelid,
instead of an incisional biopsy of the eyelid.

4. Answer: D - The appropriate codes for the service are: 11400,


for the excision of the 0.3 cm benign lesion, 11401 (X2) for the
excision of both 0.7 and 1.0 benign lesions, 11402 for the
excision of the 1.45 cm benign lesion, and 11602 (X2) for the
excision of both of the 1.5 cm malignant lesions. CPT coding
guidelines state that each malignant/benign lesion excised
needs to be reported separately; therefore each lesion removal
needs to be coded separately.

5. Answer: B - When the physician performs this procedure, she


should report code 15823 (Blepharoplasty, Upper Eyelid; with
Excessive Skin weighting Down Lid). Modifier -50 is
inappropriate because the procedure is only going to be
performed on the right side not bilaterally. Code 15820 is
inappropriate because it refers to a blepharoplasty of the lower
eyelid.

6. Answer: C - The appropriate code for this procedure is 11762-LT


(Reconstruction of the Nail Bed with Graft). The physician is
planning on using a graft of the nail bed from the left great toe,
therefore the code 11760-LT would not be correct because it
only applies to the unspecified repair of the nail bed not a repair
performed with a graft.

7. Answer: D - The correct codes for the wound repair performed by


the emergency department physician are 12004, for the simple
repair of the 4.6 cm and 4 cm lacerations of the left leg and right
arm; 12011 for the simple repair of the 2 cm laceration of the
cheek; and 12034 for the intermediate repair of the 4.5cm and
5cm lacerations of the left arm and chest. According to CPT
guidelines, when multiple wounds are repaired, the coder must
“add together the lengths of those in the same classification and
from all anatomic sites that are grouped together into the same
code descriptor.” The repair of the chest laceration is an
intermediate repair because it required the removal of particulate
matter (glass).
8. Answer: A - The correct codes for the procedure are 19300
(Mastectomy for Gynecomastia) and 19340 (Immediate
Insertion of Breast Prosthesis following Mastopexy, Mastectomy
or in Reconstruction). Both codes need to be appended with
modifier -50 to indicate that they were performed bilaterally.
Code 19303 is not appropriate, in this case, because the
mastectomy was performed due to gynecomastia.

9. Answer: C - 11765 is correct because it is the only code that is


specific to wedge excision of the skin of nail fold. Code 10060 is
incorrect because the ingrown toenail was not infected and did
not require drainage. Code 11760 is incorrect because there
was no repair done to the nail bed and 11750 is incorrect
because the wedge excision was performed to the skin, not the
nail itself, and the nail was not removed.

10. Answer: C - 11740 is correct because the subungual


hematoma was able to be evacuated by using a disposable
Bovie cautery pen in the office. Code 11720 is incorrect because
there was no debridement done of the nail. Code 11730 is
incorrect because there was no avulsion and 11750 is incorrect
because there was no excision of the nail for permanent
removal.

11. Answer: C - 10080 is correct because this was a


straightforward I&D of a pilonidal cyst. There were no
complications mentioned during the procedure. Code s10060
and 10061 are incorrect because they are used for I&Ds of
abscesses, but are not specific to pilonidal cysts. Code 10081 is
incorrect because it is for a complicated I&D of a pilonidal cyst
and the scenario specified that this was an uncomplicated
procedure.
12. Answer: A - 11771 is correct for an extensive procedure
because it involved excising the cyst and curetting it. Code
11772 is not correct because, while extensive, the excision was
not complicated. A complicated excision may include removal
down to the sacral fascia or using the surgical technique called
marsupialization. Code 11770 is incorrect because this pilonidal
cyst was more complicated and was not just a simple pilonidal
cyst. Code 10080 is incorrect because this was a complicated
pilonidal cyst and it required more than just an I&D because it
was also excised.

13. Answer: B - 11772 is correct for this extensively complicated


cyst excision requiring a z-plasty closure. Code 11770 is
incorrect because this was not a simple pilonidal cyst and it was
found to have extensions. Code 10081 is incorrect because,
although it is a complicated cyst, it is also being excised and
closed. Code 11771 is incorrect because while extensive, this
cyst was excised and repaired and also required a complicated
excision and closure.

14. Answer: B - 12036 is correct because this wound is


considered an intermediate repair due to the multiple layer
closure. Also, the wound size was 25 cm on the lower leg. Code
12035 is incorrect because, although it is the correct location,
the size is too small. Codes 12045 and 12046 are incorrect
because the location is not correct.
15. Answer: C - Code 12032 is being used as the primary
procedure code because this code is for 2.6-5 cm facial
lacerations and was considered an intermediate repair. The total
length of laceration for the arm and hand lacerations was 7 cm,
so 12002 is also correct. Because this was for a simple repair
that is less complicated than the other repair, it is listed as the
secondary procedure code. Modifier -59 is used because there
was more than one classification of wounds repaired on this
patient. Code 12002 is correct but must be listed as the second
code due to it being the less complicated repair. Code 12013 is
incorrect because this code is for simple repair of a wound 2.6
cm to 5.0 cm in length and the length of the simple repairs was a
total of 7 cm. This code is also specific to the face, ears, eyelids,
nose, lips and/or mucous membranes.

16. Answer: B - 11200 is correct because this code can be used


for the removal of up to and including 15 skin tags. Code 11101
is incorrect because these are skin tags that are being removed
and no biopsy was performed. Code 11201 is incorrect
because this is an “add-on” code for each additional 10 lesions
after the initial 15. 11300 is incorrect because there was no
mention of a shaving being performed.

17. Answer: B - 11301 is correct because this lesion was removed


by shaving. It is also 0.6 to 1.0 cm in diameter and is located on
the arm. Code 11300 is incorrect because this code is for 0.5
cm diameter lesions or smaller and this patient’s lesion is 0.8
cm. Code 11302 is incorrect because this code is used for
longer than 1.1 cm to 2 cm diameter lesions. Code 11305 is
incorrect because the lesion location is wrong for this code and
it is longer than this code indicates.

18. Answer: A - 17111 is correct because he has a total of 19


lesions that were frozen in the pediatrician’s office. This code is
for destruction of 15+ lesions. Code 17110 is incorrect because
this code is for up to 14 lesions such as warts, molluscum, or
milia. Codes 17000 and 17004 are both incorrect because they
are for actinic keratosis which is not what this little boy was
being treated for.

19. Answer: B - 19100 is correct because the needle biopsy was


completed without imaging guidance. Codes 19085 and 19086
are incorrect because these involve biopsy with breast
localization device placement. Code 19101 is incorrect
because this was done through needle biopsy and not an open
incisional biopsy.

20. Answer: C - 19355 is for correction of inverted nipples.


Code 19350 is incorrect because, although it is listed as
nipple/areola reconstruction, there is a specific code for
correction of inverted nipples. Code 19357 is incorrect
because there was no reconstruction with tissue expander.
Code 19499 is incorrect because there is a specific
procedure code for this issue.
20,000 SERIES (37 QUESTIONS) – ANSWER KEY &
RATIONALE

1. Answer: D - The difference between biopsy codes located in the


integumentary section and those found in the musculoskeletal
section is that the biopsy codes found in the integumentary
section are for biopsies of the skin and subcutaneous structures
whereas the biopsy codes found in the musculoskeletal section
are for deeper structures. Both of the sections include codes for
biopsies, but all of the codes in the musculoskeletal section are
for deeper structures underlying skin and subcutaneous
structures such as muscles and bones.

2. Answer: D - The correct code for this service is 21077


(Impression and Custom Preparation; Orbital Prosthesis).
According to the CPT guidelines, code 21077 is used when a
physician or other qualified health care professional designs
and prepares the prosthesis. The modifier -LT should be
appended to indicate that the prosthesis was created for the
left orbital socket.

3. Answer: B - The appropriate CPT code is 29804 (Arthroscopy,


Temporomandibular Joint, Surgical). The temporomandibular
joint is also referred to as the TMJ. Code 29800 is used for a
diagnostic arthroscopy with or without an synovial biopsy, but
the biopsy performed in this procedure was a soft tissue
biopsy. Code 21010 is only used if the procedure was an open
procedure. In this case, the procedure was not considered an
arthroscopic procedure.

4. Answer: C - The appropriate codes for this procedure are 22612


(Arthrodesis, Posterior or Posteriolateral Technique, Single
Level; Lumbar) and add-on code 22614 (X2) (Additional Level
of Arthrodesis). There were three levels of arthrodesis, L4, L3,
and L2, so each level needs to be reported, which can be
accomplished by using one 22612 and two 22614 codes. The
additional code 20937 needs to be included on the claim to
account for the morselized fascial bone graft performed at the
same time as the primary procedure.

5. Answer: B - The correct code for the procedure is23470


(Arthroplasty, Glenohumeral Joint; Hemiarthroplasty). Notes
under code 23195 (Resection, Humeral Head) state that the
appropriate way to report replacement of the humeral head
with implant is with code 23470 only.

6. Answer: B - The correct code for these injections is with 20553


(Injection(s); Single or Multiple Trigger Point(s), 3 or More
Muscle(s)).20553 is the only code that needs to be reported
because it allows for the multiple injections of each muscle and
it allows (for three or more muscles). There were four muscles
injected, two of which were injected multiple times.

7. Answer: A - The correct code for this service is 20690


(Application of a Uniplane (Pins or Wires in 1 Plane), Unilateral,
External Fixation System). Although multiple pins were placed,
the code description referred to multiple pins and/or wires. The
pins/wires were placed along the same plane, making the
fixation device a single plane, rather than multiple planes.

8. Answer: A - You should code for this fracture repair with code
25505 (Closed Treatment of Radial Shaft Fracture; with
Manipulation).The fracture was set with manipulation; therefore
the
code 25500 is not appropriate. Code 25500 does not include
manipulation of the fracture, while code 25505 does. Code
22515 is also inappropriate because the fracture repair was
closed and code 22515 is used for an open repair of the
fracture.

9. Answer: C - No code should be reported for this service.


According to CPT guidelines, the removal of a cast should only
be reported when the application of the cast was performed by
one physician and the removal of the cast by another physician.
The physician who removed the cast was the same that applied
the cast so no service should be reported.

10. Answer: B - You would use codes 20103 (Exploration of


Penetrating Wound; Extremity) and 12032 (Repair, Intermediate,
Wounds of Scalp, Axillae, Trunk and/or Extremities; 2.6 cm to
7.5 cm). According to CPT guidelines, the exploration of the
penetrating wound should be coded in addition to the wound
repair because the procedure required a “ligation or coagulation
of minor subcutaneous and/or muscular blood vessel(s)” The
intermediate repair should be coded separately because the
wound exploration code does not include closure of the wound
after dissection.

11. Answer: C - The correct CPT code for this procedure is 28293
(Correction, Hallux Valgus, with or without Sesamoidectomy;
Resection of Joint with Implant). Code 28290 is only used for a
simple resection without implant. Code 28292 is used for the
removal of the lateral end of the proximal phalanx and the
medial eminence of the metatarsal bone, but it does not include
the implant.

12. Answer: C - 21320 is correct for a closed treatment of the


nasal fracture with stabilization. Code 21310 is incorrect
because this was treatment without manipulation and code
21315 is incorrect because this was treatment without
stabilization. Code 21325 is incorrect because this code is for
the open treatment of a nasal fracture.

13. Answer: D - 21014 is correct for excision of a soft tissue


tumor of the scalp that is 3 cm. Codes 21011 and 21012 are
incorrect because they involve subcutaneous tumors of the
face or scalp. Code 21012 is incorrect because this tumor is
larger than 2 cm.

14. Answer: B - 21555 is correct for an excision of a


subcutaneous lesion smaller than 3 cm of the neck. Code
21550 is incorrect because this code is for a biopsy. Codes
21552 and 21556 are incorrect because of the size and type
of tumor present.

15. Answer: C - 21933 is correct for the excision of a tumor that


is 6 cm and is an intramuscular tumor. Code 21931 is incorrect
because although the size is correct, this is not a subcutaneous
tumor. Code 21932 is incorrect because the size is too small
and 21935 is incorrect because this did not involve a radical
resection.

16. Answer: B - 21930 is the correct code for a subcutaneous


soft tissue tumor excision of less than 3 cm. Code 21920 is
incorrect because this was a removal and not just a biopsy of
the tumor. Code 21931 is incorrect because of the size and
21932 is incorrect because this is not an intramuscular tumor.
17. Answer: B- 21925 is correct for a deep tissue biopsy of the
flank mass. Code 21920 is not correct because this wasn’t a
superficial tissue biopsy. Codes 21935 and 21936 are incorrect
because these codes both involve a radical resection of the
tumor.

18. Answer: C - 22558 is the only lumbar code listed for anterior
lumbar fusion and discectomy. Code 22554 is incorrect because
this is the code for cervical arthrodesis. Code 22556 is incorrect
because this code is for thoracic discectomy and 22585 is
incorrect because this is an “add-on” code for each additional
interspace procedure performed, but there was only one
performed on this patient.

19. Answer: B - 22511is correct because it is for a lumbosacral


percutaneous vertebroplasty of one vertebral body. Code 22510
is incorrect because is it for the cervicothoracic area. Code
22513 is incorrect because is more invasive, including cavity
creation and mechanical device. Code 22526 is incorrect
because it is the code for a percutaneous
intradiscalelectrothermalannuloplasty.

20. Answer: B - 22901 is correct because this tumor was 6 cm.


Code 22900 is incorrect because the size is larger than 5 cm.
Codes 22902 and 22903 are incorrect because they involve a
subcutaneous tumor, but this tumor is in the muscle of the
abdominal wall.

21. Answer: C - 22902 is correct for the excision of a 2.5 cm


subcutaneous tumor. Codes 22900 and 22901 are incorrect
because they are codes for intramuscular tumors. Code 22903
is incorrect because it is the code for a subcutaneous tumor
larger than 3 cm.
22. Answer: D - 22905 is correct for the removal of a 7.5 cm
sarcoma. Codes 22902 and 22903 are incorrect because they
involve subcutaneous tumors. Code 22904 is incorrect
because of the size of the tumor that is being resected.

23. Answer: C - 23615 is the correct code for open treatment


with internal fixation without prosthesis of the humerus. Codes
23600 and 23605 are incorrect because both refer to closed
treatment of the fracture. Code 23616 is incorrect because it
involves prosthetic replacement.

24. Answer: A - 23410 is the correct code for an acute injury that
involves an open procedure for repair of the rotator cuff. Code
23412 is incorrect because it is for a chronic rotator cuff repair,
but this was due to an acute injury. Code 23415 is incorrect
because this code is for a ligament release and code 23420 is
incorrect because it is for reconstruction of a complete shoulder
cuff avulsion.

25. Answer: D - 24655 is the correct code for closed treatment


of radial head fracture with manipulation. Code 24600 is
incorrect because it is for an elbow dislocation and 24620 is
incorrect because this is the code for closed treatment of an
ulna fracture with radial head dislocation. Code 24640 is
incorrect because it is the code for radial subluxation.
26. Answer: D - 24363 is the correct code for a total elbow
replacement, including distal humerus and proximal ulnar
prosthetic replacement. Code 24360 is incorrect because this
code is for an elbow arthroplasty with membrane. Code 24361
is incorrect because this code is for elbow arthroplasty with
distal humeral prosthesis and 24362 is incorrect because this
code includes ligament reconstruction.

27. Answer: A - 25111 is the correct code for initial ganglion cyst
excision. Code 25112 is incorrect because this code is for
recurrent cyst excisions. 25115 is incorrect because it is for
excision of the bursa and synovia of the wrist and 25130 is
incorrect because it is for the excision of bone cysts or tumor of
the carpal bones.

28. Answer: A - 26055 is the correct code for incision of the


tendon sheath to release the trigger finger. Code 26060 is
incorrect because this is for a percutaneous tenotomy. Code
26160 is incorrect because this is for an excision of a lesion of
the tendon sheath. 26170 is incorrect because this code is for
excision of the tendon.

29. Answer: C - 26587 is the correct code for reconstruction of


polydactylous digit including soft tissue and bone. Codes
26560 and 26561 are incorrect because they both involve the
repair of syndactyly. 26590 is incorrect because this is the code
for macrodactylia repair.

30. Answer: B - 26951 is the correct code because this was


an amputation of the finger with a specific phalanx noted.
Code 26910 is incorrect because this code is for amputation
of the metacarpal with finger or thumb amputation. Code
26952 is incorrect because this code involves local flaps
indicating a larger repair. 26989 is incorrect because this
code is for an unlisted procedure and the procedure above
has a specific code available.

31. Answer: B - 27130 is the correct code for arthroscopy of the


acetabular and proximal femoral prosthetic replacement also
known as “total hip replacement.” Code 27125 is incorrect
because this is the code for a partial hip replacement and 27134
is incorrect because it is for a revision of a total hip arthroscopy.
Code 27236 is incorrect because this code is for open treatment
of a femoral fracture with prosthetic replacement.

32. Answer: C - 27080 is correct for coccygectomy (removal of


coccyx). Code 27070 is incorrect because this code is for a
partial excision of the wing of the ilium. Codes 27200 and
27202 are incorrect because these are used for closed or open
treatments of coccygeal fractures not requiring removal of the
bone.

33. Answer: B - 29880 is the correct code because this football


player tore both his medial and lateral meniscus and the
procedure was meant to clean up both sides of his knee. Code
29879 is incorrect because this code is used for an arthroplasty
instead of an arthroscopy which is what this patient had done. .
Code 29881 is for one side or the other, not both medial and
lateral meniscus. Codes 29882 and 29883 are both codes for
repairing the meniscus, which in this football players case, was
not an option because both were completely torn.
34. Answer: D - 27562 is correct because it was a closed
treatment of patellar dislocation with conscious sedation, which
is considered anesthesia. Codes 27550 and 27552 are both
incorrect because they are referring to knee dislocations. Code
27560 is incorrect because this code is for a reduction without
anesthesia.

35. Answer: A - 27650 is the correct code for a primary repair of a


ruptured Achilles tendon. Code 27652 is not correct because
this code includes grafting. Code 27654 is incorrect because it is
for a secondary repair and 27658 is incorrect because this code
is for the repair of a flexor tendon and not the Achilles tendon.

36. Answer: D - 28825 is the correct code for an amputation


that was taken up to the proximal phalange. Code 22805 is
incorrect because this amputation goes up to the
transmetatarsal. Code 28810 is incorrect because it is an
amputation of the metatarsal and 28820 is incorrect because
it is up to the metatarsophalangeal joint involving the entire
toe instead of just a portion of the toe.

37. Answer: B - 28285 is the correct code and can be used for
any correction of hammertoe, including interphalangeal fusion
or partial/total phalangectomy. Code 28280 is incorrect
because this code is for syndactylization. Code 28286 is
incorrect because this code is for cock- up fifth toe with plastic
skin closure and 28290 is incorrect because this code is for the
correction of bunions.
30,000 SERIES (36 QUESTIONS) – ANSWER KEY &
RATIONALE

1. Answer: D - The difference between anterior nasal packing


versus posterior nasal packing procedure codes is that anterior
packing applies pressure and gauze to the front of the nose,
and posterior packing inserts a balloon into the back of the
nasal cavity. Posterior packing includes packing the nose with
gauze and applying pressure to the posterior aspect of the
nasal cavity. This can include inserting a balloon to the back of
the throat, which is inflated to block blood from draining into the
back of the throat.

2. Answer: A - The pediatrician should code for this service with


99214, for the office visit evaluation and management service.
This code should be appended with modifier -57 to indicate that
this procedure resulted in a decision for surgery, which was
performed later that day. Code 30310, for the removal of the
foreign body under general anesthesia, also needs to be
included to bill for the removal of the jelly bean from the nose.
Code 30320 is incorrect because it includes the removal of a
foreign body by lateral rhinotomy. Although the pediatrician
considered the lateral rhinotomy, it was not performed, and
therefore cannot be billed on the claim.

3. Answer: D - The correct code for this procedure is 31237


(Nasal/Sinus Endoscopy, Surgical; with Biopsy, Polypectomy or
Debridement). Code 31235 describes only a diagnostic
endoscopy, which would have been the case had the physician
not excised the polyps. The diagnostic endoscopy was
converted to a surgical endoscopy; therefore the only code that
needs to be included on the claim is the surgical endoscopy,
which in this case is 31237. Code 31231 only codes for a
diagnostic endoscopy, not surgical.
4. Answer: A - The appropriate code for the emergency department
procedures are 31500 for (Intubation, Endotracheal, Emergency
Procedure) and 94760, for the pulse oximetry. The pulse ox was
a multiple procedure, so it should be appended with modifier
-51. Code 31603 is not the appropriate code because it refers to
a tracheostomy. These two procedures are often confused in
coding but the code for an endotracheal intubation is within the
larynx section and the code for the tracheostomy is within the
trachea section.

5. Answer: C - No code should be used for the intubation


procedure. The correct code in this case is 31600
(Tracheostomy, Planned (Separate Procedure)).This code is
designated as a separate procedure, so it should only be
reported if it was the only procedure performed during a surgical
session. The primary procedure performed was the mandibular
excision and reconstruction; therefore the tracheostomy should
be bundled into the major procedure. The intubation should not
be reported separately.

6. Answer: D - You can find codes for cardiovascular services


within the surgery, radiology, and medicine chapters.
Cardiovascular codes are located throughout the CPT manual.
Invasive surgical procedures, such as heart transplants and
some cardiac catheterization codes are located within the
surgery chapter. Radiological guidance for cardiovascular
services such as: catheterizations are located within the
radiology chapter. Other cardiovascular codes are found
within the medicine chapter, which contains procedure codes
for non-invasive or less invasive services such as:
cardiovascular stress tests and programming of pacemakers.

7. Answer: B - The term used for the two-layered protective


membrane that surrounds the heart and the roots of the great
vessels is the pericardium. In order to perform some open
cardiac procedures, the surgeon must puncture or remove the
pericardium in order to access the heart. The pleural sac is the
membrane that surrounds each one of the lungs. The atrium is a
chamber of the heart and the aorta is the large vessel at the top
of the heart.

8. Answer: D-The atrioventricular node is located in the right atrium


on the septal wall. This node slows the impulses of the heart so
that the atria can fill with blood before it contracts. The bundle of
his is a group of muscle fibers in the center of the heart on the
septum. The bundle of his receives electrical impulses that
branch off to the right and left. The Purkinje fibers lie across the
surface of the ventricles and give the final signal for the
ventricles to contract.

9. Answer: A - A pacemaker or pacing cardioverter-defibrillator that


has pacing and sensing functions in three or more chambers of
the heart is considered a multiple lead system. Pacemaker or
cardioverter-defibrillator systems have electrical stimulation
leads that help shock the heart into rhythm. Depending on the
individual needs of the patient, some systems may have only
one lead, while others may have two, three, or more electrical
leads. Correct code selection depends on how many leads the
system has.

10. Answer: B - The correct code for this procedure is 33211


(Insertion or Replacement of Temporary Transvenous Dual
Chamber Pacing Electrodes). Codes 33216 and 33217 are in
appropriate because they are used for permanent pacemaker
systems. Code 33210 is also in appropriate because it is used
for a single chamber pacemaker.

11. Answer: C - The correct codes for the surgical procedure are:
33263 (Removal and Replacement of the Pulse Generator of a
Dual Lead System). Code 33243 needs to be included on the
claim to indicate the removal of the dual chamber electrodes.
The last code that needs to be included on the claim is 33249
(Replacement of the Dual Chamber Electrodes). The last two
codes for the service, 33243 and 33249, need to be appended
with modifier -51 to indicate that there were multiple procedures
performed on the same date of service.

12. Answer: C - The difference between arteriosclerosis and


arteriostenosis is that -sclerosis is the hardening of an artery
and -stenosis is the narrowing of an artery. Both are conditions
of the arteries, which may cause myocardial infarctions or other
vascular problems. Diagnosis of either condition can be
confirmed via cardiac catheterization procedures. These
procedures are located in the surgery and medicine chapters of
the CPT manual.

13. Answer: D - The correct code for this procedure is 35371


(Thromboendarterectomy, including Patch Graft, if Performed;
Common Femoral). Code 35301 is used for the same
procedure, but for the carotid artery. Code 35302 is also used
for the same procedure, but for the superficial femoral artery.
Code 35355 is used for the iliofemoral artery.
14. Answer: B - The correct code for the collection of the blood is
36415 (Collection of Venous Blood by Venipuncture). This code
is used when a nurse performs the venipuncture in the office, for
the purposes of collecting a blood sample. Code 36416 refers to
a capillary blood sample that is obtained from the tip of the
finger or heel of an infant. Codes 36406 and 36410 are used for
the collection of blood when it necessitates the skill of a
physician not a nurse.

15. Answer: A - The correct procedure code for the service is


38115 (Repair of Ruptured Spleen (Splenorrhaphy) with or
without Partial Splenectomy). It is not necessary to code for
38101 for the partial splenectomy, as the code 38115 includes
partial splenectomy as well as the splenorrhaphy. Code 38120
is for a laparoscopic surgical procedure, which is inappropriate.

16. Answer: D - The appropriate codes for the service are


38200 (Injection Procedure for Splenoportography) and 75810
(Splenoportography, Radiological Supervision and
Interpretation). The modifier -26 does not need to be included
on the claim because the code for 75810already includes the
supervision and interpretation component.

17. Answer: B - The correct code for the procedure performed by


the physician is 38220 (Bone Marrow; Aspiration Only). This is
the appropriate code because the physician was not harvesting
the marrow from the donor. Harvesting codes, such as 38230
and 38232 are only used when the bone marrow is harvested in
large amounts in order to transplant the marrow into the patient.
In this case, the physician was only aspirating a small sample to
test (to see if the donor and the patient were matches).

18. Answer: C - The correct code for the procedure is 38525


(Biopsy or Excision of Lymph Node(s); Open, Deep Axillary
Nodes). This code does not need to be reported with more
than one unit because the code description allows for the
excision of multiple nodes. Code 38555 is also incorrect
because this code is used for the excision of a cystic hygroma,
not a deep axillary lymph node.

19. Answer: D - The correct code for the procedure is 38792


(Injection Procedure; Radioactive Tracer for Identification of
Sentinel Node) which is the primary procedure. Code 75801
(Lymphangiography, Extremity Only, Bilateral, Radiological
Supervision and Interpretation) also needs to be reported to
indicate the radiological guidance for the lymphangiography
procedure. The procedure is not indicated as a bilateral
procedure, which would exclude code 75803, which is a bilateral
procedure and modifier -50, which indicates that the procedure
was bilateral.

20. Answer: A - The correct CPT code is 39561 (Resection,


Diaphragm; with Complex Repair).The physician utilized an
autogenous free muscle flap, therefore the repair is considered
complex. If the physician had only sutured the wound, then the
appropriate code would have been 39560 (Resection,
Diaphragm; with Simple Repair).

21. Answer: C - Mediastinotomy codes are divided based on


the approach the mediastinotomy. There are only two
mediastinotomy codes, 39000 and 39010. Mediastinotomy is
divided into
code 39000 (Mediastinotomy with Exploration, Drainage,
Removal of Foreign Body, or Biopsy; Cervical Approach) and
39010 (Transthoracic Approach (including either a
transthoracic or a median sternotomy).

22. Answer: A - The correct code for the mediastinotomy is


39000 (Mediastinotomy with Exploration, Drainage, Removal
of Foreign Body, or Biopsy; Cervical Approach). There are only
two mediastinotomy codes, 39000 and 39010. Code 39010 is
for a mediastinotomy via transthoracic approach, not a cervical
approach.

23. Answer: B - The correct code for this service is 39503 (Repair,
Neonatal Diaphragmatic Hernia, with or without Chest Tube
Insertion and with or without Creation of Ventral Hernia). Code
39540 (Repair, Diaphragmatic Hernia (Other Than Neonatal),
Traumatic; Acute) is inappropriate because the patient is only 15
days old, making her a neonate. Patients are considered
neonates until 28 days of age.

24. Answer: B - The correct code for this procedure is 39220


(Resection of Mediastinal Tumor). There are only two codes for
the removal or resection of a mediastinal mass. Code 39200 is
for the removal of a mediastinal cyst, whereas code 39220 is
for the removal of a tumor. The postoperative diagnosis states
that the mass is a tumor, therefore code 39220 is the most
appropriate code for this service.

25. Answer: A - The correct code for this procedure is 39545


(Imbrication of Diaphragm for Eventration, Transthoracic or
Transabdominal, Paralytic or Nonparalytic). An abnormal
elevation of the dome of the diaphragm is otherwise known as
an eventration. This condition is corrected by imbrication, which
essentially stacks the muscles of the diaphragm on top of each
other to reduce the abnormal elevation.
26. Answer: A - Code 30901is the correct code for a one-sided
simple cautery of the nares performed in the office. Code
30901 with modifier -50 is incorrect because it did not involve
both nares, so no modifier is needed. Codes 30903 and 30903,
-50 are incorrect because these are for a complex procedure.

27. Answer: C - 31239 is the correct code for surgical nasal


endoscopy with dacryocystorhinostomy. Code 31237 is incorrect
because an additional surgery was performed. Code 31238 is
incorrect because there was no mention of a nasal hemorrhage
and 31240 is incorrect because there was no resection
performed.

28. Answer: D - 31295 is the correct code because this was a


transnasal endoscopy with balloon dilation of the frontal sinus.
Code 31233 is incorrect because this is for a maxillary
sinusoscopy and 31256 is incorrect because this is with
maxillary antrostomy. Code 31276 is incorrect because this is
frontal sinus exploration with or without removal of tissue.
29. Answer: C - 31511 is the correct code for laryngoscopy with
foreign body removal. Codes 31505, 31510, and 31512 are all
codes for laryngoscopies, but none of these involve a foreign
body removal.

30. Answer: C - 31580 is the correct code for a laryngoplasty


performed for a laryngeal web. Code 31535 is incorrect because
it is a direct laryngoscopy with biopsy. Code 31575 is incorrect
because it is for a flexible fiberoptic laryngoscopy and 31588 is
incorrect because, while it is for a laryngoplasty, it is “not
otherwise specified.”

31. Answer: B - 31500 is the correct code for an emergency


endotracheal intubation of a patient. Code 31420 is incorrect
because there was no epiglottidectomy performed. Code 31502
is incorrect because there was no tracheotomy tube change
and 31505 is incorrect because there was no laryngoscopy
performed.

32. Answer: D - 31625 is the correct CPT code for a flexible


bronchoscopy with bronchial biopsies. Code 31622 is incorrect
because it was not just a simple bronchoscopy. Code 31623 is
incorrect because there were no brushings taken and 31624 is
incorrect because there was no alveolar lavage performed.

33. Answer: A - Code 32035 is for a thoracostomy with rib


resection for empyema. Code 32036 is incorrect because there
was no flap drainage performed. Codes 32096 and 32097 are
incorrect because they are for thoracotomies with biopsies for
lung infiltrates, nodules or masses.

34. Answer: C - 32482 is the correct code for a pneumonectomy


of two lobes of the lung, which is called a bilobectomy. Code
32440 is for a pneumonectomy and 32480 is for a single lobe
or a lobectomy. Code 32484 is for a segmentectomy where
just a single segment of a lobe is removed.

35. Answer: B - 32607 is the correct code for a unilateral infiltrate


and requires a thoracoscopy with wedge biopsy. Code 32601 is
incorrect because this is for a thoracoscopy without biopsies.
Code 32608 is incorrect because this includes a biopsy of
masses or nodules. Code 32650 is incorrect because this code
is for a surgical thoracoscopy with pleurodesis.

36. Answer: A - 38100 is the correct code for a total


splenectomy. Code 38101 is incorrect because this was a total,
and not a partial, splenectomy. Code 38102 is incorrect
because this is an add- on code. Code 38115 is incorrect
because there was no repair done to the spleen during this
procedure.
40,000 SERIES (44 QUESTIONS) – ANSWER KEY
& RATIONALE

1. Answer: A - Code 40490 (Biopsy of Lip) is the correct code for


this procedure. Code 40500 (Vermilionectomy (Lip Shave), with
Mucosal Advancement) is not correct because it is a more
extensive procedure than what was performed at the visit, which
was just a biopsy. Code 11100 (Biopsy of Skin) is also not
correct because the code 40490 is used for the lip, which is
more specific.

2. Answer: A - The correct code for the plastic repair of cleft lip with
nasal deformity when utilizing an Abbe-Estlander cross-lip
pedicle flap is 40527 (Excision of Lip; Full Thickness,
Reconstruction with Cross-Lip Flap (Abbe-Estlander)). In
parenthesis under code 40761 for plastic repair of cleft lip/nasal
deformity, it notes that if using the Abbe-Estlander type repair,
use the 40527 code. Code 42200 is only used for the repair of a
cleft palate.

3. Answer: A - The correct CPT code for the physician to report is


42400 (Biopsy of Salivary Gland; Needle). Code 42405 is not
correct because it allows for an incisional biopsy of the salivary
gland, whereas this procedure was done via needle. It is also
not correct to code for the pathology service using codes 88172
or 88173 because the physician sent the sample out to the lab.
The lab should code for the pathology service not the physician.

4. Answer: B - You should code this procedure using code 42961


(Control Oropharyngeal Hemorrhage, Primary or Secondary
(e.g. Post-Tonsillectomy); Complicated, Requiring
Hospitalization). The notes indicate that the patient was
admitted to the hospital for observation, so the code 42960
(Simple Control) is not appropriate. In addition, code 42962
refers to a secondary surgical intervention, but in this case, no
other surgical interventions were noted.

5. Answer: D - The correct code for this procedure is 43337


(Repair, Paraesophageal Hiatal Hernia, via Thoracoabdominal
Incision, Except Neonatal; with Implantation of Mesh or Other
Prosthesis). Code 39531 was deleted and replaced with code
43337, as indicated by the notes within the mediastinum and
diaphragm section. Code 43336 is inappropriate because it
does not take into account the implantation of mesh, while code
43337 does.

6. Answer: B - The correct code for this procedure is 43328


(Esophagogastric Fundoplasty; Thoracotomy). The code 43327
is correct because it is used for the same procedure, but it refers
to a laparotomy and the note states that the procedure was
performed using a lower thoracic incision. Code 43325 is also
correct because it includes a fundic patch. In this case, the notes
did mention a fundic patch.

7. Answer: A-The correct CPT code is 43753 (Gastric Intubation


and Aspiration(s) Therapeutic, Necessitating Physician’s
Skill, including Lavage if Performed). Code 43752 is only for
the placement of a naso- or oro-gastric tube and does not
include lavage and evacuation of the stomach contents.
Code 43754 includes the placement of the gastric tube and
aspiration of stomach contents, but for diagnostic purposes
not for treatment of a poisoning.
8. Answer: D - The correct code for this procedure is 43865
(Revision of Gastrojejunal Anastomosis (Gastrojejunostomy)
with Reconstruction, With or Without Gastrectomy or Intestine
Resection; With Vagotomy). Code 43860 is used for the same
procedure, but without the vagotomy. In this case, a vagotomy
was performed, making code 43865 the more appropriate code.
Codes 43850 and 43855 are used for a gastroduodenostomy
not a gastrojejunostomy.

9. Answer: B - Codes for the pancreas are listed in the digestive


system section of the CPT manual (40490 to 49999). Although
there is a subheading within the endocrine section titled
“Parathyroid, Thymus, Adrenal Glands, Pancreas, and Carotid
Body,” there are no codes for the pancreas within that section.
There is subsection, code range (48000 to 48999), within the
digestive system that is designed specifically for the pancreas.

10. Answer: A - 40650 is the only code specific to the vermilion


border repair of the lip. Code 40652 is incorrect because it is for
a lip repair, up to half of the vertical height. Code 40654 is
incorrect because the repair did not involve more than one half
of the vertical height and was not listed as complex. Code 40700
is incorrect because there was no mention of cleft lip repair.

11. Answer: B - Code 40819 Code 40820 is the code for excision
or destruction of the frenum. Code 40806 is the code for
incision of the labial frenum. Code 40820 is the code for
destruction of lesions or scars in the vestibule of the mouth and
40899 is the code for unlisted procedures of the vestibule of the
mouth.

12. Answer: B - 41251 is the correct code because this is for a


laceration less than 2.5 cm and involves the posterior tongue.
Code 41250 is incorrect because it involves the anterior tongue.
Code 41252 is incorrect because the laceration is not more than
2.6 cm or complex. Code 41599 is incorrect because there is a
listed code that works for this procedure.

13. Answer: A - 41100 code is the only code for biopsy of the
tongue involving the anterior two- thirds of the tongue. Code
41105 is incorrect because it is for biopsy of the posterior one-
third of the tongue. Code 41108 is incorrect because this is for
a biopsy of the floor of the mouth. Code 41112 is incorrect
because this is for excision of the lesion with closure of the
anterior two-thirds of the tongue and this procedure was only
for a biopsy.

14. Answer: C - 42100 is the correct code for uvula biopsy.


Code 42000 is incorrect because there was no drainage of an
abscess performed. Code 42104 is incorrect because there
was no excision of a lesion performed. Code 42140 is
incorrect because there was no uvulectomy performed.

15. Answer: A - 42200 is correct because this code is for a


palatoplasty of the soft and/or hard palates only. Code 42205
is incorrect because there was no closure of the alveolar ridge.
Code 42210 is incorrect because there was no bone graft of
the alveolar ridge. Code 42215 is incorrect because there was
no major revision performed.
16. Answer: A - 42000 is the correct code for drainage of an
abscess of the palate or uvula. Code 42100 is incorrect
because this code is for a biopsy of the palate or uvula. Code
42104 is incorrect because this is for excision of a lesion and
42140 is incorrect because this code is for auvulectomy.

17. Answer: B - 42305 is the correct code for complicated


abscess drainage of the parotid gland. Code 42300 is incorrect
because this is for a simple drainage of parotid gland abscess.
Code 42310 is incorrect because this is for an intraoral drainage
of an abscess in the submaxillary or sublingual glands. Code
42320 is incorrect because this is for the external drainage of an
abscess in the submaxillary gland.

18. Answer: C - 42408 is the correct code for excision of a


sublingual salivary cyst. Code 42400 is incorrect because this
code is for a needle biopsy of the salivary gland. Code 42405 is
incorrect because this is for an incisional biopsy of the salivary
gland. Code 42409 is incorrect because this is for
marsupialization for the sublingual salivary cyst and not for the
removal of this cyst.

19. Answer: C - 42820 is the correct code for a tonsillectomy and


adenoidectomy for a patient younger than age 12. Code 42825
is incorrect because this code is just for a tonsillectomy. Code
42830 is incorrect because this code is just for an
adenoidectomy. Code 42821 is incorrect because this is for a
tonsillectomy and adenoidectomy for a patient age 12 or over.

20. Answer: D - 42870 is correct because this code is for the


excision or destruction of the lingual tonsil. Code 42825 is
incorrect because this code is for tonsillectomy for patient under
the age of
12. Code 42826 is incorrect because this code is for
tonsillectomy for a patient age 12 or over. Code 42860 is
incorrect because this code is for excision of tonsil tags.

21. Answer: B - 43239 is correct because this procedure is EGD


with single or multiple biopsies and code 43250 is correct
because this is EGD with removal of polyps with hot biopsy
forceps. Code 43239 is correct but is incomplete because there
were two procedures performed during the course of this EGD
and a polyp was also removed. Code 43251 is incorrect
because the polyp wasn’t removed by snare technique, it was
removed by hot biopsy forceps. Code 43255 is incorrect
because there was no mention of bleeding having to be
controlled during this EGD.

22. Answer: C - 43770 is the correct code for surgical


laparoscopy for gastric restrictive procedure with placement of
adjustable gastric restrictive device, such as a gastric band.
Code 43644 is incorrect because this is for laparoscopy with
gastric bypass and Roux-en-Y gastroenterostomy. Code 43645
is incorrect because this is for a surgical laparoscopy with
gastric bypass and small intestine reconstruction. Code 43774
is incorrect because this is for the removal of the adjustable
gastric restrictive device and subcutaneous port components.

23. Answer: D - 44144 is correct because this is for partial


colectomy with resection with colostomy or ileostomy and
creation of mucofistula. Code 44140 is incorrect because this is
for partial colectomy with anastomosis. Code 44141 is incorrect
because this is for partial colectomy with
skin level cecostomy or colostomy. Code 44143 is incorrect
because this is for partial colectomy with end colostomy and
closure of distal segment (Hartmann’s procedure).

24. Answer: B - 44389 is the code for colonoscopy through stoma


with single or multiple biopsies. Code 44388 is incorrect
because this is for colonoscopy through stoma with collection of
specimens by brushing or washing. Code 44391 is incorrect
because this is for colonoscopy through stoma with control of
bleeding. Code 44392 is incorrect because this is for
colonoscopy through stoma with removal of tumor, polyps, or
lesions with hot biopsy forceps.

25. Answer: C - 44960 is correct because this is an open


appendectomy for ruptured appendix with abscess or
generalized peritonitis. Code 44950 is incorrect because this is
for an open appendectomy without abscess or peritonitis. Code
44955 is an “add-on” code and is incorrect because this is for
an appendectomy when done at the time of another major
procedure. Code 44970 is incorrect because this is a
laparoscopic appendectomy.

26. Answer: C - 44970 is the correct code because this is for


surgical laparoscopic appendectomy. Code 44950 is incorrect
because this is for open appendectomy. Code 44960 is
incorrect because this is for an open appendectomy with
ruptured appendix or generalized peritonitis. Code 44979 is
incorrect because this is for an unlisted laparoscopic
procedure of the appendix and appendectomy has a specific
code that is listed.

27. Answer: A - 44900 is correct because this is for an open


incision and drainage of an appendiceal abscess. Code 44950
is incorrect because this is for an open appendectomy. Code
44960 is incorrect because this is for an open appendectomy
with ruptured appendix with abscess or generalized peritonitis.
Code 44970 is incorrect because this code is for a surgical
laparoscopic appendectomy.

28. Answer: D -45385 is the correct code for a colonoscopy with


polyp removal by snare technique. Code 45378 is incorrect
because this is for a diagnostic colonoscopy with brushings or
washings. Code 45380 is incorrect because this is for
colonoscopy with single or multiple biopsies. Code 45384 is
incorrect because this is for colonoscopy with removal of polyps
by hot biopsy forceps.

29. Answer: B - 45805 is correct because this was a closure of


rectovesical fistula with colostomy. Code 45800 is incorrect
because this is closure of rectovesical fistula without colostomy.
Code 45820 is incorrect because this is closure of rectourethral
fistula without colostomy. Code 45825 is incorrect because this
is the code for closure of rectourethral fistula with colostomy.

30. Answer: B - 45114 is correct because this is the code for a


partial proctectomy with anastomosis using both the abdominal
and transsacral approach. Code 45113 is incorrect because this
is a partial proctectomy with rectal mucosectomy, ileoanal
anastomosis, creation of ileal reservoir with or without loop
ileostomy. Code 45116 is incorrect because this is a partial
proctectomy with anastomosis using only the transsacral
approach. Code 45123 is incorrect because this is a partial
proctectomy without anastomosis using the perineal approach.
31. Answer: A - 46221 is the correct code for internal
hemorrhoidectomy by rubber band ligation. Code 46945 is
internal hemorrhoidectomy by ligation other than rubber band
for single hemorrhoid column/group. Code 46220 is incorrect
because this is a code for excision of single external anal
papilla or tag. Code 46255 is incorrect because this is for
internal and external hemorrhoidectomy of a single
column/group.

32. Answer: A - 46750 is correct because this is the code for an


anal sphincteroplasty for incontinence or prolapse in an adult.
Code 46751 is incorrect because this is the code for this
procedure in a child. Code 46760 is incorrect because this is
an anal sphincteroplasty for incontinence in an adult, including
a muscle transplant. Code 46761 is incorrect because this is
the code for an anal sphincteroplasty for incontinence in an
adult with a levator muscle imbrication.

33. Answer: C - 46050 is the correct code for incision and


drainage of a superficial perianal abscess. Code 46040 is
incorrect because this is the code for the incision and drainage
of ischiorectal and/or perirectal abscess. Code 46045 is
incorrect because this is the code for incision and drainage of a
transanal intramural, intramuscular, or submucosal abscess
under anesthesia.

34. Answer: A - 47000 is the correct code for a percutaneous


needle biopsy of the liver. Code 47001 is incorrect because this
is an “add-on” code when the needle biopsy is done at the
same time as another procedure. Code 47100 is incorrect
because this is the code for a wedge biopsy of the liver. Code
47300 is incorrect because this is the code for marsupialization
of a cyst or abscess of the liver.
35. Answer: D - 47142 is correct because this code is for a donor
hepatectomy of the total right lobectomy. Code 47120 is
incorrect because this is the code for a hepatectomy that
involved a partial lobectomy. Code 47130 is incorrect because
this code is for the excision of a total right hepatic lobectomy not
involving a donor. Code 47140 is incorrect because, while this is
the code for a donor hepatectomy, it is for the left lateral
segments only, not for the entire right lobe.

36. Answer B - 47360 is the correct code for management of liver


hemorrhage involving complex suture of liver injury with or
without hepatic artery ligation. Code 47350 is incorrect because
this is the code for the simple suture of liver wound or injury to
manage a liver hemorrhage. Code 47361 is incorrect because
this code is for the management of liver hemorrhage with
exploration of the wound including extensive debridement,
coagulation and/or suture with or without packing of the liver.
Code 47362 is incorrect because this code is for re-exploration
of the hepatic wound for removal of the liver packing.

37. Answer: C - 47562 is correct because this is a surgical


laparoscopy with cholecystectomy. Codes 47560 and 47561 are
incorrect because there was no guided transhepatic
cholangiography with or without biopsy performed. Code 47563
is incorrect because, while there was a cholecystectomy
performed, there was no cholangiography performed.
38. Answer: C - 47715 is the correct code for excision of a
choledochal cyst. Code 47630 is incorrect because this code is
used for biliary duct stone extraction. Code 47711 is incorrect
because it is the code for the excision of a bile duct tumor. Code
47999 is incorrect because the removal of choledochal cyst has
a specific code, so it would not fall under the code for unlisted
procedure of the biliary tract.

39. Answer: C - 47554 is correct because this is the code for


percutaneous biliary endoscopy with removal of calculus/calculi.
Code 47552 is incorrect because this is the code for
percutaneous biliary endoscopy with collection of specimens by
brushing and/or washing. Code 47553 is incorrect because this
is a percutaneous biliary endoscopy with single or multiple
biopsies. Code 47555 is incorrect because this is percutaneous
biliary endoscopy with dilation of biliary duct stricture without
stent placement.

40. Answer: A - 48000 is the correct code for peripancreatic


drain placement for pancreatitis. Code 48001 is incorrect
because there was no mention of cholecystostomy,
gastrostomy or jejunostomy. Code 48020 is incorrect because
there was no removal of pancreatic calculus. Code 48510 is
incorrect because there was external drainage of pancreatic
pseudocyst.

41. Answer: B - 48120 is the correct code for excision of lesions


of the pancreas, including cysts or adenomas. Code 48102 is
incorrect because this is the code for a percutaneous needle
biopsy of the pancreas. Code 48148 is incorrect because this
code is for the excision of the ampulla of Vater. Code 48155 is
incorrect because this is the code for a total pancreatectomy.

42. Answer: B - 49652 is the correct code for laparoscopic


umbilical hernia repair with mesh insertion for a reducible
hernia. Code 49650 is incorrect because this surgery is for an
inguinal hernia. Code 49653 is incorrect because this is a
laparoscopic umbilical hernia repair that is incarcerated or
strangulated. Code 49654 is incorrect because this is for a
laparoscopic repair of an incisional hernia with mesh insertion.

43. Answer: C - 49501 is the correct code for an incarcerated


inguinal hernia for ages six months to younger than five years of
age. Codes 49492, 49496 and 49507 are all the codes for
incarcerated inguinal hernia repair. However, these codes are
different due to the ages of the patients the procedures are
performed on. Code 49492 is for preterm infants (younger than
37 weeks at birth) performed from birth up to 50 weeks
postconception age. Code 49496 is the code for full- term infants
younger than six months or preterm infants older than 50 weeks
postconception age and younger than six months. Code 49507
is the code for ages five years of age or older.

44. Answer: B - 49082 is the correct code for abdominal


paracentesis (diagnostic or therapeutic) without imaging
guidance. Code 49060 is incorrect because this is the code for
open drainage of a retroperitoneal abscess. Code 49083 is
incorrect because this code is for an abdominal paracentesis
with imaging guidance. Code 49084 is incorrect because this
code is for peritoneal lavage including imaging guidance when
needed.
50,000 SERIES (80 QUESTIONS) – ANSWER KEY &
RATIONALE

1. Answer: C - The correct code for this procedure is 50561(Renal


Endoscopy through Established Nephrostomy or Pyelostomy,
with or without Irrigation, Instillation, or Ureteropyelography,
Exclusive of Radiologic Service; with Removal or Foreign Body
or Calculus). Codes 50060 and 50075 are incorrect because
they are used for the removal of the calculus via an open
incision, or nephrolithotomy. Code 50580 is incorrect because it
is used for an endoscopic removal of a calculus through a
nephrotomy; while in this case, access for the procedure was
gained through a nephrostomy.

2. Answer: A - The correct code for this transplant is 50360 (Renal


Allotransplantation, Implantation of Graft; without Recipient
Nephrectomy) and modifier -50. Modifier -50 must be included
on the claim to indicate that the service was bilateral (it was
performed dually). Code 50365 is incorrect because it is used
for a renal transplantation with nephrectomy, meaning that the
patient’s kidneys were removed before the transplants were
implanted. This was not the case.

3. Answer: B - The appropriate CPT code is 52310


(Cystourethroscopy, with Removal of Foreign Body, Calculus, or
Ureteral Stent From Urethra or Bladder; Simple). Code 52000,
for cystourethroscopy should not be reported separately as the
term “separate procedure” is within the code description. In other
words, you do not need to report it unless it was the only
procedure performed. In this case, it is included within the code
for the primary procedure, 52310. Code 52315 is also incorrect
because there is no mention of the procedure being
complicated.
4. Answer: D - Code 51992 (Laparoscopy, Surgical; Sling
Operation for Stress Incontinence) is the appropriate code for
this procedure. Codes 57288 and 57287 are used for open
procedures and although code 51990 is also used for
laparoscopic stress incontinence, this code refers to urethral
suspension not a sling.

5. Answer: C - The correct code for this service is 51597 (Pelvic


Exenteration, Complete, for Vesical, Prostatic, or Urethral
Malignancy, with Removal of Bladder and Ureteral
Transplantations, with or without Hysterectomy and/or
Abdominoperineal Resection of Rectum and Colon and
Colostomy, or any Combination Thereof). Code 58285, for
radical hysterectomy, should not be reported separately, as code
51597 already includes hysterectomy. Code 58240 is incorrect
because it is used for a pelvic exenteration performed due to a
gynecologic malignancy. In this case, the patient had a urethral
malignancy, not gynecologic malignancy.

6. Answer: B - You should code for this procedure with 51798


(Measurement of Post-Voiding Residual Urine and/or Bladder
Capacity by Ultrasound, Non-Imaging). Modifier -50 should not
be used for a bilateral procedure. In this case, the procedure
cannot be done bilaterally.

7. Answer: A - The correct CPT code is 52235 (Cystourethroscopy,


with Fulguration and/or Resection of; MEDIUM Bladder
Tumor(s) (2.0 to 5.0 cm)). The procedure description states that
the bladder tumor was 2.3 cm, so it fits within the medium size
category for bladder tumors. Code 52234 is used for small (0.5
up to 2.0 cm) tumors and code 52240 is used for large (over
5.0 cm) tumors.

8. Answer: D - The correct code for the instillation procedure is


51720 (Bladder Instillation of Anticarcinogenic Agent (Including
Retention Time)). In this procedure, an anticarcinogenic agent
for the treatment of cancer was instilled into the bladder and
held for a certain amount of time in order to kill cancer cells. This
is a common treatment for bladder cancer.

9. Answer: C - The CPT codes that should be reported are 52010


(Cystourethroscopy, with Ejaculatory Duct Catheterization, with
or without Irrigation, Instillation, or Duct Radiography, Exclusive
of Radiologic Service) and 74440 (Vasography, Vesiculography,
or Epidemiography, Radiological Supervision and Interpretation)
Duct radiography was performed, therefore the radiological S&I
also needs to be reported. In parenthesis under code 52010, it
directs the coder to code 74440, the appropriate radiological S&I
code.

10. Answer: C - The appropriate code for this procedure is


54015 (Incision and Drainage of Penis, Deep). Codes 10061
and 10081 are incorrect because they are integumentary codes
that refer to the incision and drainage of deep abscesses of the
skin. In this case, the abscess is not in the skin of the penis, it
is in the deep tissue of the penis, therefore integumentary
codes are inappropriate.

11. Answer: C - The correct code for the circumcision is 54150


(Circumcision, Using Clamp or Other Device with Regional
Dorsal penile or Ring Block). Code 54160 is not correct
because it is used for a circumcision without using a clamp or
other device. You should not use modifier -47 (Anesthesia by
Physician) because the anesthesia was included in the
procedure description. Modifier -47 guidelines state that the
modifier is not to be used with local anesthesia and a penile
ring block is a local anesthetic.

12. Answer: D - The correct code for this procedure is with code
54640 (Orchiopexy, Inguinal Approach, with or without Hernia
Repair). Codes 54690 and 54692 are orchiopexy and
orchiectomy codes, but are not appropriate because the
procedure was not performed laparoscopically. Code 54650 is
also incorrect because it is used for an orchiopexy performed via
an abdominal approach not an inguinal approach.

13. Answer: A - The correct code for this procedure is 55250


(Vasectomy, Unilateral or Bilateral (Separate Procedure),
Including Postoperative Semen Examination(s)). This code
does not need to be appended with modifier -50. Modifier
-50indicates that the procedure was performed bilaterally. In
other words, code 55250 indicates that the procedure
performed either unilaterally or bilaterally. Code 55200 is also
incorrect because it is used for a vasotomy not the excision of
the vas deferens (a vasectomy).

14. Answer: B - The correct code for this surgical procedure is


52601 (Transurethral Electrosurgical Resection of the Prostate
Including Control of Postoperative Bleeding, Complete). The
procedure TURP stands for transurethral resection of the
prostate, which is the procedure described in the procedural
note. Code 52500 is only used for the resection of a bladder
neck not the prostate. Code 52630 is used for the regrowth of
prostatic tissue and modifier-58 is inappropriate because it is
used for the primary procedure.

15. Answer: C - Code 56440 would represent the physician’s


work. Code 56440 (Marsupialization of Bartholin’s Gland Cyst) is
the correct code because the physician did not simply perform
an incision and drainage (which would be reportable with code
56420), but sutured the sides of the cyst to leave it open for
drainage. This describes a marsupialization procedure.

16. Answer: D - The correct code for the procedure would be


57425 (Laparoscopy, Surgical, Colpopexy (Suspension of
Vaginal Apex). The other codes are also used for a colpopexy,
but they are used for procedures performed via an open
approach. This procedure states that it was performed with a
laparoscope; therefore the laparoscopic procedure is the most
appropriate code.

17. Answer: C-The OB should code for this surgical procedure


with code 59135 (Surgical Treatment of Ectopic Pregnancy;
Interstitial, Uterine Pregnancy Requiring Total Hysterectomy).
Code 58150 is inappropriate because it is used for a total
hysterectomy performed, for other reasons, not for an ectopic
pregnancy. Code 59120 is also inappropriate because it is used
for the removal of an ectopic pregnancy located on the fallopian
tube or ovary, not in the uterus. Code 59136 codes for the
removal of an interstitial uterine pregnancy, but it does not take
into account the resection of the uterus.

18. Answer: A - The physician in Atlanta should code for his


services with code 59426 (Antepartum Care Only, 7 or More
Visits).The patient only saw her Atlanta physician 9 times,
therefore the Atlanta physician can only code for the antepartum
care that he provided. Code 59425 is only 4- 6 antepartum
visits. Codes 59410 and 59430 are also incorrect because they
refer to postpartum care, which in this case, was provided by
the physician in Albuquerque.

19. Answer: C - The correct code is 59001, for the therapeutic


amniocentesis with amniotic fluid reduction. Code 76946, for
the ultrasonic guidance, would not have to be reported, as it is
included as part of code 59001. You should only use 76946 for
a diagnostic amniocentesis, which in this case, is incorrect. The
withdrawal of the amniotic fluid is a therapeutic procedure.

20. Answer: D-The OB should report code 59610 (Routine


Obstetric Care Including Antepartum Care, Vaginal Delivery and
Postpartum Care, after Previous Cesarean Delivery). Even
though the patient’s first child was born vaginally, her second
child was delivered by a cesarean section. This type of delivery
is referred to as a vaginal birth after cesarean (VBAC).

21. Answer: D - The OB should code for the procedure with


codes 59400, for the vaginal delivery. No additional modifier is
necessary for this code. Code 59412 needs to be reported in
addition to the delivery code, to indicate the additional service
that was performed, in this case the external cephalic version
procedure with tocolysis. Guidelines in the CPT manual after
code
59412 indicate that it should be reported in addition to the code
for the delivery. Modifier -51 and code 59412 should also be
included on the claim to indicate that there were multiple
procedures performed on the same date of service.

22. Answer: B - The correct procedure codes are: 60300


(Aspiration and/or Injection, Thyroid Cyst) and 76942
(Ultrasonic Guidance for Needle Placement (e.g. Biopsy,
Aspiration, Injection, Localization Device) Imaging Supervision
and Interpretation).Imaging guidance was performed so code
76942 needs to be included on the claim. Code 60000 is used
for the incision and drainage of a thyroglossal duct cyst not an
aspiration of a cyst. Code 10021 is also incorrect because the
aspiration was not performed by a fine needle.

23. Answer: A - The correct code for this service is 60220 (Total
Thyroid Lobectomy, Unilateral; With or Without Isthmusectomy).
The modifier -50 should not be included on the claim because
the procedure was only performed on the right side and mod-50
indicates that it was a bilateral procedure. Code 60240 is also
incorrect because it is used for a total or complete thyroidectomy
not just a lobe of the thyroid.

24. Answer: B - The physician should code for this service with
CPT code 60210 (Partial Thyroid Lobectomy, Unilateral; With
or Without Isthmusectomy). Modifier -LT should be appended
to indicate that the procedure was performed on the left side.
Modifier -50 would not be used because it indicates that a
bilateral service was performed not a unilateral service.

25. Answer: D - The correct code for this service is 60260


(Thyroidectomy, Removal of All of the Remaining Thyroid
Tissue Following Previous Removal or a Portion of the
Thyroid).The patient previously had a partial thyroidectomy
that caused some of her thyroid tissue to remain.
26. Answer: C - The correct codes for the procedure are: 60100
(Biopsy Thyroid, Percutaneous Core Needle) and 77002
(Fluoroscopic Guidance for Needle Placement (e.g. Biopsy,
Aspiration, Injection, Localization Device)). Pathology codes
88172 and 88173 should not be coded by the physician
because he or she did not perform the laboratory service. The
lab performed the services separately and should code for them
separately. Furthermore, the most appropriate radiology code is
77002 because the guidance provided was fluoroscopic.

27. Answer: B - The correct codes for a parathyroidectomy with


parathyroid autotransplantation on a 42-year-old male patient
are: 60500 (Parathyroidectomy or Exploration of Parathyroid(s))
and 60512 (Parathyroid Autotransplantation). Code 60512 is an
add-on code so it must be used in conjunction with the code for
the primary procedure (without modifier -51). Add-on codes are
considered modifier -51 exempt.

28. Answer: C - The correct code for this service is 60545


(Adrenalectomy, Partial or Complete; with Excision of Adjacent
Retroperitoneal Tumor). Code 60540 is not the correct code
because it does not include the excision of the retroperitoneal
tumor. In parenthesis under the adrenalectomy codes it states
“code 50323 should not be coded in addition to either 60540 or
60545.”
29. Answer: A - The correct code for a surgical laparoscopy with
partial right adrenalectomy is 60650 (Laparoscopy, Surgical with
Adrenalectomy, Partial or Complete) with modifier -RT to
indicate that the service was performed on the right adrenal
gland. Code 60540 is used for an open excisional procedure,
which does not apply in this case because the procedure was
performed laparoscopically.

30. Answer: B - Code 50590 is the correct code for


extracorporeal shock wave, also known as lithotripsy. Code
50580 is incorrect because this is a renal endoscopy with
removal of calculus. Code 50592 is incorrect because this is
ablation of a renal tumor with radiofrequency. Code 50593 is
incorrect because this is ablation of a renal tumor with
cryotherapy.

31. Answer: B - Code 50605 is the correct code for an


ureterotomy performed for the insertion of an indwelling stent.
Code 50600 is incorrect because this code is used for
aureterotomy with exploration or drainage. Codes 50610 and
50620 are both incorrect because these codes are used for
ureterolithotomy of different portions of the ureter.

32. Answer: C - 50780 with modifier -50 is the correct code for
bilateral ureteroneocystostomy, which is also known as the
anastomosis of both ureters to the bladder. Code 50780 is
incorrect because this code is for a single-sided
ureteroneocystostomy. Code 50785, with or without the
modifier -50, are both incorrect because there was no vesico-
psoas hitch or bladder flap done during this procedure.

33. Answer: A - 53600 is the correct code for the initial dilation of
a urethral stricture by passage of sound or urethral dilator for a
male. Code 53601 is incorrect because this is the code for a
subsequent urethral stricture dilation. Code 53605 is incorrect
because this code is used for the dilation of urethral stricture or
vesicle neck by passage of sound or urethral dilator with
general or conduction anesthesia of a male. Code 53660 is
incorrect because this is the dilation of a female urethra
including suppository or instillation.

34. Answer: B - 53060 is the correct code for the drainage of


Skene’s gland abscess. Code 53040 is incorrect because this
is the code for drainage of a deep periurethral abscess. Code
53080 is incorrect because this is the code for drainage of
perineal urinary extravasation that is uncomplicated and code
53085 is the code for a complicated drainage of the perineal
urinary extravasation.

35. Answer: C - 54161 is the correct code for a surgical


circumcision for someone older than 28 days of age. Code
54150 is incorrect because this code is for a circumcision done
using a clamp or other device. Code 54160 is incorrect because
this code is used for a surgical circumcision of an infant 28 day
of age or younger. Code 54162 is incorrect because this code is
for the lysis or excision of penile post-circumcision adhesions.

36. Answer: B - 54600 is the correct code for the surgical


reduction of torsion of testis with or without fixation of
contralateral testis. Code 54550 is incorrect because this code
is used for the
exploration of undescended testis. Code 54620 is incorrect
because this code is just for the fixation of contralateral testis.
Code 54640 is incorrect because this code is for an
orchiopexy.

37. Answer: C - 54530 is the correct code for a radical


orchiectomy due to tumor using the inguinal approach. Code
54520 is incorrect because this code is used for a simple
orchiectomy. Code 54522 is incorrect because this code is for a
partial orchiectomy. Code 54535 is incorrect because this is for
a radical orchiectomy with abdominal exploration.

38. Answer: C - 54840 is the correct code for excision of


spermatocele with or without epididymectomy. Code 54800 is
incorrect because this code is for a needle biopsy of the
epididymis. Code 54830 is incorrect because this code is for
the excision of a local lesion of the epididymis. Code 54860 is
incorrect because this code is for a unilateral epididymectomy.

39. Answer: C - 54861 is the correct code for a bilateral


epididymectomy. Code 54830 is incorrect because this code is
for the excision of a local lesion of the epididymis. Code 54860
is incorrect because this code is for a unilateral epididymectomy.
Code 54865 is incorrect because this code is for the exploration
of the epididymis with or without biopsy.

40. Answer: D - 54901 is the correct code for a bilateral


epididymovasostomy. Code 54861 is incorrect because this
code is for a bilateral epididymectomy. Code 54865 is incorrect
because this code is used for the exploration of epididymis with
or without biopsy. Code 54900 is incorrect because this is for a
unilateral epididymovasostomy.

41. Answer: A - 55000 is the correct code for a puncture


aspiration of a hydrocele with or without injection of
medication. Codes 55040 and 55041 are both incorrect
because there was no excision of a hydrocele performed.
Code 55060 is incorrect because there was no repair of tunica
vaginalis hydrocele.

42. Answer: C - 55041 is the correct code for a bilateral hydrocele


excision. Code 55000 is incorrect because this code is used for
a puncture aspiration of a hydrocele. Code 55040 is incorrect
because this is the code for a unilateral hydrocele excision.
Code 55060 is incorrect because this code is for the repair of a
tunica vaginalis hydrocele.

43. Answer: D - 55060 is the correct code for the Bottle type
repair of a tunica vaginalis hydrocele. Code 55000 is incorrect
because this code is for a puncture aspiration of a hydrocele.
Codes 55040 and 55041 are both incorrect because they are
used for the excision of either a unilateral or bilateral hydrocele.

44. Answer A - 55100 is the correct code for drainage of a scrotal


wall abscess. Code 55110 is incorrect because this is used for
scrotal exploration. Code 55120 is incorrect because this code
is used for the removal of a foreign body in the scrotum. Code
55150 is incorrect because this code is used for a resection of
the scrotum.
45. Answer: C - 55175 is the correct code for a simple
scrotoplasty. Code 55110 is incorrect because this is the code
for scrotal exploration. Code 55120 is incorrect because this
code is for foreign body removal within the scrotum. Code
55180 is incorrect because this code is used for a complicated
scrotoplasty.

46. Answer: D - 55180 is the correct code for a complicated


scrotoplasty. Code 55110 is incorrect because this code is for
scrotal exploration. Code 55120 is incorrect because this code
is for the removal of a scrotal foreign body and 55175 is
incorrect because this code is for a simple scrotoplasty.

47. Answer: B - 55250 is the correct code for a unilateral or a


bilateral vasectomy. Code 55200 is incorrect because this code
is for a vasotomy. Code 55300 is incorrect because this code is
for a vasotomy for vasograms, seminal vesiculograms or
epididymograms. Code 55400 is incorrect because this code is
for a vasovasostomy or vasovasorrhaphy.

48. Answer: D - 55450 is the correct code for either unilateral or


bilateral ligation of vas deferens. Code 55200 is incorrect
because there was no vasotomy performed. Code 55250 is
incorrect because no vasectomy was performed and 55400 is
incorrect because no vasovasostomy or vasovasorrhaphy was
performed.

49. Answer: A - 55500 is the correct code for the excision of a


hydrocele of the spermatic cord. Code 55520 is incorrect
because this code is for the excision of a lesion of the spermatic
cord. Code 55530 is incorrect because this is the code for an
excision of a varicocele or ligation of spermatic veins for
varicocele. Code 55550 is incorrect because this code is for a
surgical laparoscopy with ligation of spermatic veins for
varicocele.
50. Answer: C - 55550 is the correct code for surgical
laparoscopy with ligation of spermatic veins for varicocele.
Code 55520 is incorrect because this code is for the excision
of a lesion of the spermatic cord. Code 55530 is incorrect
because this code is for the excision of varicocele or ligation of
spermatic veins for varicocele. Code 55559 is incorrect
because this code is for an unlisted laparoscopic procedure
involving the spermatic cord.

51. Answer: D - 55540 is the correct code for the excision of


varicocele or ligation of spermatic veins for varicocele with
hernia repair. Code 55520 is incorrect because this code is for
the excision of a lesion of the spermatic cord. Code 55530 is
incorrect because this code is for the excision of a varicocele or
ligation of spermatic veins for varicocele. Code 55535 is
incorrect because this is for the abdominal approach of excision
of varicocele or ligation of spermatic veins for varicocele without
mention of hernia repair.

52. Answer: B - 55600 with modifier -50 is the correct code for a
bilateral vesiculotomy. Code 55600 is the correct code but is
missing the modifier to signify that it was a bilateral procedure,
instead of a unilateral procedure. Code 55559 with or without
the modifier -50 is incorrect because these are the codes for a
complicated vesiculotomy, unilateral and bilateral.
53. Answer: D - 55680 is the correct code for the excision of the
Mullerian duct cyst. Codes 55600 and 55605 are incorrect
codes because both of these are for vesiculotomy. Code 55650
is incorrect because this is used for vesiculectomy.

54. Answer: C - 55650 is the correct code for unilateral


vesiculectomy. Code 55600 is incorrect because this code is
for vesiculotomy. Code 55605 is incorrect because this code is
for a complicated vesiculotomy. Code 55680 is incorrect
because this code is for the excision of a Mullerian duct cyst.

55. Answer: A - 55700 is the correct code for a prostate biopsy


performed with a needle. Code 55705 is incorrect because this
biopsy is performed with an incision instead of a needle. Code
55706 is incorrect because this code is for prostate biopsies
with needle, transperineal, stereotactic template guided
saturation sampling including imaging guidance. Code 55720 is
incorrect because this code is for a prostatotomy.

56. Answer: D - 55815 is the correct code for a radical perineal


prostatectomy with bilateral pelvic lymphadenectomy including
the external iliac, hypogastric and obturator nodes on the same
day. Code 55801 is incorrect because this code is for a perineal
subtotal prostatectomy. Code 55810 is incorrect because this
code is for a radical perineal prostatectomy. Code 55812 is
incorrect because this code is for a radical perineal
prostatectomy with limited lymph node biopsies.

57. Answer: B - 56420 is the correct code for incision and


drainage of Bartholin’s gland abscess. Code 56405 is
incorrect because this is for an incision and drainage of a
vulva or perineal abscess. Code 56440 is incorrect because
this is the code for marsupialization of Bartholin’s gland cyst.
Code 56441 is incorrect because this code is for the lysis of
labial adhesions.
58. Answer: C - 56700 is the correct code for a partial
hymenectomy. Code 56442 is incorrect because this code is for
a simple incision hymenectomy. Code 56620 is incorrect
because this code is for a partial simple vulvectomy. Code
56800 is incorrect because this code is for plastic repair of the
introitus.

59. Answer: D - 57421 is the correct code for a colposcopy with


biopsies of the vagina/cervix. Code 57200 is incorrect because
this code is for colporrhaphy. Code 57282 is incorrect because
this code is for a colpopexy and 57420 is incorrect because this
code is for colposcopy without biopsies.

60. Answer: A - 57300 is the correct code for closure of


rectovaginal fistula by either vaginal or transanal approach.
Code 57305 is incorrect because this code is for closure of a
rectovaginal fistula through the abdominal approach. Code
57310 is incorrect because this code is for the closure of
urethrovaginal fistula and 57320 is incorrect because this code
is for the closure of a vesicovaginal fistula using the vaginal
approach.
61. Answer: A - 58300 is the correct code for intrauterine device
insertion. Code 58301 is incorrect because this code is for the
removal of the intrauterine device. Code 58321 is incorrect
because this code is for intracervical artificial insemination.
Code 58346 is incorrect because this code is for the insertion of
Heyman capsules for clinical brachytherapy.

62. Answer: A - 58150 is the correct code for a total abdominal


hysterectomy with or without removal of tube(s), with or
without removal of ovary(s). Code 58200 is incorrect because
this code is for a total abdominal hysterectomy including partial
vaginectomy with para-aortic and pelvic lymph node sampling,
with or without removal of tube(s), with or without removal of
ovary(s). Code 58260 is incorrect because this code is for
vaginal hysterectomy for uterus 250 grams or less and 58285
is incorrect because this code is for a radical vaginal
hysterectomy.

63. Answer: A - 58600 is the correct answer for the ligation or


transection of fallopian tube(s), using the abdominal or vaginal
approach and either providing a unilateral or bilateral ligation.
Code 58605 is incorrect because this code is for this procedure
when done during the postpartum hospitalization. Code 58611
is incorrect because this is an “add-on” code to a cesarean
delivery or other intra-abdominal surgery visit. Code 58615 is
incorrect because this is the code for occlusion of the fallopian
tubes by device (band, clip, Falope ring) using a vaginal or
suprapubic approach.

64. Answer: A - 58660 is the correct code for surgical


laparoscopy with lysis of adhesions (salpingolysis,
ovariolysis). Code 58661 is incorrect because this is a surgical
laparoscopy with removal of adnexal structures. Code 58671
is incorrect because this is the code for surgical laparoscopy
with occlusion of oviducts by device (band, clip or Falope
ring). Code 58673 is incorrect because this code is for
surgical laparoscopy with salpingostomy.

65. Answer: C - 58760 is the correct code for a fimbrioplasty.


Code 58672 is incorrect because this code is for surgical
laparoscopy with fimbrioplasty and there was no mention of a
laparoscopic procedure being performed. Code 58673 is
incorrect because this code is for a surgical laparoscopy with
salpingostomy and 58770 is incorrect because this code is for
salpingostomy.

66. Answer: A - 58800 is the correct code for the drainage of


ovarian cysts using the vaginal approach. Code 58805 is
incorrect because this is the code for drainage of ovarian cysts
using an abdominal approach. Code 58820 is incorrect because
this code is for the drainage of an ovarian abscess using an
open vaginal approach. Code 58822 is incorrect because this
code is used for the drainage of an ovarian abscess using the
abdominal approach.

67. nswer: B - 58943 is the correct code for partial or total


oophorectomy, unilateral or bilateral for ovarian, tubal or
primary peritoneal malignancy with para-aortic and pelvic lymph
node biopsies, peritoneal washings, peritoneal biopsies,
diaphragmatic assessments with or without salpingectomy with
or without omentectomy. Code 58940 is incorrect because this
code is for either a unilateral or bilateral partial or total
oophorectomy without mention of malignancy. Code 58950 is
incorrect because this code is for the resection of ovarian, tubal,
or primary peritoneal malignancy with bilateral salpingo-
oophorectomy and omentectomy. Code 58953 is
incorrect because this code is for a bilateral salpingo-
oophorectomy with omentectomy, total abdominal
hysterectomy and radical dissection for debunking.

68. Answer: B - 58900 is the correct code for a unilateral or


bilateral ovarian biopsy. Code 58820 is incorrect because this
code is used for the drainage of an ovarian abscess. Code
58920 is incorrect because this code is for a wedge resection
or bisection of the ovary, either unilateral or bilateral. Code
58925 is incorrect because this code is for a unilateral or
bilateral ovarian cystectomy.

69. Answer: C - 60252 is the correct code for total thyroidectomy


for malignancy with limited neck dissection. Code 60220 is
incorrect because this code is used for total unilateral thyroid
lobectomy with or without isthmusectomy. Code 60240 is
incorrect because this code is used for a total or complete
thyroidectomy. Code 60254 is incorrect because, while this code
is for a thyroidectomy for malignancy, it involves a radical neck
dissection instead of the limited dissection that was described in
the procedure above.

70. Answer: A - 60500 is the correct code for parathyroidectomy.


Code 60502 is incorrect because this code is for re-exploration
of the parathyroid’s. Code 60505 is incorrect because this code
includes mediastinal exploration using the sternal split or
transthoracic approach. Code 60512 is incorrect because this is
an “add-on” code for parathyroid auto transplantation.

71. Answer: B - 60502 is the correct code for re-exploration of


the parathyroid(s). Code 60500 is incorrect because this code
is for parathyroidectomy or initial exploration of parathyroid(s).
Code 60505 is incorrect because this code is for
parathyroidectomy or exploration of parathyroid(s) with
mediastinal exploration, using sternal split or transthoracic
approach. Code 60512 is incorrect because this is an “add-on”
code for parathyroid auto transplantation.

72. Answer: A - 60520 is the correct code for a total thymectomy


using the transcervical approach. Code 60521 is incorrect
because this code is for a thymectomy using the sternal split or
transthoracic approach without radical mediastinal dissection.
Code 60522 is incorrect because this code includes radical
mediastinal dissection and 60540 is incorrect because this
code is for an adrenalectomy.

73. Answer: C - 60522 is the correct code for total thymectomy


using the transthoracic approach with radical mediastinal
dissection. Code 60520 is incorrect because this code is for a
partial or total thymectomy using the transcervical approach.
Code 60521 is incorrect because this code is for partial or total
thymectomy using a sternal split or transthoracic approach
without radical mediastinal dissection. Code 60650 is incorrect
because this code is for a surgical laparoscopy with
adrenalectomy.

74. Answer: B - 60521 is the correct code for a thymectomy


using sternal split approach without radical mediastinal
dissection. Code 60520 is incorrect because this thymectomy
uses a transcervical approach. Code 60522 is incorrect
because this thymectomy includes radical mediastinal
dissection. Code 60540 is incorrect because this code is for
an adrenalectomy.
75. Answer: C - 60650 is the correct code for a surgical
laparoscopy with total adrenalectomy. Codes 60540 and 60545
are both incorrect because these codes are for adrenalectomies,
but are not for the laparoscopic approach. Code 60659 is
incorrect because this code is for an unlisted laparoscopic
procedure of the endocrine system and there is a code listed for
laparoscopic adrenalectomy.

76. Answer: A - 60540 is the correct code for a partial or


complete adrenalectomy or exploration of the adrenal gland with
or without biopsy using either the transabdominal, lumbal or
dorsal approaches. Code 60545 is incorrect because there is no
mention of removal of adjacent tumor. Codes 60650 and 60659
are both incorrect because this was not a laparoscopic
procedure.

77. Answer: B - 60545 is the correct code for a partial or complete


adrenalectomy with excision of an adjacent retroperitoneal
tumor. This code can be used with or without biopsy. Surgery
can be performed using multiple approaches including
transabdominal, lumbar, or dorsal. Code 60540 is incorrect
because this procedure is not just for the adrenalectomy, it also
needs to include the excision of the tumor. Codes 60650 and
60659 are incorrect because both of these codes are for
laparoscopic procedures.

78. Answer: B - 60605 is the correct code for excision of both the
carotid body tumor and the carotid artery. Code 60600 is
incorrect because this code is for just the excision of the carotid
body tumor and does not involve excision of the carotid artery.
Code 61703 is incorrect because this code is for surgery of
intracranial aneurysm, including application of an occluding
clamp to the carotid artery. Code 61705 is incorrect because
this code is for an aneurysm surgery of a vascular malformation
by intracranial and cervical occlusion of the carotid artery.

79. Answer: A - 60600 is the correct code for excision of carotid


body tumor without excision of the carotid artery. Code 60605 is
incorrect because this code includes excision of the carotid
artery. Codes 61703 and 61705 are both incorrect because
these are codes for aneurysm surgeries involving carotid artery
occlusion.

80. Answer: C - 60659 is the correct code for an unlisted


laparoscopic procedure involving the endocrine system. Code
60600 is incorrect because this code is for excision of the
carotid body tumor without carotid artery excision. Code 60605
is incorrect because this code is for excision of both the carotid
body tumor and the carotid artery. Code 60699 is incorrect
because there is a specific “unlisted” laparoscopic procedure
code for the endocrine system.
60,000 SERIES (20 QUESTIONS) – ANSWER KEY
& RATIONALE

1. Answer: C - Definitive procedure, approach procedure, and


repair or reconstruction are the three things by which surgeries
of the skull base are categorized. The approach procedure is
the procedure that is done to gain access or exposure to the
lesion or operation site. The definitive procedure is the actual
biopsy, excision, or treatment of the lesion or operation site. The
repair or reconstruction procedure is only reported separately if
it is extensive or complicated, and it includes the closure of the
surgical site such as skin grafts.

2. Answer: D - The correct code for the surgery is 61500


(Craniotomy; with Excision of Tumor or Other Bone Lesion of
Skull).Modifier -52 and -53 do not need to be used to indicate
that it was a discontinued procedure because the procedure was
completed. The original procedure, a craniotomy was converted
to a modified procedure, a craniotomy. Code 61518 is also
incorrect because it is used for a craniotomy, for the excision of
brain tumor, rather than excision of bone lesion of skull.

3. Answer: C - The correct code for the procedure is 61313


(Craniectomy or Craniotomy for Evacuation of Hematoma,
Supratentorial; Intracerebral) combined with modifier -22 to
indicate that the procedure was an increased procedural
service. Code 61315 is inappropriate because it is used for an
infratentorial, intracerebellar hematoma.

4. Answer: B - The code for the procedure is 62201


(Ventriculocisternostomy, Third Ventricle; Sterotactic,
Neuroendoscopic Method). Code 62200 is for the same
procedure, but without using the neuroendoscope. Code 62180
is also for the same procedure, but it does not take into account
the third ventricle. Code 62220 is used for the creation of a
shunt, but in this case, a shunt was not created.

5. Answer: C - The correct code for this service is with 62270


(Spinal Puncture, Lumbar, Diagnostic) combined with modifier
-63 to indicate that the patient weighed less than 4 kg. Code
62270 (Spinal Puncture, Therapeutic, for Drainage or
Cerebrospinal Fluid) is incorrect because the lumbar puncture
was performed to evaluate the spinal column fluid not treat the
spinal column by performing the lumbar puncture.

6. Answer: A - The physician should code for this service with


codes 63030 (Laminotomy (Hemilaminectomy), with
Decompression or Nerve Root(s), Including Facetectomy,
Foraminotomy and/or Excision of herniated Intervertebral Disc;
1 Interspace, Lumbar) with the add-on code 63035 (Each
Additional Interspace, Cervical or Lumbar). Although there were
only two herniated discs removed, there were three interspaces
involved in the procedure therefore the correct number of units
for the add-on code 63035 is two.

7. Answer: B - The correct CPT codes for this service are 62350
(Implantation, Revision, or Repositioning of Tunneled,
Intrathecal or Epidural Catheter, for Long-Term Medication
Administration via an External Pump or Implantable
Reservoir/Infusion Pump; without Laminectomy) and 62355
(Removal of Previously Implanted Intrathecal or Epidural
Catheter).
The removal of the intrathecal catheter needs to be coded
separately with modifier -51, indicating that it was multiple
procedures.

8. Answer: D - The correct code for the procedure is 63283


(Laminectomy for Biopsy/Excision of Intra spinal Neoplasm;
Intra dural, Sacral). Code 63275 is incorrect because it is used
for an extradural, cervical lesion. Code 63273 is incorrect
because it is used for the excision of a lesion other than a
neoplasm, and code 63278 is incorrect because it is used for an
extra dural, sacral lesion.

9. Answer: D - The correct codes for this procedure are: 63700


(Repair of Meningocele; Less than 5 cm Diameter) and 13101
(Repair, Complex, Trunk; 2.6 cm to 7.5 cm). Code 63700 should
not be appended with modifier -63 because the coder is
specifically instructed not to append modifier - 63 to the code in
parenthesis under the code. Modifier -51 should be appended to
13101to indicate that it was multiple procedures.

10. Answer: A - The codes that need to be reported are: 64831


(Suture of Digital Nerve, Hand or Foot; 1 Nerve) and 64832
(Suture of Digital Nerve, Hand or Foot; Each Additional Digital
Nerve) with two units. These codes include all three sutures for
the digital nerves in the left hand. The other codes that need to
be reported are 64834 (Suture of 1 Nerve; Hand or Foot,
Common Sensory Nerve) and 64837 (Suture of Each Additional
Nerve, Hand or Foot) with two units. This accounts for the three
additional common sensory nerves that were sutured in the
feet.

11. Answer: B - 62270 is the correct code for a diagnostic lumbar


spinal puncture. Code 62269 is incorrect because this is the
code for a biopsy of the spinal cord with a percutaneous
needle. Code 62272 is incorrect because this is the code for a
therapeutic spinal puncture for drainage of cerebrospinal fluid
by needle or by catheter. Code 62273 is incorrect because this
code is for the injection/epidural of blood or clot patch.

12. Answer: C - 63005 is the correct code for lumbar laminectomy


except when done for spondylolisthesis. Code 63001 is
incorrect because this code is for cervical laminectomy. Code
63003 is incorrect because this is for thoracic laminectomy and
code 63011 is incorrect because it is for sacral laminectomy.

13. Answer: B - 64553 is the correct code for a percutaneous


implantation of a cranial nerve neurostimulator electrode array.
Code 64550 is incorrect because this code is for the application
of a surface or transcutaneous neurostimulator. Code 64555 is
incorrect because this code is for a peripheral nerve and 64561
is incorrect because this code is used for the sacral nerve.

14. Answer: A - 64400 is the correct code for an aesthetic agent


injection into the trigeminal nerve, also known as a trigeminal
nerve block. Code 64402 is incorrect because this code is for
the injection of a facial nerve. Code 64405 is incorrect because
this code is for the greater occipital nerve and 64408 is
incorrect because this code is for the vagus nerve.
15. Answer: C - 64740 is the correct code for transection or
avulsion of the lingual nerve. Code 64732 is incorrect because
this is the transection or avulsion of the supraorbital nerve.
Code 64734 is incorrect because this code is for the infraorbital
nerve and 64742 is incorrect because this code is for the
transection or avulsion of the facial nerve.

16. Answer: D - 64712 is the correct code for a neuroplasty of


the sciatic nerve. Code 64702 is incorrect because this code is
for a digital neuroplasty and 64704 is incorrect because this
code is for a neuroplasty of a nerve in the hand or foot. Code
64708 is incorrect because this is for a neuroplasty of a major
peripheral nerve of the arm or leg.

17. Answer: B - 64447 is the correct code for a single femoral


nerve anesthetic agent injection, also known as a nerve block.
Code 64445 is incorrect because this code is for the sciatic
nerve and code 64449 is incorrect because this code is for the
lumbar plexus nerve. Code 64450 is incorrect because this is
for “other” peripheral nerve or branch anesthetic agent injection.

18. Answer: C - 64517 is the correct code for anesthetic agent


injection of the superior hypogastric plexus. Code 64505 is
incorrect because this injection is for the sphenopalatine
ganglion. Code 64510 is incorrect because this code is for
injection of the stellate ganglion. Code 64520 is incorrect
because this code is for injection of the lumbar or thoracic
nerves.

19. Answer: C - 64680 is the correct code for destruction of the


celiac plexus nerve by neurolytic agent with or without radiologic
monitoring. Code 64646 is incorrect because this code is for the
chemodenervation of trunk muscles. Code 64650 is incorrect
because this code is for the chemodenervation of eccrine
glands. Code 64681 is incorrect because this is the code for the
destruction of the superior hypogastric plexus.

20. Answer: B - 64760 is the correct code for the transection of


the vagus nerve (vagotomy) using an abdominal approach.
Code 64755 is incorrect because this is the code for transection
of the vagus nerves limited to the proximal stomach. Codes
64763 and 64766 are both incorrect because these codes
involve the transection of the obturator nerves.
ANATOMY (32 QUESTIONS) – ANSWER KEY &
RATIONALE

1. Answer: C-An umbilical hernia protrudes from the belly button


(the umbilical area). An umbilical hernia is a tear in the wall in
the umbilical area, which allows the small intestines to protrude
through the belly button.

2. Answer: A-A patient with an injured neck sustained an injury to


the cervical area of the spine. The cervical area is the top
section of the spine, which lies in the neck. The lumbar area is
the lower section of the spine, which lies in the lower back. The
thoracic area is in the middle section of the spine. The
coccygeal area is the tailbone, which lies at the very end of the
spinal column.

3. Answer: A-Adipose tissue, also known as fat, is a connective


tissue that helps the body function. Epithelial tissue covers and
lines different areas of the body, rather than connecting it
together. Nervous tissue transfers messages throughout the
body, and cardiac tissue is a type of muscle tissue.

4. Answer: C - The pancreas is part of the endocrine and


gastrointestinal systems. The pancreas is considered part of the
endocrine system because it contains endocrine cells and it is
considered part of the gastrointestinal system because it aids in
digestion.

5. Answer: D - The epididymis is the site of sperm maturation. The


prostate secretes and buffers seminal fluid, sperm ducts
transport sperm between the epididymis and prostate, and
seminal vesicles secrete additional fluid that promotes
movement of sperm through the urethra.
6. Answer: C - The three types of blood cells are erythrocytes,
leukocytes, and thrombocytes (or platelets). Neutrophils,
eosinophils, and basophils are all types of leukocytes.
Monocytes and leukocytes contain the same types of blood
cells because a monocyte is a type of leukocyte.

7. Answer: D-Spongy bone tissue makes up the insides of bones.


Spongy bone tissue helps make the bones lighter in weight and
provides an area for the bone marrow to produce red blood
cells. Compact bone tissue makes up the structure of bones
and surrounds spongy bone tissue.

8. Answer: A - Inflammation of tendons or the connective tissue


that attaches tendons to muscles or bones is referred to as
tendonitis. Scoliosis is a congenital curvature of the spine. Gout
is a metabolic disease that occurs when excess uric acid
crystalizes and deposits in the joints, muscles or tendons.
Bursitis is an irritated or infected bursa that is caused by
repetitive movements, congenital defects, and strains.

9. Answer: B - The three types of muscles are smooth, skeletal


and cardiac. Cardiac muscles are special types of involuntary
muscles that are only found in the heart. These muscles not
only form the walls of the heart, they also force blood from the
atria into the blood vessels and arteries when they contract.
10. Answer: B - Jaundice is a condition caused by an excess of
bilirubin in the body, which causes a yellowing of the skin. Uric
acid content in the body does not affect skin color. Melanin in
the body results in a darkening of the skin. Carotene gives the
skin a yellowish hue, but it does not cause jaundice.

11. Answer: C - The patient’s level of injury is at the thoracic


level, which is in the middle of the back. This is indicated by
his ability to move his arms, but not his legs. If it was in the
cervical area he would be paralyzed from the neck down and
would not be able to move his arms. If it was in the lumbar
area he would still be able to move his arms, but not his legs
because his injury would be lower than the chest area.

12. Answer: B - The tympanic membrane is often referred to as


the ear drum. Taste buds are on the tongue and have nothing to
do with the auditory system. The stirrup or stapes is a small
bone in the middle ear and the inner ear is the internal part of
the auditory system.

13. Answer: D - A nasal polyp can obstruct the nasal passageway


making it difficult to breathe. If this is the case, surgical removal
of the nasal polyps may be necessary. They are normally
benignant generally do not cause cancer in the patient. In
addition, they are not large enough to change the shape of the
nose or release histamines that cause allergic reactions,
although they can develop as a result of chronic inflammation.

14. Answer: B-The term “peripheral” refers to the extremities, so


the type of cyanosis that presents on the fingers and toes is
peripheral cyanosis. Central cyanosis refers to the center of the
body or the torso. Medial cyanosis refers to the midline of the
body. Embolic cyanosis refers to cyanosis caused by a blocked
blood vessel, not an area of the body.
15. Answer: B - The term “alopecia” is commonly referred to as
hair loss. Alopecia can be acute or chronic, resulting in
baldness. It can be an inherited trait but can also be the result
of chemotherapy, a hormonal imbalance, infections, severe
stress, medication side-effect.

16. Answer: A - When a doctor manipulates a dislocated joint


back into place he reduces the subluxation. A subluxation is a
joint dislocation and the manipulation of a joint back into place is
commonly referred to as a reduction.

17. Answer: B - The difference between a craniotomy and a


craniectomy is that a craniotomy temporarily removes part of
the patient’s skull to allow access inside the skull or brain during
the procedure. The skull is then replaced once the procedure is
completed, while a craniectomy removes a part of the patient’s
skull and does not replace it once the procedure is completed.

18. Answer: A - If a doctor performed only the antepartum


portion of a patient’s obstetrical care, she provided services to
the patient before she delivered the baby. Antepartum means
before delivery. Postpartum refers to the period after delivery.
19. Answer: B - If a patient is in the prone position, he is lying flat
on his stomach. If the patient lies flat on his back he is in the
supine position. If the patient lies flat on his back with his feet
elevated he is in the Trendelenburg position. If the patient is
sitting up straight he is in the Fowler’s position.

20. Answer: D - The superior eyelid is also referred to as the


upper eyelid. The term “superior” means “up” or “above” and so
the superior eyelid would be the eyelid on the top. The “inferior”
eyelid would be the one on the bottom.

21. Answer: A-This procedure included removing part of the


patient’s lung. The term “partial” refers to something less than
total and the suffix “-ectomy” means removal, rather than cutting
into or creating an opening into. The prefix “pnuemo-” refers to
the lung.

22. Answer: C - The suffix “-megaly” refers to the enlargement


of, as in enlargement of the heart. The suffix for inflammation
is “-itis.” The suffix for pain is “-algia.” The suffix for softening
is “- malacia.”

23. Answer: D - The medical term for a heart attack, which refers
to the death of heart muscle, is myocardial infarction. The term
“myocardial” refers to the cardiac muscle of the heart, or the
middle layer of the heart, and “infarction” refers to tissue death.
So “myocardial infarction” refers to death of the tissue of the
middle layer of the heart or heart muscle, which is commonly
referred to as a heart attack.

24. Answer: B - The nasal turbinates are part of the respiratory


system. The nasal turbinates help to humidify the air that we
breathe, as well as provide intonation for the voice, thus, they
are part of the respiratory system. They also serve to reduce
pressure in the head.
25. Answer: E- Contusions and hematoma are both referred to as
bruises. Hematomas are localized masses of extravasated
blood, which typically remain confined in a specific area such as
the skin, where the bruise forms. A contusion is an injury that
causes internal damage and bruising, but does not break the
skin.

26. Answer: C - The difference between a laparoscopy and a


laparotomy is that one involves the use of an endoscope and
the other is a cutting procedure. They are both invasive
procedures that involve cutting into the skin, but a laparoscopy
uses an endoscope to percutaneously perform the procedure,
while a laparotomy makes a large incision into the patient in
order to perform a different procedure.

27. Answer: A - The ulna, radius and humerus are the three major
bones found in the arm. The radius and ulna make up the lower
half of the arm and the humerus supports the upper half of the
arm. The tibia and fibula make up the lower half of the leg and
the femur supports the upper half of the leg.
28. Answer: A - The vertebral interspace is a non-bony
compartment between two vertebral segments that includes the
intervertebral disc. A paravertebral facet joint is where the
sides of one vertebra overlap the one next to it on the posterior
side. The vertebral corpus is the main portion of the vertebra
itself. The vertebral foramen is the opening in each vertebra
through which the spinal cord passes.

29. Answer: D - The bones inside of the nose that are shaped
like a spiral shell serve to humidify, filter, and warm the air
breathed in through the nose and are referred to as nasal
turbinates. Nasal turbinates are also referred to as the nasal
conchae. They are divided into three sections; inferior, middle,
and superior. Portions or all of the nasal turbinates may be
removed due to chronic congestion or growth of a neoplasm
within the turbinates.

30. Answer: B - The region that separates the lungs and holds
the heart, esophagus, trachea, and great vessels is referred to
as the mediastinum. The visceral pleura is the sac that holds
the lungs, and the parietal pleura is a continuation of the
visceral pleura, which lines the diaphragm and thoracic cavity.
The diaphragm is a muscle that aids in breathing and divides
the thoracic cavity from the abdominal cavity.

31. Answer: B - Diaphragmatic hernia repair codes are divided


based upon the age of the patient and whether or not the hernia
is acute or chronic. There are only three hernia codes, 39503
(Neonatal Diaphragmatic Hernia Repair) and codes 39540 and
39541 (Diaphragmatic Hernia Repair for Other than an
Neonate). The codes 39540 and 39541 are divided into
traumatic acute and traumatic chronic hernias.

32. Answer: C - The diaphragm is voluntary and involuntary. It is


involuntary because you do not have to consciously think
about taking a breath every time you inhale or exhale. This is
illustrated when you hiccup involuntarily. It is also voluntary
because you can change the way you breathe, for instance
when you hold your breath.
ANESTHESIA (15 QUESTIONS) – ANSWER KEY
& RATIONALE

1. Answer: B-The time reported for an anesthesia service begins


when the anesthesiologist begins prepping the patient and ends
when the anesthesiologist is no longer in personal post-
operative attendance. According to anesthesia coding
conventions, the time reported for an anesthesia service begins
when the anesthesiologist begins in the pre-operative session,
remains throughout the operative session, and ends in the post-
operative session when the patient is no longer under the care
of an anesthesiologist and can be transferred to post-operative
supervision.

2. Answer: A - The anesthesiologist should report 4 hours 45


minutes as the time performed for this service. According to
anesthesia coding guidelines, the time reported should begin
when the anesthesiologist begins prepping the patient for
anesthesia services and end when the anesthesiologist is no
longer in attendance. Because the anesthesiologist was in
attendance an additional 30 minutes in the post-op recovery
room, the full time should be reported.

3. Answer: C-General anesthesia is typically administered


intravenously, results in a complete loss of consciousness and
affects the entire body. Regional anesthesia only affects a
specific body area, and local anesthesia affects a local body
area. Intrathecal anesthesia is a type of local anesthesia that is
injected directly into the spinal fluid.

4. Answer: A - Modifier-23 (Unusual Anesthesia) is the correct


modifier. The dental procedure would have normally been
performed under local anesthesia, but due to the unusual
circumstances, an anesthesiologist was required to administer
a heavier dose of anesthesia to the patient. Modifier -47 is
incorrect because the anesthesia was administered by the
anesthesiologist not a surgeon.

5. Answer: D - The correct anesthesia codes for this service is


01960 (Anesthesia for vaginal delivery only), and the correct
physical status modifier is P2, which indicates that the patient
has a mild systemic disease. It is necessary to report that the
patient has well-controlled Type I diabetes mellitus, which in this
case is not a severe systemic disease. This condition should be
reported with physical status modifier P2.

6. Answer: D - The anesthesia service that should have been


reported is 01990-P6, (Physiological support for harvesting or
organ(s) from brain-dead patient). Modifier -P6 also should
have been reported to indicate that the patient’s physical status,
which in this case, is a declared brain- dead patient whose
organs were being removed for donor purposes.

7. Answer: C - The correct anesthesia code is 01214, (Anesthesia


for arthroscopic procedures of hip joint; total hip arthroplasty
only). Anesthesia guidelines state that when multiple surgical
procedures are performed during a single anesthetic
administration, the anesthesia code representing the most
complex procedure is reported. In this case, the hip arthroplasty
is the most complex procedure, which makes 01214 the only
code that should be reported.
8. Answer: C - The correct codes are 01952, (Anesthesia for
second and third-degree burn excisions or debridement, with or
without skin grafting, any site, between 4% and 9% of total body
surface area), which accounts for the first 9% of burned area
and 01953, (Anesthesia for second and third-degree burn
excisions or debridement, with or without skin grafting, any site,
each additional 9% total body surface area or part
thereof),which accounts for the remaining 4%. The code 99100,
(Anesthesia for patient of extreme age, younger than 1 year and
older than 70) also needs to be reported to account for the
patient’s advanced age.

9. Answer: B - The correct code is 92225-47, (Ophthalmoscopy,


extended, with retinal drawing, with interpretation and report).
Modifier -47, (Anesthesia by Surgeon) must be appended to
indicate that the anesthesia service was performed by the
physician performing the procedure. According to modifier
guidelines, modifier -47 is not to be used on the anesthesia
procedure code, but rather appended to the basic service code
which in this case is 92225. The anesthesia service code,
00148, should not be reported.

10. Answer: C - The correct code is 00792, (Anesthesia for intra-


peritoneal procedures in upper abdomen including laparoscopy;
partial hepatectomy or management of liver hemorrhage) and
the correct physical status modifier is -P5, which indicates that
the patient is in a moribund state and not expected to live
without the procedure. The tibia manipulation and percutaneous
fixation should not be reported separately because it is
considered a minor anesthetic procedure in comparison to the
management of the liver hemorrhage.

11. Answer: D - The correct code for the procedure is 01490,


(Anesthesia for lower leg cast application, removal, or repair).
The patient received anesthesia for the cast application, which
was only administered in the lower leg. There were no other
anesthesia services provided.

12. Answer: C - The correct codes are 00192 (Anesthesia for


procedures on facial bones or skull; radical surgery), 99135,
(Anesthesia complicated by utilization of controlled
hypotension), and 99100, (Anesthesia for patient of extreme
age, younger than 1 year or older than 70). 00211 (Anesthesia
for intracranial procedures; craniotomy or craniectomy for
evacuation of hematoma) is not appropriate, in this case,
because the craniectomy was not performed as a result of a
hematoma. Also, because both qualifying circumstances
codes apply, both of them must be appended to the anesthesia
code.

13. Answer: C - Time starts when the anesthesiologist is present


when he/she begins to prepare the patient for induction of
anesthesia in the operating room (or equivalent area) and ends
when the anesthesiologist is no longer in personal attendance.
Answer A is incorrect because the anesthesiologist wasn’t
present when the time first began and ended at the end of
surgery, not at the end of the anesthesiologist’s attendance.
Answer B is incorrect because the time ended at the surgery
end time and not at the end of the anesthesiologist’s
attendance. Answer D is incorrect because the time started
before the anesthesiologist was in attendance.
14. Answer: B- 00834 is the correct code for a hernia repair in the
lower abdomen for a child younger than one year of age. 99100
is a correct add on code for patients of extreme age (younger
than one year and older than 70). Code 99140 is a correct add
on code for a procedure complicated by emergency conditions,
such as significant hypotension. Code 99135 is an incorrect
add-on code because this code is for anesthesia complicated by
utilization of controlled hypotension.

15. Answer: C - 00567 is the correct code for coronary artery


bypass grafting (CABG) with pump oxygenator. Code 99116 is
the correct add-on code for anesthesia complicated by the
utilization of total body hypothermia. Code 00566 is incorrect
because this code is for CABG without pump oxygenator. Code
99135 is an incorrect add-on code because this is the code for
anesthesia complicated by the utilization of controlled
hypotension.
CODING GUIDELINES (19 QUESTIONS) – ANSWER
KEY & RATIONALE

1. Answer: A-In an outpatient setting, the primary diagnosis is the


reason the patient came in for the visit, and received services
on that day. The primary diagnosis will not always be the code
that the doctor lists first, as the doctor may not know the coding
guidelines. The problem that causes the patient the most pain
will only be the primary diagnosis if it is the reason the patient
receives services, but not all diagnoses cause pain. Diagnosis
codes also do not have any effect on reimbursement, only
procedure codes do.

2. Answer: C - Appointments for problems that are not related to


the procedure are not typically included in a global package.
Global packages include: all aspects of a surgical service,
including, but not limited to the surgical procedure itself, any
follow-up appointments within the global period, and
appointments for problems related to the surgical procedure.
Appointments for problems that are not related to the surgery
are always separate charges, regardless of whether they occur
within the global period or not.

3. Answer: B - Unbundling a code occurs when multiple procedures,


services and supplies are listed with their own separate, distinct
codes. Many surgical procedures consist of multiple elements
such as: gaining access to an organ by performing an open
incision, completing the surgery, and closing the wound. The
primary surgical code includes all of these elements. In other
words, it is a bundled code. Unbundling the code would consist
of listing the incision, surgical procedure, and surgical closure
codes separately.

4. Answer: C - When a CPT code has the words “separate


procedure” in parenthesis after the code description, you only
code for this procedure if it was the only thing performed. Codes
that are designated as “separate procedures” should only be
coded if they are the only procedures performed on that part of
the body on the specified day. If any other procedure was
performed on that part of the body, another procedure code
should be used and this “separate procedure” code is
considered bundled into the major procedure.

5. Answer: C - 1100 is a surgery section CPT code because it is


within the surgery section code range, 10021 to 69990. 99212
is an evaluation and management section CPT code, 85025 is
a pathology and laboratory section CPT code, and 70020 is a
radiology section CPT code.

6. Answer: A - When you see the symbol # in front of a CPT code,


it means that the code is listed out of numerical order.
Throughout the CPT book, whenever you see the # symbol in
front of a numerical CPT code, it means that the code has been
re-sequenced so that it is not listed in numerical order with the
rest of the CPT codes within that section.

7. Answer: C - When you include an unlisted service procedure


code on a claim, you must also include a special report. This
special report needs to include a precise description of the
service or procedure and the nature of and need for the service.
The special report should also include the time, effort, and
equipment used to complete the service.
8. Answer: D - In the CPT manual, Appendix C lists clinical
examples of evaluation and management procedures. In
Appendix C, you will find multiple examples of each level of
evaluation and management procedures. Reviewing these
examples will not only help you determine which specific levels
of codes should be used in each coding situation, it will also help
you make sure the right code is assigned.

9. Answer: B - Before you include an unlisted services procedure


code on a claim, you do NOT have to review Category II codes
to make sure an appropriate code does not exist. In order to
assign an unlisted procedures code, you first must review all of
the CPT codes in that section to make sure a more appropriate
code does not exist. You must also check to see a modifier is
available to include on the claim. Lastly, before assigning an
unlisted procedure code, you must check if a Category III code
is more appropriate.

10. Answer: B - The respiratory system subsection in the CPT


manual contains, but is not limited to, the nose, accessory
sinuses, and trachea/bronchi. Procedure codes for the mouth,
throat, and stomach are located in the digestive system
subsection, while procedure codes for the heart are located in
the cardiovascular system subsection.

11. Answer: D - Modifier-32 is the appropriate modifier to use on


the claim. Modifier-32, mandated services, is used when a
procedure is performed because an official body such as a car
or life insurance agency, requests it. Modifier-22 is appropriate
for a unusual procedural service, Modifier-51 is used for
multiple procedures, and Modifier-99 is used for multiple
modifiers.

12. Answer: A - The modifier that you would use to indicate that
they were two separate procedures performed on the same date
of service would be -59 (Distinct Procedural Service). In the
description of modifier -59, it states that it is used to report two
services that were performed on “different site or organ
system[s],” and as such, it is used to report a repair done on the
same date, on two separate body areas (the scalp and arm).

13. Answer: B - Modifier -90 (Outside Laboratory)is the


appropriate modifier in this case. Modifier - 90 is used when the
physician’s office bills for the laboratory service, even though
they sent the lab sample to an outside laboratory. When the
insurance company pays the physician for the lab sample, the
physician’s office reimburses the lab for the cost of the service.

14. Answer: B- - The two most common modifiers used in


radiological services are -TC and -26. Modifier -TC is used to
indicate that only the technical component of the service was
performed. The technical component is used to bill for both the
radiology technician’s services and the machine. Modifier -26 is
used to indicate professional radiological services (radiological
supervision and interpretation). This modifier indicates that the
physician is billing for reading the image and interpreting the
results not for taking the image.

15. Answer: C - The organizations that approved the ICD-10-CM


guidelines and who make up the Cooperating Parties are the
American Hospital Association (AHA), American Health
Information Management Association (AHIMA), Centers for
Medicare & Medicaid Services (CMS), and the
National Center for Health Statistics (NCHS). The ICD-10-CM is
based on the ICD-10, which is the statistical classification of
disease that is published by the World Health Organization. The
U.S. Federal Governments Department of Health and Human
Services is mentioned in the official guidelines because two of
its departments (CMS and NCHS) are part of the organizations
that make up the Cooperating Parties for the ICD-10-CM.

16. Answer: A - The proper way to locate a code in the ICD-10-


CM is to locate the code in the Alphabetic Index then flip to the
Tabular List to identify any guided notations. It is essential to use
both of these when locating a code. The Alphabetic Index alone
will not always give full code information and may leave out
certain notations that are required for the code to be complete.
The Tabular List is identified easily by locating the right code in
the Alphabetic Index. Both need to be used to locate a code.

17. Answer: B - Laterality is specified in the ICD-10-CM codes.


Some codes have “right,” “left,” and “bilateral” options listed.
Other codes do not have “bilateral” listed as an option, in which
case you would use both the “left” and “right” codes individually
to stipulate bilateral diagnoses. These codes can only be used
if the laterality is mentioned in the medical record.

18. Answer: C - Both sign/symptom and unspecified codes are


appropriate to use when the actual diagnosis is unable to be
made. In some cases, the infecting organism needs to be
identified for a specific code to be used, so in these cases when
the infecting organism isn’t known at the time of the patient’s
encounter an “unspecified” code is appropriate to be used. ICD-
10-CM does have codes for signs and symptoms or
“unspecified” codes. Although they are not listed for every
possible diagnosis, if they are an option they are allowed to be
used based on the information available at the time of the
patient’s visit/encounter.
19. Answer: D - Sequela or “late effects” are considered the
conditions or residual effects after the acute phase of the
illness or injury has ended. There is no time limit. It could occur
months or years after the initial encounter.
COMPLIANCE & REGULATORY (29 QUESTIONS) –
ANSWER KEY & RATIONALE

1. Answer: C - The two parts of Medicare insurance that the patient


will need to have to pay for the charges are: Medicare Parts B
and D. Medicare Part B covers outpatient office visits, which in
this case, would cover the dermatologist’s office visit charges,
while Medicare Part D covers pharmacy or prescription
charges, which in this case, would cover the patient’s pain
medication charges. Medicare Part A only covers inpatient
services, such as inpatient hospital or hospice services, while
Medicare Part C is a combination program that combines
Medicare A and B services.

2. Answer: B - Tricare patients typically must see physicians at their


military treatment facility. Depending on the Tricare plan, the
patient may be able to see a physician off-base, but typically the
must see the physician on-base, at his military treatment facility.
A patient can see a private practice physician, if he has Tricare
Prime Remote or if he has an non-availability statement that
states that he has permission to see a physician off-base.

3. Answer: B - Medicaid is always the payer of last resort. This


means that if a patient has more than one type of insurance
coverage, and one of the insurances is Medicaid, then the biller
must bill the other insurance first and Medicaid second.
Medicaid will never pay first, if the patient has more than one
type of insurance coverage.

4. Answer: C - Worker’s compensation insurance will only cover his


medical costs if Larry was wearing the back brace and hernia
belt as he was lifting the box because that is one of his safety
requirements. The safety requirement states that Larry must
wear the proper safety equipment when performing his job
duties. If he is injured while not wearing the proper safety
equipment, then worker’s compensation insurance is not liable
for his injury .

5. Answer: B - During the processing of the claim, insurance will


likely pay for the claim as medically necessary. Even though it is
a rhinoplasty, which can be done as a cosmetic procedure, in
this case it is considered medically necessary due to the fact
that the child is experiencing breathing problems from the
congenital deformity and needs a rhinoplasty to correct the
problem. If the procedure was purely for cosmetic reasons (if the
patient wanted simply to change the shape of his nose), then
insurance would not pay for it, as it would not be medically
necessary.

6. Answer: A - You bill Blue-Cross Blue-Shield first as the


primary payer, and send a secondary claim to Medicaid, once
the primary insurance has paid. Medicaid is always the payer
of last resort, and as such is always billed after a commercial
insurance processes the claim.

7. Answer: D - Medical billing fraud is billing for services that were


not performed in the office. An example would be billing for an
office visit for a patient who never came into the office. Medical
billing abuse is billing for services that were not medically
necessary. An example of medical billing abuse would be billing
a high-level E&M service when only a low-level E&M service
was performed.
8. Answer: B - The purpose of an internal audit is to allow the
coders and billers in your office to make sure your claims were
billed correctly. There are two types of internal audits:
prospective, which are completed before claims are sent out
and retrospective, which are performed after claims are sent
and paid. Both internal audits are conducted to make sure your
office is coding and billing correctly.

9. Answer: D - All of the above, HIPAA was created to protect


patient privacy, enact ways to uncover fraud and abuse, and to
create standards of electronic transactions. HIPAA protects
patient privacy through its strict standards of confidentiality,
allows organizations like the OIG to uncover fraud and abuse,
and gives these organizations the power to investigate and
prosecute suspected fraud and abuse cases. HIPAA also
creates standards of electronic transactions, such as the ANSI
5010 update and requires encryption and passwords on
websites that contain patient data.

10. Answer: D - The purpose of a compliance plan is to help your


office follow the correct coding and billing protocols. A
compliance plan is a document, or multiple documents, that
describes the HIPAA requirements that your office is required to
follow and uphold. Among other things, the compliance plan
outlines how often your office should conduct audits, how your
employees should be trained in regards to confidentiality, and
how you should select a compliance manager to ensure that all
components of the compliance plan are carried out.

11. Answer: A - HIPAA stands for Health Insurance Portability


and Accountability Act. HIPAA is an Act of Congress, not an
association or organization. Those that do not follow HIPAA
requirements can be prosecuted. HIPAA also joins with other
organizations to ensure that everyone involved in patient
healthcare follow its stipulations.

12. Answer: B - The purpose of provider credentialing is to make


sure that your provider is correctly licensed to perform
procedures. In the credentialing process, insurance companies
check a provider’s credentials, to make sure he is legal and
valid. This process is required as part of the provider’s contract
with the insurance company.

13. Answer: A - In the RBRVS calculation, the GPCI takes into


account the geographic location of a practice or provider. GPCI
stands for Geographic Practice Cost Index, and it takes into
account the relative price differences in geographical location.
The GPCI is a part of the RBRVS (Resource Based Relative
Value Scale), which calculates a reasonable fee for procedures.

14. Answer: C - The proper way to code is to identify the reason


for the encounter using the documentation for support. Then to
consult the Alphabetic Index, review stipulations for modifiers,
and confirm with the Tabular List. Using the Tabular List,
specificity can be coded at its highest level, based on
documentation. Assign the code, making sure that the
sequence is correct before submitting to the insurances.
Always remember to consult the Alphabetic List, reading all
options, cross references, abbreviations, and/or modifiers
before moving onto the Tabular List. The Tabular List will allow
the highest specificity by referring to the number of
characters required in the code and prompting this change
before sequencing and submitting to the insurance company for
payment.

15. Answer: A - Medically Unlikely Edits (MUEs) are considered


to be units of service edits created by CMS to lower the
Medicare Fee-For-Service Paid Claims Error Rate.

16. Answer: D - “Place of service” reporting refers to the location


of the setting where the patient is treated or the service is
provided. Evaluation and management codes can be specific to
the setting in which the patient is seen. Certain procedures or
services may be specific to a setting, such as therapeutic,
diagnostic or prophylactic. Facilities may also be specified
whether in the hospital, at home, in a physician’s office or in a
nursing home. Whatever location in which the encounter occurs
is the “place of service.”

17. Answer: C - Both A and B are correct. Services provided


within the home by someone part of an agency are considered
facility services. Services provided by a physician or provider
who is not part of an agency are not facility services, they are
termed “non-facility” services.

18. Answer: B - The CPT guidelines specify that terms such as


“physician,” “qualified healthcare professional,” and “individual”
are not used to exclude, instead it is meant that other entities
besides those specific ones can report services too, unless
specifically stipulated in a procedural guideline.

19. Answer: A - This is a federal law that prevents fraud and


abuse from happening when one provider may refer to another
specific provider and when that provider would then benefit from
anything of value. Physicians are not allowed to accept money
or other payment of value for recruitment or referrals of patients.
The law doesn’t protect providers from being charged with a
felony when this happens; instead it specifies the fines
associated with this felony. It doesn’t protect patients from being
referred out, only from physician’s receiving a kickback from the
referral.

20. Answer: B - These edits are updated quarterly and can be


found on the CMS website. The NCCI Edit Tables are updated
quarterly, not monthly, semi-annually or annually.

21. Answer: A - Mutually exclusive NCCI edits are code pairs that
are unlikely to be performed on the same patient on the same
day. When mutually exclusive procedures are submitted on the
same day’s claim, only the service of lesser value is
reimbursed. Mutually exclusive NCCI code pairs are those
unlikely to be performed on the same day on the same patient.
This could be two different tests that give the same results, or
two different procedures that usually aren’t performed at the
same time. Bundled services are considered
comprehensive/component edits instead of mutually exclusive
NCCI edits.

22. Answer: B - No, you should not report component codes


separately as they are only a “piece” of the service provided.
They should be billed as part of the comprehensive code or
“bundled” service to maximize reimbursement. Medicare payers
will typically only pay for the higher-
valued procedure if more than one bundled procedure is
submitted for the same patient on the same day.
Comprehensive codes are component codes that are billed
together “bundled”; they are broader codes. Component codes
are reimbursed at lower rates and, if reported in addition to the
comprehensive code, will not be reimbursed. A provider should
not unbundle comprehensive codes. Doing so will result in lower
reimbursement.

23. Answer: C - The correct answer is National Coverage


Determinations (NCDs) and Local Coverage Determinations
(LCDs). The Medicare Coverage Database (MCD) includes all
National Coverage Determinations (NCDs) and Local Coverage
Determinations (LCDs). This database also includes other
national coverage policy documents and resource guides.

24. Answer: A - NCD/LCDs specify the specific circumstances


necessary to consider a specific service reasonable and
necessary. The NCD/LCDs do not specify unreasonable or
unnecessary codes and are not all inclusive. Any codes part of
NCD/LCD can be reconsidered, so certain circumstance may be
covered. Local medical review policies (LMRPs) specify benefit
categories and exclusionary provisions.

25. Answer: C - There are four parts of each NCD. These include
Tracking Information, Description Information, Transmittal
Information, and National Coverage Analyses. Tracking
Information includes publication number, effective date and
version number of the policy. Description Information provides a
description of the service provided, limitations and indications,
as well as the benefit category. Transmittal Information includes
the transmittal number and history of revisions that apply to the
particular policy. National Coverage Analyses includes reference
information used to decide the policy as well as any other NCDs
that might be related that apply to the one in question. LCDs
have three parts, including LCD Information, Coding Information
and General Information.3

26. Answer: A - An ABN is a written document that must be given


to the beneficiary before furnishing items or services that are
usually covered by Medicare but that may not be covered in a
specific instance, such as lack of medical necessity. It allows the
beneficiary to make an informed decision about whether to
accept financial responsibility if Medicare doesn’t pay for the
service/item furnished. This document does not guarantee
coverage by Medicare, instead it is a means to protect the
provider’s office or institute from being financially liable for a
service or item that may not be paid for reasons, such as lack of
medical necessity. If the beneficiary is not given written notice,
they may not be held responsible for any fees not covered by
Medicare. Patients do not have to sign this form, but services
then do not have to be provided.

27. Answer: D - All of the above is the correct answer. Common


reasons for Medicare denial of services include the service or
item being experimental or investigational, the service or item is
not indicated for a particular diagnosis or treatment, and the
service or item isn’t considered safe and effective. An ABN
must be issued if there is a reasonable expectation of Medicare
not covering a service or item.
28. Answer: A - There are three different types of RVUs, including
RVUw, PE-RVU, and RVUm, that make up the Total Relative
Value Unit. Diagnosis and Work RVUs do not exist, so are not
part of what makes up the Total Relative Value Units. Facility
(PE-f) can be part of the Practice RVU.

29. Answer: C - Practice Relative Value Units (PE-RVUs) are


divided into two subcategories, including Facility (PE-f) and non-
facility (PE-nf). This differentiation eliminates site of service
differential, which in the past reduced PE-RVUs by 50% for
certain codes performed outside of the physician’s office. There
are no outpatient, interagency, inpatient, physician or non-
physician designations under Practice Relative Value Units (PE-
RVUs).
EVALUATION & MANAGEMENT (59 QUESTIONS) –
ANSWER KEY & RATIONALE

1. Answer: B - The OB should code for the procedure performed in


the office with 99215 to represent the evaluation and
management service. The physician spent over 40 minutes
counseling the patient on the decision for surgery (which is
indicated by modifier -57), therefore the level of E/M service can
be determined on this basis alone, and in this case it is a level 5
visit. Code 81025 also needs to be included on the claim to
indicate that the OB completed a confirmation urinalysis test for
pregnancy in the office. The last code that needs to be included
on the claim is 76801, for the abdominal ultrasound, which was
also provided in the office on the same day to confirm the
ectopic pregnancy status.

2. Answer: B - Dr. Johnson will bill the follow-up visit as an


established patient. New v. established rules state that if a
patient has not seen the doctor within three years, he is
considered a new patient, but in this case, Mr. Beck saw his
physician twice within a week. Although Mr. Beck did not
schedule any follow-up visits with Dr. Johnson for four year, he
did see Dr. Johnson that week in the ER, and as such he is
once again an established patient.

3. Answer: A - This statement is true. Under certain circumstances,


an evaluation and management service can be based solely on
the face-to-face time spent with the patient. There are seven
components of an E&M service with time being one of them.
E&M coding guidelines state that time shall be the key factor
when counseling dominates (more than 50%) of a patient
encounter.

4. Answer: B - When selecting an evaluation and management


code, the first thing that the coder needs to determine is the
appropriate category of E&M service. This means that the
coder needs to determine where the service was provided (i.e.,
hospital inpatient, office/outpatient, hospital observation, etc.).
Whether the patient was new or established would only be a
question in certain categories. In addition, the time spent with
the patient is only a determining factor in certain categories.

5. Answer: B - The correct E&M service is 99202. The patient was


new so the codes: 99212 and 99211 would be inappropriate.
Code 99201, for a new patient, requires a problem-focused
history assessment, problem-focused examination, and a
straightforward MDM, but in this case, the examination and
MDM levels are higher, allowing us to code for a higher level of
service. The correct E&M code for this case is 99202.

6. Answer: A - The correct procedure code is 99238 (Hospital


discharge day management; 30 minutes or less). Code 99239
is used when the physician spends 30 minutes or more on the
hospital discharge so that code is incorrect because the
discharge note states that the physician only spent 25 minutes
on the patient’s hospital discharge. The discharge note also
states that the discharge was for a patient in the hospital, so
code 99217 (Hospital observation patient) or 99315 (Nursing
home patient) would also not be appropriate.
7. Answer: D - The E&M service codes that would be reported for
this service are: 99222 (Initial Hospital Care Admission). This
code only needs to be reported once per admission. The next
code would be 99232 (X3) (Hospital Stay). This code is used to
report daily visits by the hospital physician. In this case, the
hospital stay would be reported as three units to represent the
three days (an extended amount of time) that the physician
spent caring for the patient. The next code would be 99231
(Inpatient Stay). This code would be reported as two units to
represent the last two days before discharge, at a less
complicated level of care, in response to an improved patient
condition. The last code would be 99238 (Discharge). This code
would be used to report the time it took to discharge the patient,
which in this case, took less than 30 minutes.

8. Answer: A - In order for a physician to appropriately code for a


consultation service, three things must be documented. These
three things are: the referral or request from the PCP, the
rendering of the opinion by the specialist or consultant, and the
written report or findings sent from the specialist to the PCP.
These three things can be easily remembered by the “Three
R’s:” “Referral to Specialist,” “Rendering of Service” and “Report
to PCP.”

9. Answer: C - The correct level of E&M service is 99283


(Emergency Department Visit, Level Three). Code 99291 is not
correct because it represents critical care services, not
emergency department services. The note states that both the
history assessment and examination were expanded problem-
focused, and that the MDM was of moderate complexity. In the
emergency services category, these three factors make up a
level three visit, which should be reported using the code
99283.
10. Answer: A - The correct evaluation and management code is
99342 (Home Visit For New Patient, Level Two). Codes 99348
and 99349 are not correct, because the patient is new to the
physician. Code 99343 is also not correct. Although the patient’s
history places her at the 99343 level, the examination and MDM
are at a lower level, which results in a code of 99342.

11. Answer: D - You would report this office visit with the E&M
codes: 99215 (Established Patient Office Visit, Level Five), and
99354 and 99355 (X2) (Prolonged Services). Code 99215 is
established by the detailed history, comprehensive examination,
and the MDM of high complexity. The appropriate prolonged
services codes are: 99354 (First 30-74 minutes Over), which
represents the time spent beyond what is allowed for the 99215
code and the 99355 (X2) (Rest of the Time Spent with the
Patient), which in this case is a total of 1 hour and 45 minutes. If
the physician spent 1 hour and 44 minutes, then the correct
number of units for 99355 would be 1, but that is not the case.

12. Answer: C - You should code this service using information


from the evaluation and management chapter of the CPT
manual. A pelvic examination without anesthesia should be
coded as part of the regular evaluation and management
service. This information can be found in the evaluation and
management chapter. If the patient needed a pelvic
examination under general anesthesia, it should be coded as a
service greater than an E&M service and reported using code
57410.
13. Answer: B - The OB/GYN should code for this visit with 99395
(Routine Physical and Pelvic Examination). OB/GYNs are
considered primary care physicians who can perform routine
physical examinations. Code 57170 (Diaphragm or Cervical
Cap Fitting with Instructions) also needs to be included on the
claim because the OB performed an additional service.
Furthermore, it is necessary to combine the modifier -59 with
code 57170 to indicate that there was a distinct procedural
service provided to the patient on the same day as an E/M
service, which in this case, is a routine physical and pelvic
exam.

14. Answer: B - The correct E&M code is 99386 (Periodic


Comprehensive Preventive Medicine, New Patient, 40-64 Years
of Age). The patient is new to the physician, so the codes:
99396 and 99397 are incorrect.99396 and 99397 are used for
established patients. Furthermore, code 99387 is used for
patients at least 65 years old.

15. Answer: B - The E&M service that can be reported for this
phone call is 99442 (Telephone E&M Service Provided by a
Qualified Non-Physician Health Care Professional; 11-20
Minutes of Medical Discussion). The phone call did not originate
from a previous visit (within the last 7 days) and it also did not
occur in response to an upcoming doctor’s visit (within the next
24 hours), therefore you can code for this service. In addition,
code 99442 is more appropriate than 98967 because a nurse
practitioner qualifies as a non-physician health care professional.

16. Answer: A - 99218 is the correct code for an initial


observation care of low complexity with a detailed history and
examination. Code 99219 is incorrect because this code is for
medical decision making of moderate complexity. Code 99220
is incorrect because this code is for initial observation care of a
patient requiring high complexity medical decision making.
Code 99224 is incorrect because this code is for subsequent
observation care and not for the initial encounter.

17. Answer: B - 99219 is the correct code for an initial


observation care requiring moderate complexity medical
decision making. Code 99218 is incorrect because this
observation patient required more than low complexity or
straightforward medical care. Code 99220 is incorrect because
the hospitalist did not spend enough time on the unit with this
patient initially and this patient did not require high complexity
medical decision making. Code 99224 is incorrect because this
code is for subsequent observation care and not the initial care.

18. Answer: D - 99226 is the correct code for subsequent


observation care of a patient requiring more than 35 minutes of
care requiring high complexity medical decision making with
detailed interval history and detailed examination since initial
observation care was given. Code 99220 is incorrect because
this code is for the initial observation care of a high complexity
medical decision making encounter. Code 99224 is incorrect
because, while it is the code for a subsequent observation
encounter, this is for a patient that is straightforward or of low
complexity. Code 99225 is incorrect because this code is for a
patient of moderate complexity typically only requiring 25
minutes of care at the bedside or on the patient’s unit.

19. Answer: D - 99239 is the correct code for hospital discharge


day management that took more than 30 minutes. Codes
99235 and 99236 are both incorrect because these codes are
for
observation or inpatient hospital care not involving discharge.
Code 99238 is incorrect because this code is for discharge
requiring less than 30 minutes of management.

20. Answer: C - 99244 is the correct code for an office


consultation involving 60 minutes of care, a comprehensive
history and physical examination and medical decision making
of moderate complexity. Code 99242 is incorrect because this
code is for an office consultation that requires straight forward
medical decision making and an expanded problem-focused
history and examination. Code 99243 is incorrect because this
code is for an office consultation involving a comprehensive
history and examination as well as moderate-complexity
medical decision making. Code 99245 is incorrect because this
office consultation code is for a visit that is usually 80 minutes,
requiring high-complexity medical decision making.

21. Answer: B - 99252is the correct code for an inpatient


consultation involving an expanded problem-focused history and
examination as well as straightforward medical decision making
that required approximately 40 minutes of time at the bedside or
on the hospital’s unit. Code 99251 is incorrect because this code
is for a problem-focused history and examination involving
straightforward medical decision making involving 20 minutes of
time. Code 99253 is incorrect because this code is for inpatient
consultation that requires 55 minutes of low complexity medical
decision making with detailed history and examination. Code
99254 is incorrect because this code is for inpatient consultation
with 80 minutes of time spent at the patient’s bedside or hospital
unit involving moderate-complexity medical decision making and
a comprehensive history and examination.

22. Answer: D - 99288 is the correct code for physician or other


qualified healthcare professional direction of EMS emergency
care or advanced life support. Code 99281 is incorrect because
this code is for an emergency department visit involving a
problem-focused history and examination that requires
straightforward medical decision making. Code 99284 is
incorrect because this code is for an emergency department
visit involving a detailed history and examination with moderate-
complexity medical decision making. Code 99285 is incorrect
because this code is for an emergency visit that requires a
comprehensive history and examination with high-complexity
medical decision making. All of these codes are for visits within
the emergency department, not one that is being completed
over two-way voice communication with the patient outside of
the hospital.

23. Answer: A - 99281 is the correct code for an emergency


department visit that requires a problem-focused history and
examination as well as straightforward medical decision
making. While all of these codes are used for emergency
department visits, 99282 is incorrect because this code
requires expanded problem-focused history and examination
as well as low- complexity medical decision making. Code
99283 is incorrect because this code requires an expanded
problem-focused history and examination with moderate-
complexity medical decision making. Code 99284 is incorrect
because this code requires a detailed history and examination
with moderate-complexity medical decision making.
24. Answer: C - 99291 is the correct code for 30-74 minutes of
evaluation and management of a patient in critical care. Codes
99285 and 99288 are incorrect because these codes are about
emergency room evaluation and management. Code 99292 is
incorrect because this code is for each additional 30 minutes as
an “add on” code for critical care evaluation and management.

25. Answer: D - 99291 is the correct code for the first 30-74
minutes of evaluation and management of a critical care patient.
99292 is also correct because this is the “add on” code for each
additional 30 minutes after the first 74 minutes. Code 99292
needs to be charged two times because this patient required
120 minute of care or two hours. Code 99291 is incorrect by
itself. Code 99292 is incorrect by itself and must be charged
however many times is needed for every 30 minutes after the
first 74 minutes.

26. Answer: D- 99291 and 99292 x 4 is the correct code


combination because this patient required 190 minutes of
critical care evaluation and management. Code 99291 is the
correct code for the first 30-74 minutes of critical care evaluation
and management. 99292 is also correct, but must be calculated
correctly to add each 30 minutes after the initial 74 minutes.

27. Answer: B - 99305 is the correct code for the initial nursing
facility care evaluation and management requiring a
comprehensive history and examination as well as moderate-
complexity medical decision making requiring 35 minutes of time
at the nursing facility or by the patient’s bedside. Code 99304 is
incorrect because this code is for a low-complexity or
straightforward medical decision making evaluation and
management of initial nursing facility care. Code 99306 is
incorrect because this code is for an initial evaluation and
management of a patient at a nursing facility that requires high-
complexity medical decision making. Code 99307 is incorrect
because this code is for subsequent visits and not the initial
evaluation and management visit to the nursing facility.

28. Answer: D - 99316 is the correct evaluation and management


CPT code for a discharge from a nursing facility that takes more
than 30 minutes. Code 99304 is incorrect because this code is
for an initial nursing facility care encounter. Code 99307 is
incorrect because this code is for a subsequent nursing facility
care encounter. Code 99315 is incorrect because, while this is a
discharge code from a nursing facility, the time involved with this
patient is not 30 minutes or fewer.

29. Answer: C - 99309 is the correct code for the care of this
patient within a nursing facility. A detailed interval history was
performed because the previous history was five days old, a
detailed examination was performed and the physician spent 25
minutes with this patient either at the bedside or on the patient’s
unit making up a plan of care for her. Code 99307 is incorrect
because this code is for a problem-focused interval history, a
problem-focused examination and straightforward medical
decision making, spending 10 minutes or fewer during the
encounter. Code 99308 is incorrect because this code is for an
expanded problem-focused interval history and examination as
well as low-complexity medical decision making, taking about 15
minutes of time. Code 99310 is incorrect because this code is
for a comprehensive history and examination
as well as high-complexity medical decision making typically
taking 35 minutes, which is longer than this physician spent
with this patient’s evaluation and management.

30. Answer: A - 99324 is the correct code for a new patient in a


domiciliary home where a problem- focused history and
examination is performed as well as straightforward decision
making. Code 99325 is incorrect because this is the code for a
low-complexity medical decision making encounter. Codes
99326 and 99327 are both incorrect because these codes are
for an evaluation and management that is of moderate
complexity.

31. Answer: D - 99340 is the correct code for this type of offsite
management of a patient in an assisted living facility taking
more than 30 minutes a month. Code 99324 is incorrect
because this code is for the initial assessment of someone in
an assisted living facility. Code 99334 is incorrect because
this code is for subsequent visits for an established patient.
Code 99339 is incorrect because, although this code’s
definition is correct, the required time is for 15-29 minutes and
this physician spends more than 30 minutes a month
managing the care of this patient.

32. Answer: C - 99336 is the correct code for an established


patient at an assisted living facility who requires a detailed
interval history, a detailed examination and moderate-complexity
medical decision making. Code 99334 is incorrect because this
code is for an encounter that is problem focused and involves
straightforward medical decision making. Code 99335 is
incorrect because this code is for an expanded problem-focused
history and examination as well as low-complexity medical
decision making. Code 99337 is incorrect because this code is
for a comprehensive history and examination and moderate-to-
high-complexity medical decision making.
33. Answer: C - 99343 is the correct code for a new patient home
visit that involves a detailed history and examination and
moderate-complexity medical decision making. Code 99341 is
incorrect because this new patient home visit included a
problem-focused history and examination as well as
straightforward medical decision making. Code 99342 is
incorrect because this visit involves a new home visit with an
expanded problem-focused history and examination and low-
complexity medical decision making. Code 99344 is incorrect
because this code for a new patient home visit involves a
comprehensive history and examination as well as moderate-
complexity medical decision making.

34. Answer: C - 99349 is the correct code for an established


patient home visit that involved a detailed examination and
moderate-complexity medical decision making. Code 99347 is
incorrect because this code is for an established patient but is
problem focused and involves straightforward medical decision
making. Code 99348 is incorrect because this code involves
low-complexity medical decision making. Code 99350 is
incorrect because this code involves moderate-to-high-
complexity medical decision making along with comprehensive
history and/or examination.

35. Answer: D - All of the above statements are correct about


prolonged service codes. They may be used for inpatient or
outpatient settings. They may include face-to-face and non-face-
to-face
encounters that are done on the same day, and they are
reported in addition to the initial evaluation and
management code.

36. Answer: C - 99358 is the correct code for prolonged service


without direct patient care. This physician spent 60 minutes
going through her previous records and talking with her previous
oncologist to provide the best care he can for this patient. Codes
99356 and 99357 are codes used for prolonged direct patient
care contact in the inpatient or observation setting. Code 99359
is the “add-on” code for prolonged service without direct patient
contact over the first 60 minutes. If this physician had provided
30 more minutes of care over the first 60, then this code would
be used in addition to 99358.

37. Answer: B - 99363 is the correct code for the initial 90 days of
Warfarin therapy with a minimum of eight INR measurements.
Code 99360 is incorrect because this code is used for standby
services. Code 99364 is incorrect because this code is the “add-
on” code for anticoagulant management services for every
subsequent 90 days, after the first which must include a
minimum of three INR measurements. Code 99366 is incorrect
because this code is regarding medical team conferences.

38. Answer: B - 99366 is the correct code for a medical team


conference lasting longer than 30 minutes where the patient
and/or family members are present. Code 99363 is incorrect
because this code is used for anticoagulant management and
there is no mention of anticoagulant therapy with this patient.
Codes 99367 and 99368 are incorrect because these codes are
for medical team conferences, but are codes used when patient
and/or family are not present.

39. Answer: C - To be considered a medical team conference, a


minimum of three healthcare professions from different
specialties or disciplines must participate. All of these
participates must actively be involved in the care of the patient
and in the development of the medical plan of action and all
must have had direct patient contact within the previous 60
days.

40. Answer: B - 99375 is the correct code for supervision of a


patient in a domiciliary home that involves more than 30
minutes of care. Code 99374 is incorrect because this code is
for supervision of patients in a domiciliary home, but is meant
for 15-29 minutes of time involvement. Codes 99377 and
99378 are incorrect because these codes are for supervision
of hospice patients.

41. Answer: C - 99377 is the correct code for the supervision of a


hospice patient that takes between 15-29 minutes per month.
Answers 99374 and 99375 are incorrect codes because these
codes are used for the supervision of patients in a home health,
domiciliary or equivalent environment. Code 99378 is incorrect
because, although this code is used for supervision of hospice
patients, it is meant for those patients that need more than 30
minutes of supervised care per month.

42. Answer: D- 99380 is the correct code for a nursing home


patient that requires more than 30 minutes of supervised care
per month. 99374 is incorrect because this code is for the
supervision of a patient in home health and not in a nursing
facility or nursing home. 99377 is incorrect because this code is
for the supervision of a hospice patient. Code 99379 is
incorrect because while it is a correct code for nursing home
supervision it is meant for 15-29 minutes of supervised care.

43. Answer: C - 99444 is the correct code for this type of an


online medical evaluation encounter because it does not
originate from a related E/M service provided within the
previous seven days. Codes 99441 and 99442 are codes
used for non-face-to-face telephone services. Code 99446 is
not correct because this code refers to interprofessional
telephone/internet consultations.

44. Answer :B - 99442 is the correct code for 11-20 minutes of


telephone services that are well documented in the chart and
that do not result in an E/M service within the next 24 hours or
soonest appointment and that does not relate to a previous E/M
service within one week prior to the telephone call. Code 99441
is incorrect because this code is for telephone calls that last 5-
10 minutes. Code 99443 is incorrect because this code is for
telephone services between 21-30 minutes. Code 99444 is
incorrect because this code is for online medical evaluation
instead of telephone services.

45. Answer: B - 99447 is the correct code for a consultation that


lasts 15 minutes and that does not result in immediate transfer
of care of other face-to-face service within the next 14 days.
Code 99446 is incorrect because this code is for consultation
time between 5-10 minutes. Code 99448 is incorrect because
this code is for consultation between 21-30 minutes. Code
99449 is incorrect because this consultation code is used for 31
minutes or more.
46. Answer: B - 99450 is the correct code for a basic life and/or
disability examination. Code 99449 is incorrect because this
code refers to interprofessional telephone consultations. Codes
99455 and 99456 are both used for work related or medical
disability examinations.

47. Answer: C - 99455 is the correct code for a work related or


medical disability evaluation performed by the treating
physician. Code 99444 is incorrect because this code is used
for online medical evaluations. Code 99450 is incorrect
because this code is for basic life and/or disability
examination. 99456 is incorrect because this code is for a
work related or medical disability examination by someone
other than the treating physician.

48. Answer: D - 99456 is the correct code for a work related or


medical disability examination performed by someone other
than the treating physician. Code 99449 is incorrect because
this code refers to interprofessional telephone consultations.
Code 99450 is incorrect because this code is for basic life
and/or disability examination and 99455 is incorrect because
this code is for an examination that is performed by the treating
physician.

49. Answer: A - 99460 is the correct code for the initial hospital or
birth center care for the evaluation and management of a normal
newborn infant. Code 99461 is incorrect because this is for the
initial evaluation and management of a normal newborn infant
seen somewhere other
than the hospital or birthing center. Code 99462 is incorrect
because this is for subsequent hospital care management.
Code 99463 is incorrect because this is the code for an infant in
the hospital or birthing center who is admitted and discharged
on the same date.

50. Answer: B- 99461 is the correct code for initial care of the
evaluation and management of normal newborn infants seen in
other than hospital or birthing center. Code 99460 is incorrect
because this code is for the infant initially seen at the hospital
or birthing center. Code 99462 is incorrect because this code is
for subsequent days and 99463 is incorrect because this code
is for infants admitted and discharged on the same day.

51. Answer: D - 99463 is the correct code for a normal newborn


infant who is admitted and discharged on the same date. Code
99460 is incorrect because this code is for the initial care of a
newborn in the hospital or birthing center. Code 99461 is
incorrect because this code is for the initial assessment not
within the hospital or birthing center. Code 99462 is incorrect
because this code is for subsequent hospital care of the normal
newborn infant.

52. Answer: A - 99466 is the correct code for interfacility transport


of a critical care patient under the age of 24 months. This code
is for the first 30-74 minutes of hands-on care during transport.
Code 99467 is incorrect because this code is an add-on code
for additional minutes after the initial 74 minutes for code 99466.
Code 99485 is incorrect because this is the code for supervision
by a physician via a two-way communication with the transport
team before transport, at the referring facility and during the
transport. This code is for the first 30 minutes of this type of
supervised care. Code 99486 is an add on code to 99485
indicating each additional 30 minutes of supervised time that is
provided.
53. Answer: C - 99471 is the correct code for initial inpatient
pediatric critical care of a critically ill infant or young child, 29
days through 24 months of age. Codes 99468 and 99469 are
incorrect because these codes are for infants 28 days or
younger. Code 99472 is incorrect because this code is for
subsequent inpatient pediatric critical care of a child 29 days
through 24 months of age.

54. Answer: D - 99476 is the correct code for subsequent days


for the inpatient pediatric critical care patient ages two through
five years. Codes 99468, 99471 and 9947599476 are all
incorrect codes because all of these codes are for initial
evaluation and management of various aged pediatric patients.

55. Answer: D - Complex chronic care management patients must


be considered to require moderate- or high-complexity medical
decision making, as well as at least 60 minutes of clinical staff
care management. Straightforward and low-complexity medical
decision making isn’t complicated enough to entitle these
patients to be considered eligible for using the evaluation and
management codes for complex chronic care management. At
least 60 minutes must be spent by a physician or other qualified
healthcare professional in clinical staff care management to use
these codes.
56. Answer: C - 99487 is the correct code for the initial 60 minutes
of clinical staff time directed by a physician within a calendar
month. Code 99489 is correct for each additional 30 minutes of
clinical staff time directed by a physician. This add-on code is
multiplied by the number of additional 30-minute increments
spent in this patient’s care. Code 99487 is incorrect by itself
because the time involved with the patient was more than 60
minutes. Code 99489 is incorrect by itself because this is an
“add-on” code and must be used in addition to another code.
Codes 99487, 99489 x 2 is incorrect because the additional 30-
minute increment would cause this code to be used for two
hours or more of clinical staff time directed by a physician.

57. Answer: B - A patient must have at least two chronic or


episodic health conditions that are expected to last at least 12
months, or until the death of the patient. One chronic condition
is not enough to qualify a patient for complex chronic care
management services. A patient may have more than two
conditions to qualify, but the minimum is two conditions.

58. Answer: D - Transitional care management services begin on


the day of discharge and continues for the following 29 days.
The timeframe for transitional care management has nothing to
do with the date admitted to the hospital. This timeframe ends
after 29 days, so subsequent time isn’t within the time
guidelines for transitional care management services.

59. Answer: A - 99495 is the correct code for transitional care


management services of a patient requiring at least moderate-
complexity medical decision making, who is contacted within
two days of discharge date by phone, direct contact or email
and who has a face-to-face appointment scheduled within 14
days of discharge. Code 99496 is incorrect because this code
requires face to face within seven days and also requires high-
complexity medical decision making. Codes 99497 and 99498
are both incorrect codes because these refer to advance care
planning.
HCPCS LEVEL II (14 QUESTIONS) – ANSWER KEY
& RATIONALE

1. Answer: D - HCPCS Level II ambulance service modifiers


represent where the patient was picked up and where the
patient was dropped off. They are two letters, like all other
HCPCS Level II modifiers, but the first letter represents where
the patient was picked up and the second letter represents
where the patient was transported to or dropped off. The
locations and letters are found at the beginning of the A-section
of the HCPCS manual.

2. Answer: D - When listing both CPT and HCPCS modifiers on a


claim, you list the CPT modifier first. When you report a
procedure code with more than one modifier, you must list the
modifier that will affect the payment first on the claim.
Typically, CPT modifiers will affect the payment of a claim, but
HCPCS modifiers may not.

3. Answer: B - When you append modifier -TC to a service, you are


indicating that the provider only performed the technical
component of the service. Some CPT codes have two
components to them, such as radiology services. The
professional component of the service is the supervision and
interpretation of the x-ray results, whereas the technical
component is actually taking the x- ray picture. Sometimes it is
necessary to indicate that the physician only performed one or
the other part of the service.

4. Answer: B-HCPCS modifiers can be used for CPT codes, but


CPT modifiers cannot be used to HCPCS codes. CPT
modifiers can only be used with CPT codes, whereas HCPCS
modifiers can be used with HCPCS and CPT codes.
5. Answer: C - The HCPCS Level II modifier -E1 stands for Upper
Right, Eyelid. Many HCPCS modifiers indicate anatomical
locations, such as the -E series modifiers for eyelids. Other
HCPCS modifiers indicate a section of the vertebral column,
and digits of the hands or feet.

6. Answer: B - The HCPCS manual includes codes for supplies,


services, and procedures that are not found in the CPT manual.
Although some procedures found in the HCPCS manual are
also found in the CPT manual, the most appropriate answer is B
because HCPCS codes also include supplies and services not
found in the CPT manual.

7. Answer: B - Vision and Hearing services are located in the


V0000 through V5999 section of the HCPCS manual.
Pathology and Laboratory services are located in the P0000
through P9999 section. Dental Procedures are located in the
D0000 through D9999 section. Transportation services are
located in the A0000 through A0999 section.

8. Answer: A - The best, most effective way to locate the correct


code for a service or supply in the HCPCS manual is to look in
the Index for the name of the service or supply, and it will direct
you to the correct code or range of codes. Checking the Index
first is the most efficient way of locating the correct code
because it lists all codes that may be applicable for that service
or supply. This way you can check all codes to make sure you
assign the correct one.
9. Answer: A - HCPCS J-Codes are used to represent drugs
administered by methods other than the oral method. The J-
codes are used to bill drugs administered to the patient, while in
the office. Other sections in the HCPCS manual represent
durable medical equipment and temporary national codes.
Dental procedures are not represented at all in the CPT manual,
and are reported with D-codes.

10. Answer: A - The abbreviation “VAR” stands for “Various


Routes.” Other routes of administration include: “IA” for “Intra-
arterial,” “IM” for “Intra-muscular,” “INH” for “Inhalant solution,”
and “OTH” for “Other routes of administration.”

11. Answer: A - HCPCS Level II codes are updated every quarter


by CMS (Centers for Medicare and Medicaid Services).
Updates to HCPCS Level II codes are published on the CMS
website at the beginning of each new quarter. The HCPCS
Level II manual, however, is only published once per year.

12. Answer: B - Appendix A in the HCPCS Level II manual


contains a table of drugs. This table lists all of the drugs in
alphabetical order and can be found in the HCPCS manual. The
listings are also organized according to the drugs administration
route and unit information.

13. Answer: C - The purpose of temporary national codes in the


HCPCS Level II manual is to allow the establishment of codes
prior to the January 1st annual update. Temporary codes are
also developed in order to meet the needs of newly
established policies and legal requirements. If the temporary
code becomes permanent, it is deleted and becomes a
permanent code.

14. Answer: C - J0696 x 2 is the correct code because this code


is for Ceftriaxone sodium 250 mg. Because this patient
received two times this amount of medication, this code needs
to reflect this multiplication. Code J0715 is incorrect because
this code is for 500 mg Ceftizoxime sodium. J0696 is incorrect
unless doubled, because this code is only for 250 mg of
Ceftriaxone sodium. Code J0697 is incorrect because this code
is for 750 mg Cefuroxime sodium. All of these codes can be
used for IM or IV administration.
ICD-10-CM VOLUMES 1 & 2 (32 QUESTIONS) –
ANSWER KEY & RATIONALE

1. Answer: B - The “X” placeholder is used in certain codes to


indicate future expansion. The “X” must be used where a
placeholder exists to allow that code to be considered valid.
Certain codes need further expansion and are incomplete
without the “X” placeholder. There are no 8th characters. ICD-
10-CM codes only go up to the 7th character, in some instances.

2. Answer: B - “Other” and “other specified” codes should only be


used when a code does not exist for the detail that is provided in
the medical record. “Other” codes can be found in the Tabular
list and are listed as “NEC” or “not elsewhere classified” in the
Alphabetic Index entry. “Other” codes should only be used when
a code truly does not exist. If another code exists for that
particular diagnosis or sign/symptom then that code should be
utilized. The Alphabetic Index and Tabular List should both be
utilized to identify proper codes or absence of a code.

3. Answer: A - A combination code is a single code used to classify


two diagnoses. It can be used only after reviewing all inclusion
and exclusion criteria and when the combination code fully
identifies the diagnostic conditions listed in the medical record.
Multiple codes should not be used when a combination code
exists that identifies all of the elements documented. However,
when a combination code lacks the necessary specificity, an
additional code such as a sign/symptom code may be used in
addition to the combination code, although the combination code
should always be used as the primary code.

4. Answer: C - “Borderline” codes do exist for some diagnoses and


must be confirmed using the notations in the ICD-10-CM coding
book. These codes may only be used if the provider specifically
states this in the medical record. If “borderline” is unclear in the
documentation and there are any questions about the diagnosis
in the documentation, then the coder should query the provider
for clarification.

5. Answer :B - Severe sepsis is only coded as the secondary code


and never as the primary. This is due to the fact that there is
always an underlying infection in sepsis and that systemic
infection needs to be coded first. Sepsis does have
subcategories and it is important to use a subcategory when the
cause is known and only when it is clearly documented in the
medical record. Severe sepsis can never be assigned as the
primary/principal diagnosis. If severe sepsis develops during an
encounter, than the admitting or initial diagnosis is used as the
primary code and severe sepsis is listed as the secondary code
along with the underlying systemic infection.

6. Answer: D - Pain codes can be used independently or with


other codes as appropriate. Pain codes can be used as the
primary or secondary code depending upon why the patient is
being encountered. With ICD-10-CM coding, if laterality is
listed in the medical record, then the appropriate laterality
code should be used.

7. Answer: B - It is important to always refer to the Alphabetic


Index and Tabular List when obtaining the correct codes.
There are different codes for the types of glaucoma and some
of these codes do have laterality specified. The stages will
need to be identified by using the
appropriate seventh character and the Tabular List will aid in the
correct code identification. If it is documented that different types
of glaucoma are in different eyes, then the coding needs to
reflect this and not just reflect the most severe. Stages are
identified by the seventh character.

8. Answer: C - No, there is no national requirement for


mandatory ICD-10-CM external cause reporting, however
certain states may require mandatory reporting and certain
payers may require this as well. When not a mandatory
requirement, providers are still encouraged to voluntarily
report these codes when able.

9. Answer: B - Z codes are not procedure codes and only indicate


the reason for an encounter. They may be used as either
primary or secondary codes and cannot be used independently.
These codes are used to identify health factors influencing
health status and contact with health services for things such as
contact/exposures, vaccinations, patient status codes,
screenings, observation, aftercare, follow up, and counseling.

10. Answer: A - Conditions in the patient’s medical history do not


need to be coded unless they require or affect that patient’s
treatment care plan. Only those codes that directly apply to the
encounter should be utilized. However there are “history
codes” that can be used to indicate previous medical history
that directly impacts the current diagnoses and treatment plan.

11. Answer: C - A04.7, B20 is the correct answer because code


A04.7 is the primary code for enterocolitis due to Clostridium
difficile, which is the primary reason for admission. The HIV is
subsequent in this particular case because it is not the main
reason the patient was being admitted. However, it is still
necessary to consider for best plan of care and treatment of the
patient. B20 is not the primary code so should not be listed first.
R75 is the code for inconclusive serologic evidence of HIV which
is incorrect for this patient since this patient has been diagnosed
with HIV in the past.

12. Answer: C - D64.81, C91.00, T45.1X5A is the correct code


sequence for a patient who was admitted for anemia requiring
a blood transfusion due to the chemotherapy, from the
treatment he is undergoing for acute lymphoblastic leukemia.
The anemia code is the primary code and is specific to the
chemotherapy. The secondary code is the type of leukemia
that he has; he is currently undergoing treatment, so he is not
in remission. The last code is for an adverse effect of
antineoplastic and immunosuppressive drugs, initial encounter.
C91.10 is incorrect because this is not the right code for acute
lymphoblastic leukemia.

13. Answer: A- E10.10 is the correct code for Type 1 diabetes


mellitus with ketoacidosis without coma. E10.319 is the correct
code for Type 1 diabetes mellitus with unspecified diabetic
retinopathy without macular edema. Code Z79.4 is the code
used for patients who routinely use insulin and this patient has
an insulin pump. Code E10.11 is incorrect because this code is
for Type 1 diabetes mellitus with ketoacidosis with coma. Code
E10.311 is incorrect because this code is for Type 1 diabetes
mellitus with unspecified diabetic retinopathy with macular
edema.
14. Answer: B - F33.1, F41.1, F17.210 is the correct ICD code
sequence for this patient. F33.1 is the correct code for moderate
recurrent major depressive disorder. F41.1 is the correct code
for generalized anxiety disorder and F17.210 is the correct code
for nicotine dependency by cigarettes without complications.
F33.0 is incorrect because this code is for mild recurrent major
depressive disorder. F41.9 is incorrect because this code is for
unspecified anxiety disorder. F17.200 is incorrect because this
code is for unspecified uncomplicated nicotine dependence and
this patient’s nicotine of choice is cigarettes.

15. Answer: D - G44.229 is the correct code for chronic tension


type headache that is not intractable. Code G43.10 is incorrect
because this code is for migraine with aura and G44.019 is
incorrect because this code is for episodic cluster headaches.
Code G44.201 is incorrect because this code is for unspecified
tension-type headaches that are intractable.

16. Answer: A - H00.11 is the correct code for a chalazion of the


right upper eyelid. Code H00.12 is incorrect because this code
is for a chalazion of the right lower eyelid. Code H00.13 is
incorrect because this is the code for a chalazion of the right
eye without the eyelid specified. Code H00.14 is incorrect
because this code is for a chalazion of the left upper eyelid.

17. Answer: C - H65.01, H61.22 is the correct sequence for this


patient. H65.01 is the correct code for right acute serous otitis
media. H61.22 is the correct code for impacted cerumen in the
left ear. Code H65.02 is incorrect because this code is for left
acute serous otitis media. Code H65.03 is incorrect as well
because this code is for bilateral acute serous otitis media. Code
H61.21 is incorrect because this code is for cerumen impaction
in the right ear.
18. Answer: D - J45.21, J00 is the correct sequence of codes for
this patient because J45.21 is the correct code for mild
intermittent asthma with acute exacerbation and J00 is the
correct code for acute nasopharyngitis, also known as the
common cold. Code J45.20 is incorrect because this code is for
uncomplicated mild intermittent asthma. Code J45.31 is
incorrect because this code is for mild persistent asthma with
acute exacerbation and J22 is incorrect because this code is for
unspecified lower respiratory infection.

19. Answer: C - K50.8 is the correct code for Crohn’s disease of


both small and large intestines without complications. Code
K50.0 is incorrect because this code is specific to the small
intestine and K50.1 is incorrect because this code is for disease
of the large intestine. Code K50.9 is incorrect because this code
is for unspecified Crohn’s disease.

20. Answer: B - L01.01 is the correct code for non-bullous


impetigo. Code L01.00 is incorrect because this is for
impetigo that hasn’t been specified. Code L01.02 is incorrect
because this code is for Bockhart’s impetigo, also known as
perifolliculitis, and code L01.03 is incorrect because this is for
bullous impetigo.

21. Answer: C - M43.6 is the correct code for torticollis. This


code excludes certain types of torticollis, but the patient
described above doesn’t fit any of the exclusion criteria listed.
Code M43.2 is incorrect because this code is for fusion of the
spine. Code M43.5 is incorrect because
this is the code for recurrent vertebral dislocation and M43.8
is incorrect because this is the code for other specified
deforming dorsopathies.

22. Answer: A - N20.0 is the correct code for calculus of the


kidney. Code N20.1 is incorrect because this code is for
calculus of the ureter. Code N20.2 is incorrect because this is
the code for calculus of kidney and calculus of ureter. N21.0 is
incorrect because this code is for calculus in the bladder.

23. Answer: A - O24.410 is the correct code for gestational


diabetes in pregnancy that is controlled by diet. Code
O24.414O24.41 is incorrect because this code is for
gestational diabetes that requires insulin. O24.419 is incorrect
because this is for unspecified control of gestational diabetes.
Code O24.42 is incorrect because this code is for gestational
diabetes in childbirth.

24. Answer: A - P04.41, P05.18 is the correct sequence of codes.


Code P04.41 is correct because the patient is suspected to be
affected by maternal use of cocaine. Code P05.18 is correct for
newborn small for gestational age at 2000-2499 grams. Code
P04.49 is incorrect because this code is for “other drugs of
addiction.” Code P05.17 is incorrect because ,while it is a code
for newborn who is small for gestational age, it is for newborns
weighting 1750-1999 grams.

25. Answer: B - Q21.0 is the correct code for ventricular septal


defect, also known as Roger’s disease. Code Q20.2 is incorrect
because this is the code for a double outlet left ventricle. Code
Q21.1 is incorrect because this code is for an atrial septal
defect and Q22.0 is incorrect because this code is for
pulmonary valve atresia.
26. Answer: D - J18.9 is the correct code for pneumonia with
unspecified organism. Code R06.02 is the code specifically for
shortness of breath. Code J13 is incorrect because the
organism is unknown that is causing the pneumonia. Code
R06.00 is incorrect as well because this is for unspecified
dyspnea and it is specified that patient has significant
shortness of breath.

27. Answer: B - Codes R73.01 and R78.0 are the correct codes
for impaired fasting glucose and for finding of alcohol in the
blood. Code R73.0 is incorrect because this code is for
abnormal glucose, but there is a subcategory for impaired
fasting glucose. Code R73.02 is incorrect because this is for
impaired oral glucose tolerance and R78.1 is incorrect because
this code is for finding opiates in the blood.

28. Answer: D - A62.04A, V80.010A, Y93.52 is the correct ICD-


10-CM code sequence for this patient encounter. Code A62.04A
is the correct code for initial encounter of right wrist fracture of
the scaphoid bone. Code V80.010A is the correct code for the
initial encounter of animal-rider injured by fall from or being
thrown from a horse in a non-collisional accident. Code Y93.52
is the correct code for the activity of horseback riding. The
important distinction between all of the codes listed above is the
seventh character designation. “A” is for initial encounter, “D” is
for subsequent encounter, and “S” is for sequela. This patient
came to the ER for an initial encounter so subsequent or
sequela encounters are incorrect.
29. Answer: B - Z34.82, Z3A.16 is the correct ICD-10-CM code
sequence for this patient encounter. Z34.82 is the correct code
for a second trimester encounter for supervision of “other”
normal pregnancy and this patient was specified to be in her
second pregnancy. Code Z3A.16 is the correct code to indicate
the number of weeks gestation that patient is being supervised
at. Code Z34.02 is incorrect because this code is for a normal
first pregnancy encounter. Code Z3A.15 is incorrect because
this code is for an encounter at 15-weeks’ gestation. Code
Z34.01 is also incorrect because this code is for a normal first
pregnancy in the first trimester.

30. Answer: C - S60.141A is the correct code for initial encounter


of a contusion of the right ring finger with damage to the nail.
Code S60.111A is incorrect because this code is for initial
encounter of contusion of the right thumb with damage to the
nail. Code S60.121D is incorrect because this code is for
subsequent encounter of right index finger with damage to the
nail. Code S60.141D is incorrect because this code is for
subsequent encounter of contusion of right ring finger (4th digit)
with damage to the nail.

31. Answer: A - 96365 is the correct code for an intravenous


infusion for therapy, prophylaxis or diagnosis for up to one-hour
infusion. J0690 is the correct code for ceftriaxone sodium.
Code 96366 is incorrect because this is the add-on code for
each additional hour after the first hour. Code 96367 is also
incorrect because this is an add-on code for each sequential
infusion of a new drug/substance over one hour. Code 96368
is incorrect because this code is the add-on code for a
concurrent infusion.

32. Answer: A - 96360 is the correct code for initial intravenous


hydration infusion for 31 minutes to one hour in duration. This
code can be used for pre-packaged fluids and electrolytes.
Code 96361 is incorrect because this code is for each additional
hour of infusion. Codes 96365 and 96366 are incorrect because
these codes are for intravenous infusions for therapy,
prophylaxis or diagnosis.
LABORATORY & PATHOLOGY (54 QUESTIONS) –
ANSWER KEY & RATIONALE

1. Answer: A - The correct code for the CBC only is with 85025
(Blood Count; Complete, Automated, and Automated Differential
WBC Count). This code needs to be indicated twice, as the test
was performed twice and the modifier -91 needs to be appended
to the second code to indicate that it was a repeat laboratory
test. This code and modifier indicate that the same test was
repeated in the office, as medically necessary and is not a
mistake. Both tests are reimbursable.

2. Answer: C - The correct codes for this laboratory service are:


82040, 82247, 82374, 82435, 82565, 82947, 84075, 84132,
84155, 84295, 84460, 84450 and 84520. These are all of the
chemistry tests included as part of a comprehensive metabolic
panel, minus the total calcium test. If the physician was able to
get a sample of the calcium, the service would be reportable
with the full panel code 80053, but because the calcium was
not taken, each test must be reported individually.

3. Answer: A - Suspecting that the patient had been drinking and


driving, the physician ordered a qualitative alcohol pathology
test. After determining that the patient did indeed have alcohol in
their system, the physician then ordered a quantitative
pathology test to check the amount of alcohol in the
bloodstream. A qualitative pathology test checks only for the
presence of a specific substance, in this case alcohol. After the
qualitative results come in positive, a quantitative test checks for
the actual amount of the substance in the blood stream (the
actual quantity of the substance).

4. Answer: D - The correct code for this laboratory service is 80100,


for the testing of all three drug classes using one
chromatographic test. Code 80102 also needs to be included to
indicate that a confirmation test was performed on one of the
drug classes, which tested positive. Code 80101 is only
appropriate if the laboratory equipment indicated that only one
drug class was tested at a time. If this is the case, code 80101
should be reported with three units.

5. Answer: D-The correct OB only code is 81025 (Urine


Pregnancy Test by Visual Comparison Methods). Codes 81002
and 81003 should not be included on the claim because they
are used for in-office urinalyses with results that indicate
something other than pregnancy such as: protein or white
blood cells in the urine. Code 81025 is the appropriate code for
the urine pregnancy test because it includes the urine sample
and indicates a pregnancy.

6. Answer: B - The name of the regulations, which were passed in


1988, to ensure quality standards for all laboratory testing, are
the Clinical Laboratory Improvement Amendments (CLIA).
These tests fall into three categories: waived tests, tests of
moderate complexity, and tests of high complexity. Any lab or
clinic providing any of the three types of tests must have a CLIA
number. In addition, the bills for the laboratory tests must
include the CLIA number of the provider completing the test.
7. Answer: B - Code 83015 (Chemistry Screening for Heavy Metal
(e.g., Arsenic, Barium, Beryllium, Bismuth, Antimony, Mercury
Screen) is the most appropriate in this case. Code 83018 is not
correct because it used when a qualitative test is performed for
each heavy metal detected in the patient. Code 80050 is also
incorrect because it is used for a general health panel, not a
screening for heavy metals.

8. Answer: C - The surgical pathologist should code for this service


with 88305 (with 4 units). Code 88305refers to the four separate
biopsies performed during the surgical session. The three
vaginal biopsies should be coded separately because they
account for the three separate samples. Furthermore, the
vaginal biopsies fall within the Level IV surgical pathology
section. The cervical biopsy should also be coded with a Level
IV surgical pathology code because it also falls within that
category. If the sample was a conization of the cervix, it should
have fallen within the Level V surgical pathology category and
coded with 88307.

9. Answer: D - Level 1 studies in the surgical pathology section of


the Laboratory and Pathology chapter of the CPT manual
describe gross examination only. A gross examination is an
inspection of the entire surgical specimen without dividing it
into blocks for microscopic examination. A gross examination
is performed by the naked eye. Level 1 surgical pathology
codes are the only codes that include a gross examination;
the other five levels all include a microscopic examination at
increasingly complicated levels.

10. Answer: D - The pathologist should code for this service


using code 88014 (Necropsy, Gross Examination Only; Stillborn
or Newborn with Brain). Code 88025 is not correct because it
refers to gross and microscopic examinations, whereas, in this
case, the pathologist only performed a gross examination.
Codes 88005 and 88012 are also incorrect because they do not
refer to the examination of a stillborn infant.

11. Answer: D - The surgical pathologist should code for this


service using codes 88304 with 3 units, to indicate the sampling
of three separate skin tags. Code 88309 with 4 units should be
used to indicate the four separate samples of breast tissue.
Level VI codes (88309) are more appropriate than Level V
codes (88307) because the procedure was a mastectomy with
removal of regional lymph nodes. If the procedure had been a
partial or simple mastectomy, then code 88307 (Level
V) would have been the correct surgical pathology code.

12. Answer: B - 80050 is the correct code for a general health


panel and, because the general health panel includes the
comprehensive metabolic panel, this panel is not coded in
addition to the 80050. Code 80053 is incorrect because this
code is for the comprehensive metabolic panel, but because
another panel that is more extensive that includes the
comprehensive metabolic panel was also ordered, this code
overlaps with the general health panel code 80050. Code 80051
is incorrect because this code is for an electrolyte panel, which
is part of the comprehensive metabolic panel and part of the
general health panel. Code 80048 is also incorrect because this
is a basic metabolic panel, which is included within the
comprehensive metabolic panel.
13. Answer: A - 80300 is the correct code for direct optical
observation drug testing of any number of drug classes from
Drug Class A. Because no additional testing was performed,
there is no need for additional codes. Code 80301 is incorrect
because this is for a single drug class using instrumented tests
systems. Code 80302 is incorrect because this code is for Drug
Class List B and 80303 is incorrect because this code is used
for any number of drug class screens using chromatography
procedures and not direct optical observation.

14. Answer: A - 80162 is the correct code for a total Digoxin


level. Code 80163 is incorrect because this code is for a free
Digoxin level. Code 80159 is incorrect because this code is for
the therapeutic drug assay for clozapine and code 80178 is the
code for a lithium therapeutic drug assay.

15. Answer: C - 80185 and 80186 are the correct codes for total
and free Phenytoin, and code 80201 is the correct code for
Topiramate therapeutic drug assay tests. Code 80184 is
incorrect because this code is used for the drug Phenobarbital.
Code 80183 is incorrect because this code is for the drug
Oxcarbazepine and code 80200 is incorrect because this code is
for the drug Tobramycin.

16. Answer: B - 80194 is the correct code for Quinidine


therapeutic drug assay testing. Code 80190 is incorrect
because this code is for Procainamide. Code 80195 is incorrect
because this code is for Sirolimus and 80197 is incorrect
because this code is for the drug Tacrolimus.

17. Answer: B - 80400, 82533 x 2 is the correct code for the


ACTH stimulation panel test because the particular analyte
(Cortisol) is used with the number of times it is performed during
the test. Code 84143 is incorrect because this code is for the
particular analyte 17 hydroxypregnenolone.
18. Answer: A - 80438, 84443 x 3 are the correct codes for the
laboratory component of the TRH stimulation panel. 80438 is
the correct code for the one-hour version of this test and 84443
x 3 is the correct code for 3 TSHs to be performed in this panel.
Code 80439 is incorrect because this code is for the 2-hour
version of this test.

19. Answer: C - 80434, 82533 x 5, 82947 x 5 is the correct code


for this test. Code 80434 is the correct code for the insulin
tolerance panel for ACTH insufficiency. Code 82533 is the
correct code for Cortisol, which is tested five times, and 82947
is the correct code for Glucose, which is tested five times in this
panel. Code 80432 is incorrect because this code is for the
insulin induced C-peptide suppression panel.

20. Answer: C - 80502 is the correct code for a comprehensive


consultation with pathology, including review of patient’s history
and medical records. Code 80439 is incorrect because this code
is for a 2-hour thyrotropin releasing hormone (TRH) stimulation
panel. Code 80500 is incorrect because this code is for a limited
clinical pathology consultation and 81000 is incorrect because
this code is for a urinalysis.
21. Answer: A - 80500 is the correct code for a limited clinical
pathology consultation. Code 80502 is incorrect because this
code is for a comprehensive consultation. Codes 81000 and
81005 are incorrect because both of the codes refer to urinalysis
testing.

22. Answer: C - 80500 is the correct code for limited clinical


pathology consultation. Codes 80400 and 80402 are both
incorrect because these codes reference
evocative/suppression testing codes. Code 80502 is incorrect
because this code is for a comprehensive clinical pathology
consultation including review of the patient’s history and
medical records.

23. Answer: C - 81002 is the correct code for a non-automated,


without microscopy urinalysis. Code 81000 is incorrect because
this code is for a non-automated urinalysis with microscopy.
81001 is incorrect because this code is for an automated
urinalysis with microscopy. Code 81003 is incorrect because this
code is for automated urinalysis without microscopy.

24. Answer: D - 81025 is the correct code for a urine pregnancy


test done by using visual color comparison methods. Codes
81000, 81005, and 81015 are all incorrect because they all refer
to codes for urinalysis instead of a pregnancy test.

25. Answer: C - 81214 is the correct code for BRCA1 gene


analysis with full sequence analysis and common
duplication/deletion variants. Code 81211 is incorrect because
this code is for both BRCA1 and BRCA2 gene analysis. Code
81212 is incorrect because this code is for both BRCA1 and
BRCA2 with additional specific variants. Code 81216 is
incorrect because this code is for BRCA2 only.
26. Answer: B - 81225 is the correct code for the CYP2C19 gene
analysis test. Code 81220 is incorrect because this code is for
CFTR gene analysis. Code 81226 is incorrect because this
code is for CYP2D6 gene analysis and code 81227 is incorrect
because this code is for CYP2C9 gene analysis.

27. Answer: D - 81511 is the correct code for the test for fetal
congenital abnormalities including AFP, uE3, hCG, and DIA.
Code 81508 is incorrect because this code is for the testing of
the biochemical assay of two proteins (PAPP-A, hCG). Code
81509 is incorrect because it is also testing for proteins, but
involves three proteins instead of just the two above. This code
tests for PAPP-A, hCG, and DIA). Code 81510 is incorrect
because this code is for the testing of three analytes, including
AFP, uE3, hCG, but does not include the fourth analyte listed
above, the DIA.

28. Answer: B - 81506 is the correct code for an endocrinology


(type 2 diabetes) biochemical assay of seven analytes. Code
81504 is incorrect because this code refers to oncology testing.
Code 81507 is incorrect because this code is for a fetal
aneuploidy DNA sequence analysis. Code 81508 is incorrect
because this code is assessing for specific fetal congenital
abnormalities.

29. Answer: B - 81502 is the correct code for ovarian oncology


biochemical assay of five proteins, including CA-125,
apolipoprotein A1, beta-2 microglobulin, transferring, and pre-
albumin. Code 81500 is incorrect because, although this code
is for ovarian oncology, it is only assessing two
proteins, which include CA-125 and HE4. Code 81504 is
incorrect because this code is for oncology of a specific tissue
of origin, including a microarray gene expressing profiling.
Code 81506 is incorrect because this code is specific for an
endocrinology biochemical assay.

30. Answer: D - 82106 is the correct code for AFP test using
amniotic fluid. Codes 82103 and 82104 are both incorrect
because these codes are for alpha-1-antitrypsin tests. Code
82105 is incorrect because this code is for AFP using serum
instead of amniotic fluid.

31. Answer: A - 82607 is the correct code for the lab test
cyanocobalamin (Vitamin B-12). Code 82608 is incorrect
because, although this is a test for Vitamin B-12, it is using the
unsaturated binding capacity. Code 82610 is incorrect because
this code is for Cystatin C and code 82615 is incorrect
because this code is for cysteine and homocystine using urine.

32. Answer: D - 85247 is the correct code for factor VIII, von
Willebrand factor, multimetric analysis. Code 85210 is incorrect
because this code is for clotting factor II, prothrombin specific.
Code 85245 is incorrect because this is for factor VIII, von
Willebrand factor ristocetin cofactor and 85246 is incorrect
because this is specific for factor VIII, von Willebrand factor
antigen. It is important to know the specific test ordered. In this
case the physician had ordered the multimetric analysis of factor
VIII von Willebrand factor.

33. Answer: B - 85175 is the correct code for whole blood dilution
clot lysis time. Code 85170 is incorrect because this code is for
clot retraction. Code 85210 is incorrect because this code is for
prothrombin clotting factor II. Code 85300 is incorrect because
this code is for clotting inhibitors or anticoagulants for
antithrombin III activity.
34. Answer: C - 85652 is the correct code for automated
erythrocyte sedimentation rate. Code 85635 is incorrect
because this code is for the reptilase test. 85651 is incorrect
because this code is for a non-automated erythrocyte
sedimentation rate. Code 85660 is incorrect because this is for
sickling of red blood cells.

35. Answer: B - 86141 is the correct code for high sensitivity C-


reactive protein. Code 86140 is incorrect because this code is
for C-reactive protein, but not the high sensitivity version.
Code 86146 is incorrect because this code is for Beta 2
Glycoprotein I antibody and code 86147 is incorrect because
this code is for Cardiolipin (phospholipid) antibody.

36. Answer: D - 86341 is the correct code for Islet cell antibody.
Code 86336 is incorrect because this code is for Inhibin A.
Code 86337 is incorrect because this code is for insulin
antibodies and 86340 is incorrect because this code is for
intrinsic factor antibodies.

37. Answer: C - 86361 is the correct code for absolute CD4 T-


cell count. Code 86359 is incorrect because this is the code for
total T-cell count. Code 86360 is incorrect because this is for
the absolute CD4 and CD8 T-cell count including ratio. Code
86367 is incorrect because this code is for stem cell total count.
38. Answer: B - 86900 is the correct code for ABO serologic blood
typing. Code 86890 is incorrect because this code is for
autologous blood or component collection processing and
storage. Code 86901 is incorrect because this is for Rh typing
and code 86902 is incorrect because this is antigen testing of
donor blood using reagent serum.

39. Answer: B - 86885 is the correct code for indirect qualitative


antihuman globulin test (Coombs test) for each reagent red cell.
Code 86880 is incorrect because this code is for the direct
antihuman globulin test (Coombs test). Code 86886 is incorrect
because this code is for the indirect testing of the antihuman
globulin test (Coombs test), each antibody titer. Code 86890 is
incorrect because this code is for autologous blood or
component collection processing and storage.

40. Answer: C - 86985 is the correct code for splitting of blood or


blood products for each unit. If more than one unit was needed
to be split, then this code would be multiplied by however many
units were required for the patient. Code 86965 is incorrect
because this code is for pooling of platelets or other blood
products. Code 86975 is incorrect because this code is for
pretreatment of serum for use in RBC antibody identification
using incubation with drugs. Code 86999 is incorrect because
this code is for unlisted transfusion medicine procedure, but
there was a specific code available for the splitting of blood or
blood products.

41. Answer: B - 87045 is the correct code for aerobic stool culture
with isolation and preliminary examination for Salmonella and
Shigella species. Code 87040 is incorrect because this code is
for blood bacterial culture. Code 87046 is incorrect because,
even though this is an aerobic stool culture, it is for additional
pathogens with identification of isolates. Code 87070 is incorrect
because this code is for culture of any other source except for
urine, blood or stool.

42. Answer: C - 87177 is the correct code for direct smear for
concentration and identification of ova and parasites. Code
87169 is incorrect because this code is for macroscopic
examination for parasites. Code 87172 is incorrect because this
is for a pinworm exam. Code 87181 is incorrect because this
code is for an antimicrobial susceptibility study.

43. Answer: B - 87110 is the correct code for a chlamydia culture


from any source. Code 87109 is incorrect because this code is
for a culture for mycoplasma. Code 87116 is incorrect because
this code is for tubercle or other acid-fast bacilli culture with
isolation and presumptive identification of isolates. Code 87118
is incorrect because this code is for mycobacterial culture with
definitive identification.

44. Answer: C - 88007 is the correct code for a necropsy


(autopsy) with gross examination only with the brain and spinal
cord. Code 88000 is incorrect because this code is for necropsy
without central nervous system Code 88005 is incorrect
because this code is for necropsy with the brain but not with the
spinal cord. Code 88012 is incorrect because this code is for
necropsy of stillborn or newborn with brain examination.
45. Answer: C - 88028 is the correct code for gross and
microscopic examination of necropsy of an infant with brain
examination. Code 88012 is incorrect because this is the code
for a necropsy with only gross examination. 88020 is incorrect
because this code is for a gross and microscopic necropsy
without central nervous system examination. Code 88029 is
incorrect because this code is for the examination of a stillborn
or newborn with brain examination.

46. Answer: A - 88104 is the correct code for cytopathology of


fluids, washings or brushings except cervical or vaginal
sampling of smears with interpretation. Code 88106 is incorrect
because this is for cytopathology of fluids, washings or
brushings except-instead of using the smear method- this is
using a simple filter method. Code 88108 is incorrect because
this code is using a concentration technique and code 88112 is
incorrect because it is using a selective cellular enhancement
technique.

47. Answer: A - 88125 is the correct code for forensic


cytopathology on sources like sperm. Codes 88130 and 88140
are incorrect because both of these codes are for sex chromatin
identification. Code 88141 is incorrect because this code is for
cervical or vaginal cytopathology.

48. Answer A - 88182 is the correct code for flow cytometry of the
cell cycle or DNA analysis. Code 88184 is incorrect because
this is the code for flow cytometry of the cell surface,
cytoplasmic or nuclear marker involving only the first marker.
Code 88185 is the add-on code for additional markers to be
tested. Code 88187 is incorrect because this is for interpretation
flow cytometry of two to eight markers.

49. Answer: A - 88245 is the correct code for chromosome


analysis for breakage syndromes for baseline SCE of 20-25
cells. Code 88248 is incorrect because this code is for baseline
breakage, core 50-100 cells, count 20 cells, and 2 karyotypes.
Code 88249 is incorrect because this code is for score 100
cells, clastogen stress. Code 88261 is incorrect because this
code is for chromosome analysis with count 5 cells, 1 karyotype
with banding.

50. Answer: C - 88291 is the correct code for the interpretation


and report of cytogenetics and molecular cytogenetics. Code
88240 is incorrect because this code is for cryopreservation.
Code 88241 is incorrect because this is for thawing and
expansion of frozen cells. Code 88299 is incorrect because this
code is for an unlisted cytogenetic study.

51. Answer: C - 88235 is the correct code for tissue culture for
non-neoplastic disorders using amniotic fluid or chorionic villus
cells. Code 88230 is incorrect because this code is for tissue
culture of non-neoplastic disorders with lymphocyte and code
88233 is incorrect because this is the code to use for skin or
other solid tissue biopsy for tissue culture. Code 88237 is
incorrect because this code is for tissue culture of neoplastic
disorders using bone marrow or blood cells.

52. Answer: D - 89322 is the correct code for semen analysis


including volume, count, motility and differential using strict
morphologic criteria (e.g., Kruger). Code 89300 is incorrect
because this code is for semen analysis for the presence and/or
motility of sperm including Huhner test (post coital). Code 89320
is incorrect because this code is for semen analysis including
volume, count,
motility and differential but isn’t using specific criteria for
morphology. Code 89321 is incorrect because this is the code
for semen analysis with sperm present and motility of sperm if
performed.

53. Answer: B - 89337 is the correct code for cryopreservation of


mature oocytes. This code does not include the code for
storage. Code 89335 is incorrect because this is
cryopreservation of testicular reproductive tissue. Code 89352 is
incorrect because this code is for thawing of cryopreserved
embryos and 89398 is incorrect because this is the code for
unlisted reproductive medicine laboratory procedure.

54. Answer: B - 89268 is the correct code for insemination of


oocytes. Code 89264 is incorrect because this code is for sperm
identification from testis tissue, either fresh or cryopreserved.
Code 89272 is incorrect because this code is for extended
culture of oocytes or embryos. Code 89280 is incorrect because
this code is for assisted oocyte fertilization using the
microtechnique for less than or equal to 10 oocytes.
MEDICAL TERMINOLOGY (33 QUESTIONS) – ANSWER
KEY & RATIONALE

1. Answer: D - The term “Cervix Uteri” does not refer to a fallopian


tube. The cervix uteri is the neck of the uterus, located below the
uterus. This is where the cervix is located. The cervix is the
organ that tapers down from the corpus uteri (uterus) to connect
to the vagina. The terms “oviducts,” “uterine tubes,” and
“salpinges,” are all additional terms used to describe the
fallopian tubes.

2. Answer: D - In an episiotomy, the obstetrician will incise the


perineum. The perineum is the area between the vaginal
opening and the anus. This procedure is usually performed to
allow a larger area for the baby to pass through during delivery,
and is not considered a separate procedure during delivery. It is
only reportable as a separate service, if it is performed by any
physician other than the attending OB.

3. Answer: B - A cervical conization takes a cone of cervical


tissue for biopsy. A loop-electrode excision procedure (LEEP)
is also a procedure performed for a cervical biopsy, but it is
performed with a small metal loop, which cuts away at the
tissue and electro-cauterizes it. A Papanicolaou smear (also
referred to as a Pap smear) is a laboratory test performed to
see if there are molecular changes on the cervix, which could
indicate cancer.

4. Answer: B - The three major components of an intestinal


transplant are: the living donor enterectomy, preparation of the
intestine, and transplantation with enterectomy. As with all major
organ transplants, there are three distinct phases of work that
need to be performed. The first is the removal of the organ from
a living or cadaver donor. The second is the preparation of the
organ for transplant (backbench work). The third is the
transplantation of the organ into the recipient, which can be
performed with or without recipient enterectomy.

5. Answer: A - The thymus is the gland that produces thymosin


and stimulates T-cells to produce hormones to control immune
system function. The thyroid controls the rate that the body
uses energy and the parathyroid glands maintain the body’s
calcium levels to allow for the proper functioning of the nervous
and muscular systems. Among other things, the adrenal gland
secretes epinephrine and norepinephrine into the bloodstream.

6. Answer: A - The anterior segment of the eyeball, directly behind


the cornea, is filled with a clear salty fluid called the aqueous
humor. Light from the aqueous humor enters a convex disc
suspended behind the iris, which is called the crystalline lens.
After the light passes through the crystalline lens, it enters the
vitreous humor, which is the jelly-like substance that fills the
posterior chamber of the eyeball.

7. Answer: B - The system that produces tears is called the


lacrimal system. This is a system of glands that produce tears
behind the eyebrows, which flow into the eyes and drain out
through the lacrimal punta, into the nose. A common ailment of
babies is lacrimal ductal stenosis, a condition that occurs when
the lacrimal ducts become clogged, causing a buildup in the
inner corners of the eyes.
8. Answer: C - The procedure known as blepharoplasty is
performed to plastic repair a droopy eyelid. The prefix
“blephar/o” means “eyelid,” and as such, a “blepharoplasty”
would be performed to repair the skin of the eyelid. A
blepharoplasty is commonly performed to correct a droopy
eyelid, a condition also known as ptosis. Codes for
blepharoplasty can be found in both the Eye and Ocular Adnexa
subsection as well as the Integumentary subsection of the
surgery Chapter in the CPT manual.

9. Answer: A - The external ear contains the auricle and external


auditory canal, which leads up to the tympanic membrane. The
tympanic membrane separates the outer ear canal from the
middle and inner ear. The middle ear includes the incus,
stapes, and malleus. The inner ear contains the cochlea,
semicircular canals and Eustachian tube. This is where sounds
waves are converted into nerve impulses, which are read by
the brain.

10. Answer: C - This procedure is referred to as a mastoidectomy.


A mastoidectomy is the removal of the mastoid bone. This
procedure may also be performed to make room for a cochlear
implant, which is inserted during a cochlear implantation
procedure. Ossicular surgeries are performed to repair the
ossicular chain or to correct defects in the oval or round
windows in the inner ear. A retrobulbar injection in administered
to the back of the eyeball or the retrobulbar area.

11. Answer: B - This organ is called the pharynx. The esophagus


is the tube that arises from the pharynx, the organ that carries
food through the diaphragm into the stomach. The trachea is not
a digestive organ; it is the respiratory organ that connects the
nose to the mouth and the mouth to the lungs.
12. Answer: A - The condition that occurs when the lining of the
esophagus becomes inflamed and is generally caused by an
infection or irritation of the esophagus is called esophagitis. In
medical terminology, the suffix “itis" means inflammation so
esophagitis literally means “inflammation of the esophagus.”

13. Answer: C - A dilated and enlarged varicose vein that


developed inside the rectum and slipped outside of the anus is
called a prolapsed hemorrhoid. An external hemorrhoid is one
that occurs on the outside of the rectum, while an internal
hemorrhoid is one that occurs inside the rectum and is still
inside the rectum. A prolapsed hemorrhoid has slipped from the
inside of the rectum to the outside.

14. Answer: A - The pair of tubular glands located above the


prostate and behind the bladder that lubricate the duct system,
nourish the sperm, and contribute fluid to the ejaculate is
called seminal vesicles. The testes produce and store sperm
cells. The vas deferens transports semen from the epididymis
to the pelvis. The epididymis is a coiled tube that connects the
testicles to the vas deferens.
15. Answer: B - One of the most common prostatic disorders is
benign prostatic hyperplasia (BPH), which is an enlargement of
the prostate gland. This disorder may require a transurethral
resection of the prostate (TURP).Benign prostatic hyperplasia is
caused by the excessive growth of prostatic nodules. BPH can
compress the urethra, leading to partial or complete obstruction
of the urethra, urinary hesitancy, frequency, dysuria, urinary
retention and an increased risk of urinary tract infections.

16. Answer: B - The classification that should be used for a


vulvectomy with the removal of skin and deep subcutaneous
tissues is a radical vulvectomy. A simple vulvectomy is the
removal of skin and superficial subcutaneous tissue not deep
subcutaneous tissues. A partial vulvectomy is the removal of
less than 80% of the vulvar area, and a complete vulvectomy is
the removal of more than 80% of the vulvar area.

17. Answer: B - The muscular tube that carries urine from the
kidneys to the bladder is called the ureter. The human body
has two ureters, one coming from each kidney. The muscles in
the ureters constantly tighten and relax to force urine down
from the renal pelvis toward the bladder. The urethra is the
tube that connects the urinary bladder to the outside of the
body.

18. Answer: D - The most common voiding disorder is urinary


incontinence. Voiding disorders are common conditions that
affect urination, therefore fecal incontinence is not a voiding
disorder. Types of urinary incontinence include: urge, stress,
overflow, functional, and mixed incontinence. Other voiding
disorders include: bladder neck obstruction, bladder
diverticulum, neurogenic bladder, and detrusor instability.

19. Answer: D - The spleen, which is part of the hemic system,


does not create stem-cells. Stem cells are created by the bone
marrow. The bone marrow also produces red and white blood
cells, and stem-cells. Stem-cells are special cells that can
develop into many different cell types. The cells are created in
the bone marrow.

20. Answer: C - A blood disease that is characterized by an


abnormal increase in the amount of white blood cells in the body
is normally referred to as leukemia. Leukemia is the most
common disease of the blood and bone marrow. The term
leukemia is a broad term for a cancer of the bone marrow, which
results in an increase of white blood cells. Lymphoma is a
cancer in the lymph system, which can spread to the bone
marrow.

21. Answer: B - The term “erythrocyte” means “red cell.” This is a


common term for the red blood cells that carry oxygen and
carbon dioxide throughout the body. The prefix “erythr/o” means
“red” and the suffix “-cyte” means “cell.” A leukocyte is a white
blood cell, a granulocyte is a white blood cell that has a granular
appearance, and a thrombocyte is a blood cell fragment that
does not contain a nucleus. Thrombocytes can form a thrombus
(blood clot).

22. Answer: C - The lymphatic system contains for organs: the


spleen, tonsils, Peyer’s patches, and thymus gland. The spleen
is a part of the hemic system, which creates and stores red
blood cells. The tonsils are an initial line of defense against
bacteria entering through the throat. Peyer’s
patches are located in the small intestine and help prevent
bacteria from infecting the intestines. The thymus gland
produces T-cells, which are used in the immune system and
also aid in auto-immunity or keeping the body from attacking
itself.

23. Answer: D - The primary function of the lymph nodes is to filter


lymph fluid to remove bacteria and viruses, as well as other
harmful materials. Lymph nodes are aggregated in the neck,
under the arms, and in the groin. They are also located
throughout the body. They function together to filter harmful
organisms out of the body, which is a major function of the
immune system.

24. Answer: B - The small muscles under the dermis that serve to
help the hair follicles stand on end when the body is chilled are
called arrector pili muscles. Arrector pili muscles are small
muscles under the skin that are attached to the end of hair
follicles in humans. Contraction of these muscles causes the
hair follicle to stand on end, creating goose bumps when the
body is chilled.

25. Answer: B - The Rule of Nines calculates the extent of burn on


the human body. In order to classify the extent of a burn on the
human body, the body is theoretically divided up into sections
with each composing 9% of the body. The Rule of Nines is
adjusted according to the age of the patient and the
percentages change when describing the extent of burns for
children and adolescents. The Rule of Nines is commonly used
in integumentary section codes when describing the extents of
burns. The coder must be able to assign the correct code, by
taking into account the extent of burn on the patient’s body.

26. Answer: C - The tibialis anterior muscle controls the frontal


flexion of the lower leg. The term “tibialis” refers to the tibia
muscle that is located in the lower leg. The term “anterior”
refers to the front of the body.

27. Answer: D - The thoracic plexus is not a nerve plexus found


in the human body. Although there are thoracic spinal nerves in
the body, there is no intersection of thoracic nerves culminating
in a thoracic plexus. The cervical plexus serves the head, neck,
and shoulders. The lumbar plexus serves the back, abdomen,
groin, thighs, knees and calves. The sacral plexus serves the
pelvis, buttocks, genitals, thighs, calves, and feet.

28. Answer: B - The difference between meningitis and


encephalitis is that meningitis is the inflammation of the lining of
the brain and encephalitis is the inflammation of the brain itself.
Myelitis is the inflammation of the spinal cord, and
encephalomyelitis is a combination of the inflammation of the
brain and spinal cord. Generally speaking, when coding for any
of the above conditions, the coder should report the organism
responsible for the inflammation first and the inflammation
second.

29. Answer: B - If a patient receives a chest x-ray with a


posteroanterior projection, the beams are traveling from the
back of the patient to the front of the patient. The term
“posteroanterior” literally means “from the back to the front,”
while the term “post” means “back” and “anterior” means “front.”
30. Answer: D - The three main ways that contrast material can
be introduced are intravascularly, intra-articulary, or
intrathecally. Intravascular contrast material is injected into a
vein. Intra- articular contrast material is injected into a joint.
Intrathecal contrast material is injected within a sheath (within
subarachnoid/spinal fluid).

31. Answer: A - A radiographic image of the colon’s interior is


referred to as a colonography. A colonoscopy occurs when
an endoscope is passed into the colon for visualization,
while a duodenoscopy occurs when the endoscope is
passed from the colon into the duodenum. A
cholangiography is a radiographic image of the bile duct.

32. Answer: A - A bronchoscopy is a procedure that occurs when


an endoscope is passed through the oropharynx and vocal
chords and extended beyond the trachea into the right bronchus.
According to the rules of endoscopies, the coder should report
the farthest extent reached by the endoscope. In this case, the
farthest extent was the bronchus, and so the endoscopy was a
bronchoscopy. If the endoscope had only been extended to the
larynx or trachea, the procedure would have been considered a
laryngoscopy or tracheoscopy.

33. Answer: C - The purpose of a mediastinotomy is to provide


an opening for access to the mediastinum. Any procedure that
removes a portion of a body part is considered an -ectomy. An
-otomy procedure, as in “mediastinotomy,” is a procedure that
is performed to create an opening into a body cavity to provide
access to that cavity.
MEDICINE (85 QUESTIONS) – ANSWER KEY &
RATIONALE

1. Answer: C - The correct codes for this visit are: 99393, 90471,
90700, 90472 (X4), 90713, 90707, 90716 and 90658. The
correct E&M code for the well-child exam is 99393 (Periodic
Comprehensive Preventive Medicine; Late Childhood (age 5
through 11 years)). The correct administration codes for the
vaccines are: 90471 (Immunization Administration) and 90472
(X4) (Immunization Administration; Each Additional Vaccine)
because the vaccines were not provided with counseling. The
correct influenza vaccine is 90658 (Influenza Virus Vaccine,
Trivalent, Split Virus, When Administered to Individuals 3 Years
of Age and Older) because code 90657 is used for individuals
under 3 years of age and code 90660 is used for intranasal use.

2. Answer: A - The correct code for these vaccines are: 90460,


90461 (X2), 90700, 90460, 90713, 90460, 90461 (X2), 90707,
90460, 90716, 90460 and 90658. Code 90460 (Immunization
Administration…with Counseling by the Physician; First
Component of Each Vaccine Administered) refers to the first
components of each vaccine. Five vaccines were administered,
so 90460 must be coded to account for the first components of
each vaccine. In other words, five 90460 codes need to be
reported. Additionally, there are two vaccines that have three
components, so 90461(Immunization Administration…with
Counseling by the Physician; Each Additional Vaccine
Component) needs to be listed for each additional component
for the multiple component vaccines, therefore four 90461
codes need to be listed.

3. Answer: B - The correct code for the patient’s dialysis care is


90951 (ESRD-Related Services Monthly, for Patients Younger
than 2 Years of Age to Include Monitoring of the Adequacy of
Nutrition, Assessment of Growth and Development, and
Counseling of Parents; with 4 or More Face-to-Face Visits by
the Physician Per Month).This code is used because it includes
4 or more visits by the physician. Code 90951 only needs to be
reported once.

4. Answer: C - The correct code for the patient’s dialysis care is


90967 (X14) (ESRD-related Services for Dialysis Less than an
Full Month of Service, Per Day; for Patients Younger than 2
Years of Ages).Code 90967 (X14) is used because the patient
only received dialysis services from June 1 to June 14. In
addition, code 90967 (X14) needs to be reported with 14 units,
in reference to the 14 individual days of service.

5. Answer: D - The correct code for the procedure is 65222


(Removal of Foreign Body, External Eye; Corneal, with Slit
Lamp). Code 65220 is inappropriate because it does not include
the use of the slit lamp, and codes: 65205 and 65210 are
inappropriate because they do not include the removal of a
foreign body from the cornea. Furthermore, codes: 65205 and
65210 do not include the use of a slit lamp. A slit lamp is a small
lamp that consists of a high-intensity light source. This lamp can
be used to aid surgeons in the removal of small foreign bodies.

6. Answer: A - The physician should code the procedure using the


code 66852 only (Removal of Lens Material; Pars Plana
Approach, With or Without Vitrectomy). Codes: 66983 and
66982 are used for the removal and implantation of a lens, but
this procedure does not include the
implantation of a lens, only the removal. Code 66840 is also
inappropriate because it is used to report the removal of the
lens via an aspiration technique.

7. Answer: B - The correct codes for this procedure are: 67312


(Correction of the Two Horizontal Muscles), 67314 (Correction of
One Vertical Muscle), and67320 (Transposition Procedure to
Correct the Superior Oblique Muscle). Code 67318 is also
incorrect. Although code 67320 is used for the correction of the
superior oblique muscle, it does not include the use of the
transposition procedure. Modifier -51 is also not necessary on
code 67320, as it is an add-on code, and add-on codes are
modifier-51 exempt.

8. Answer: C - The correct code for the procedure is 67820-RT


(Correction of Trichiasis; Epilation, by Forceps Only). Code
67825 is incorrect because it is used to report any method
other than forceps. Code 67830 is incorrect because it refers to
the incision of the lid margin. Modifier -RT is the correct
modifier, in this case, because it is used to acknowledge that
the procedure was performed on the right side only. Modifier
-50 is incorrect because it is used to indicate a bilateral
procedure.

9. Answer: A - The correct code for Mr. Longoria’s ophthalmology


service today is with 92002, (Ophthalmological Services;
Medical Examination and Evaluation with Initiation of
Diagnostic and Treatment Program; Intermediate, New Patient.
Codes 92012 and 92014 are incorrect because they are used
for established patients. In this case, the patient is new to this
physician and the correct level of evaluation is intermediate
because the patient has an established (known) cataract. The
patient’s cataract is not newly diagnosed. Furthermore, the
patient did not receive a comprehensive examination.
10. Answer: A - You would code the procedure with 69436-50
(Tympanostomy: Requiring Insertion of Ventilating Tube,
General Anesthesia).You would also use modifier -50, which
would indicate that the procedure was performed bilaterally, on
each ear. Code 69433 would not be appropriate because it is
used for a tympanostomy that was performed under local or
topical anesthesia, rather than general anesthesia. Code 69020
(Drainage External Auditory Canal, Abscess) would also need
to be reported. Code 11000 would be inappropriate for the
drainage of the abscess because it is generally used for the
integumentary system, rather than the external ear canal.

11. Answer: D - You would code this procedure with 69930-50


(Bilateral Cochlea Device Implantation, With or Without
Mastoidectomy).You would use this code because it refers to
the cochlear device implantation procedure and it includes the
mastoidectomy. The two procedures do not need to be reported
separately. Code 69900 also needs to be included on the report
to indicate that the physician used an operating microscope to
aid in the procedure.

12. Answer: C - The correct code for the cardiologist’s office is


93041 (Rhythm ECG, 1-3; Tracing Only Without Interpretation
and Report).The interpretation of the ECG was performed by
the patient’s primary care physician, therefore the cardiologist
can only bill for the technical component of the ECG, which in
this case is the tracing only. The primary care physician’s office,
on the other hand, can only bill for the interpretation of the
report, because the PCP only interpreted the test results
and diagnosed the patient.

13. Answer: D - The electrocardiogram should be reported with


the code 93040 (Rhythm ECG, 1-3 Leads; with Interpretation
and Report). Codes 93000 and 93010 should not be reported
because the ECG was a rhythm ECG, not a routine ECG. Code
93042 (Rhythm ECG, 1-3 Leads; Interpretation and Report
Only) also should not be reported because the physician
performed the ECG, interpreted and reported the results of the
ECG, instead of just interpreting the results.

14. Answer: D - The correct code for this service is 93288


(Interrogation Device Evaluation with Analysis Review and
Report; Single, Dual, or Multiple Lead Pacemaker
System).Code 33208 is used for the actual insertion of the
pacemaker, while code 33213 is used for the insertion of
pacemaker pulse generator only. Code 93289 is used for the
analysis of a cardioverter- defibrillator system not a
pacemaker system.

15. Answer: C - The correct codes for this service are 93453
(Combined Right and Left Heart Catheterization including
Intraprocedural Injections(s) for Left Ventriculography, Imaging
Supervision and Interpretation, when Performed) and 93464
(Physiologic Exercise Study (e.g. Bicycle or Arm
Ergometry).Code 93531 is only reported when the combined
catheterization is performed for congenital cardiac anomalies
and codes 93451 and 93452 are only used for either a right or
left heart catheterization.

16. Answer: A - The correct codes for this office visit are:
99214 (E&M service), 94060 (Bronchodilation
Responsiveness, Spirometry), 94010 (Pre and Post
Bronchodilator Administration), 94760 (Pulse Oximetry
Reading) and 94640 (Nebulizer Treatment). Codes A7015,
A4616, and J7630 are nebulizer treatment supply codes.
Code 94010 is incorrect because it does not include both
the before and after nebulizer treatment spirometry.

17. Answer: D - The testes and ovaries, pineal gland, and pituitary
gland are missing from the endocrine section in the CPT manual
(codes 60000 to 60699). Codes for the testes and ovaries are
located in the male and female genital systems sections of the
CPT manual and codes for the pineal and pituitary glands are
located in the nervous system section of the CPT manual, which
is where the glands’ within the brain are located.

18. Answer: C - The correct for the endocrinology service located


in the medicine section that allows for the ambulatory continuous
glucose monitoring of interstitial tissue fluid (includes the
physician’s reading and interpretation of the report’s findings
over a 72-hour period) is 95251. Code 95250 is used for the
above procedure, but just for the recording of the findings
without interpretation and report of the findings by a physician. It
is stated that code 99091 should not be used with either 95250
or 95251.

19. Answer: B - The correct codes for the infusion service are:
96360 (Intravenous Infusion, Hydration; Initial, 31 Minutes to 1
Hour) and 96361 (X2) (Intravenous Infusion, Hydration; Each
Additional Hour). The total time spent on the infusion was 3
hours 15 minutes. Code 96360
accounts for the first hour, while code 96361 (X2) accounts for
the next two hours. The last 15 minutes is not reported because,
according to CPT guidelines, code 96361 is only used to report
hydration infusion intervals, greater than 30 minutes beyond the
1 hour increments.

20. Answer: A - No telephone consultation service code should


be reported because the call resulted in an appointment the
next day. The code 98967 (Telephone Assessment and
Management Services Provided by a Qualified Non-Physician
HealthCare Professional to an Established Patient, Parent, or
Guardian not Originating from a Related Assessment and
Management Service Provided within the Previous 7 Days nor
Leading to an Assessment and Management Service or
Procedure within the Next 24 Hours or Soonest Available
Appointment).In this case, the 11-20 minutes of medical
discussion should not be reported because the telephone
conversation resulted in an appointment the next morning.

21. Answer: A - The additional code that should be appended to


the E&M service or any other services provided at the visit is
99050 (Services Provided in the Office at Time Other than
Regularly Scheduled Office Hours, or Days When The Office
is Normally Closed, in Addition to Basic Services). 99050 is
the most appropriate code because the office stayed open late
to accommodate the patient, at a time when the office would
normally be closed.

22. Answer: D - 90460, 90672 is the correct code sequence for


this vaccine. Code 90460 is the correct code for immunization
administration through 18 years of age by any route, with
counseling from the physician. Code 90672 is the correct code
for quadrivalent live, intranasal influenza vaccine. Code 90660
is incorrect because this code is for the live trivalent intranasal
influenza vaccine. Code 90471 is incorrect because this code is
for immunization administration of greater than 19 years old via
percutaneous, intradermal, subcutaneous, or intramuscular
injection. Code 90473 is incorrect because this code is for
immunization administration by intranasal or oral route, but is
not specific to pediatrics.

23. Answer: B - 90791 is the correct code for a psychiatric


diagnostic evaluation. Code 90785 is incorrect because this
code is an add-on code, indicating interactive complexity. Code
90792 is incorrect because this is psychiatric diagnostic
evaluation with medical services. Code 90832 is incorrect
because this code is for psychotherapy.

24. Answer: B - 90839, 90840 is the correct code for


psychotherapy for crisis. Code 90839 is the correct code for
the first 60 minutes, and 90840 is the correct code for each
additional 30 minutes. This patient was seen for 70 minutes.
Had the patient been seen for 90 minutes, the add-on code
would have been multiplied based on each additional 30
minutes.

25. Answer: C - 90880 is the correct code for hypnotherapy. Code


90870 is incorrect because this code is for electroconvulsive
therapy. Code 90875 is incorrect because this code is for
individual psychophysiology therapy with biofeedback. Code
90885 is incorrect because this code is for psychiatric evaluation
of hospital records and other psychiatric reports.
26. Answer: C - 90901 is the correct code for biofeedback training
by any modality. Codes 90875 and 90876 are incorrect because
these codes are for psychophysiological therapy. Code 90911 is
incorrect because this code is specific for EMG and/or
manometry of the perineal muscles, anorectal or urethral
sphincter.

27. Answer: D - 90911 is the correct code for anorectal


biofeedback training. Codes 90875 and 90876 are incorrect
because these codes are for psychophysiological therapy.
Code 90901 is incorrect because this is the code for
biofeedback training by any modality.

28. Answer: A - 90901 is the correct code for biofeedback


training by any modality. Code 90911 is incorrect because this
is the code for EMG and/or manometry biofeedback training for
perineal muscles, anorectal or urethral sphincter. Codes 90875
and 90876 are incorrect because these codes are for
psychophysiological therapy incorporating biofeedback.

29. Answer: A - 90935 is the correct code for hemodialysis


procedure with single evaluation by a physician or other
qualified healthcare professional. Code 90937 is incorrect
because this is the code for hemodialysis requiring repeated
evaluations. Code 90940 is incorrect because this is the code
for a hemodialysis access flow study and 90945 is incorrect
because this code refers to dialysis procedure other than
hemodialysis.

30. Answer: C - 91020 is the correct code for a gastric motility


(manometric) study. Code 91010 is an incorrect code because
this is the code for an esophageal motility study. Code 91013 is
an add- on code for the esophageal motility study with
stimulation or perfusion. Code 91022 is incorrect because this
code is for duodenal manometric (motility) study.
31. Answer: B - 91030 is the correct code for the Esophagus acid
perfusion (Bernstein) test for esophagitis. Code 91022 is
incorrect because this code is for a duodenal motility study.
Codes 91034 and 91035 are incorrect codes because these are
used to test for gastroesophageal reflux testing.

32. Answer: B - 91110 is the correct code for gastrointestinal tract


imaging, intraluminal (capsule endoscopy) of the esophagus
through the ileum with interpretation and report. Code 91040 is
incorrect because this code is for esophageal balloon distension
provocation study. Code 91111 is incorrect because this is the
code for capsule endoscopy of just the esophagus. Code 91112
is incorrect because this is for gastrointestinal transit and
pressure measurement of the stomach through the colon.

33. Answer: C - 92612 is the correct code for flexible fiberoptic


endoscopic evaluation of swallowing by cine or video recording.
Code 92610 is incorrect because this is the code for the
evaluation of oral and pharyngeal swallowing function. Code
92611 is incorrect because this is the code for motion
fluoroscopic evaluation of swallowing function by cine or video
recording. Code 92614 is incorrect because this is the code for
flexible fiberoptic endoscopic evaluation of laryngeal sensory
testing by cine or video recording.
34. Answer: C - 93990 is the correct code for a duplex scan of
hemodialysis access including arterial inflow, body of access
and venous outflow. Code 93970 is incorrect because this code
is for duplex scan of extremity veins. Code 93975 is incorrect
because this is for a duplex scan of arterial inflow and venous
outflow of abdominal, pelvic, scrotal contents and/or
retroperitoneal organs. Code 93998 is incorrect because this
code is for unlisted noninvasive vascular diagnostic study.

35. Answer: A - 93880 is the correct code for a complete


bilateral duplex scan of the extracranial arteries. Code 93882
is incorrect because this code is for a unilateral or limited
study. Codes 93886 and 93888 refer to transcranial Doppler
studies.

36. Answer: B - 93978 is the correct code for a complete duplex


scan of aorta, inferior vena cava, iliac vasculature, or bypass
grafts. Code 93975 is incorrect because this code is for a
duplex scan of arterial inflow and venous outflow of abdominal,
pelvic, scrotal contents and/or retroperitoneal organs. 93979 is
the incorrect code for this procedure because this code is for a
unilateral or limited study instead of a complete study. Code
93980 is incorrect because this code is for a duplex scan of
arterial inflow and venous outflow of penile vessels.

37. Answer: B - 94620 is the correct code for a simple pulmonary


stress test. Code 94610 is incorrect because this code is for
intrapulmonary surfactant administration through endotracheal
tube. Code 94621 is incorrect because this is for a complex
pulmonary stress test. Code 94640 is incorrect because this is
for a pressurized or nonpressurized inhalation treatment for
acute airway obstruction.

38. Answer: C - 94660 is the correct code for initiation and


management of CPAP ventilation. Codes 94644 and 94645 are
incorrect because these codes refer to continuous inhalation
treatment with aerosol medication. Code 94662 is incorrect
because this code is for continuous negative pressure
ventilation initiation and management.

39. Answer: A - 95004 is the correct code for percutaneous tests


with allergenic extracts. Code 95017 is incorrect because this
code is for combination testing, including percutaneous and
intracutaneous tests with venoms. Code 92018 is incorrect
because this is a combination test of percutaneous and
intracutaneous testing with drugs or biological. Code 95024 is
incorrect because this test is specific for intracutaneous tests
with allergenic extracts.

40. Answer: B - 95076, 95079 is the correct code for three hours
of ingestion challenge testing. Code 95076 is the correct code
for ingestion challenge test for the first 120 minutes of testing.
Code 95079 is correct as well because this is an add-on code
for each additional 60 minutes of testing. This patient had three
hours of testing performed, so the add-on code only has to be
used once for a total of 180 minutes.

41. Answer: C - 95130 is the correct code for allergen


immunotherapy for single stinging insect venom. Code 95120
is incorrect because this code is for allergen immunotherapy in
the office including provision of allergenic extract for a single
injection. Code 95125 is incorrect because
this code is for 2 or more injections of allergenic extracts and
95131 is incorrect because this code is for two stinging insect
venoms and not just one, as specified above

42. Answer: C - 95250 is the correct code for the placement, hook
up, calibration of monitor with patient training, removal of sensor
and printout of recording for continuous glucose monitoring.
Code 95180 is incorrect because this is the code for a rapid
desensitization procedure. Code 95199 is incorrect because this
is the code for an unlisted allergy/clinical immunologic service or
procedure. Code 95251 is incorrect because this is the code for
interpretation and report of the printout of the continuous
glucose monitoring report.

43. Answer: D - 95807 is the correct code for a sleep study


attended by a technologist simultaneously recording ventilation,
respiratory effort, ECG ECT, and oxygen saturation. Codes
95800, 95801 and 95807 are all incorrect codes because these
codes are for unattended sleep studies.

44. Answer: D - 95864 is the correct code for four extremities


electromyography. Code 95860 is incorrect because this code
is for an electromyography of one extremity. Code 95861 is
incorrect because this code is for two extremities. Code 95863
is incorrect because this code is for a three extremity EMG.

45. Answer: B - 95819 is the correct code for EEG, including


recording awake and asleep. Code 95816 is incorrect
because this code is for recording awake and drowsy EEG.
Code 95822 is incorrect because this recording is done
during a coma or only during sleep. Code 95824 is incorrect
because this code is for EEG performed for cerebral death
evaluation only.
46. Answer: C - 96040 x 3 is the correct code for 90 minutes, as
this code is initially for 30 minutes and must be multiplied for
the actual amount of time spent in counseling with the patient
and/or family.

47. Answer: A - 96040 is the correct code for genetic


counseling services for 16 to 30 minutes of face-to-face time.

48. Answer: B - 96040 x 2 is the correct code for 46 + minutes of


genetic counseling services

49. Answer: C - 96116 is the correct code for neurobehavioral


status exam. Code 96110 is incorrect because this code is for
developmental screening. Code 96111 is incorrect because this
code is for developmental testing and 96118 is incorrect
because this code is for neuropsychological testing.

50. Answer: B - 96111 is the correct code for developmental


testing, including assessment of motor, language, social,
adaptive, and/or cognitive function by standardized
developmental instruments with interpretation and report. Code
96110 is incorrect because this code is for developmental
screening. Code 96116 is incorrect because this code is for
neurobehavioral status examination and 96118 is incorrect
because this code is for neuropsychological testing.
51. Answer: B - 96120 is the correct code for neuropsychological
testing administered by a computer with qualified healthcare
professional interpretation and report. Code 96119 is incorrect
because this code is for neuropsychological testing
administered by a technician. Code 96125 is incorrect because
this code is for standardized cognitive performance testing.
Code 96127 is incorrect because this is the code for brief
emotional/behavioral assessment with scoring and
documentation.

52. Answer: A - 96150 is the correct code for the initial health
and behavior assessment. Code 96151 is the incorrect code
because this code is for re-assessment. Codes 96152 and
96153 are both incorrect codes because these involve health
and behavior interventions, not initial assessments.

53. Answer: D - 96155 is the correct code for a health and


behavior intervention with only the family.

54. Answer: B - 96153 is the correct code for group health and
behavior intervention of two or more patients. Code 96152 is
incorrect because this is the code for individual health and
behavior intervention. Code 96154 is incorrect because this
code is for family health and behavior intervention with the
patient present. Code 96155 is incorrect because this code is for
family health and behavior intervention without the patient
present.

55. Answer: C - 96360, 96361 is the correct code for initial


hydration intravenous infusion of 90- minute duration. Code
96360 is the correct code for minutes 31 to 60. Add-on code
96361 is also correct for each additional hour after the first
one.
56. Answer: C - 96372 is the correct code for subcutaneous or
intramuscular therapeutic, prophylactic or diagnostic injection.
J0690 is the correct code for Ancef. Code 96365 is incorrect
because this is the code for intravenous infusion for therapy,
prophylaxis or diagnosis. Code 96369 is incorrect because this
code is for subcutaneous infusion for therapy or prophylaxis.
Code 96374 is incorrect because this code is for intravenous
push of therapeutic, prophylactic or diagnostic injection.

57. Answer: B - 96374, 96376 is the correct code combination.


Code 96374 is the correct code for initial intravenous push and
96376 is the add-on code for each additional sequential
intravenous push of the same substance. J2250 is the correct
code for Versed x 2 because two doses were given. Code 96372
is incorrect because this code is for the subcutaneous or
intramuscular administration of therapeutic, prophylactic or
diagnostic injection. Code 96375 is incorrect because this code
is for each additional sequential intravenous push of a new
substance/drug. Code 96379 is incorrect because this code is
for unlisted therapeutic, prophylactic, or diagnostic intravenous
or intra-arterial injection or infusion.

58. Answer: B - 96422 is the correct code for intra-arterial


chemotherapy administration using the infusion technique for
up to one hour. Code J9260 is the correct code for 50 mg of
Methotrexate. Code 96420 is incorrect because this code is for
the push technique. Code 96423 is incorrect because this code
is the add-on code for each additional hour after the first. Code
94625 is incorrect because this code is for a prolonged infusion
(more than eight hours).

59. Answer: D - 96422 is the correct code for chemotherapy


administration using intra-arterial infusion technique, but this
code is only used for the first hour. Code 96423 is the correct
add- on code for each additional hour after the first hour.
Because this patient had a total of four hours, this code is
multiplied by three. Code J9190 is the correct code for the drug
Fluorouracil.

60. Answer: A - 96401 is the correct code for chemotherapy


administration using subcutaneous or intramuscular technique.
J9019 is the correct code for 1000 units of Asparaginase. Code
96102 is incorrect because this is for a hormonal anti-neoplastic
drug. Code 96405 is incorrect because this is chemotherapy
administered intralesionally. Code 96409 is incorrect because
this is the code for intravenous push technique of the drug and
this patient received this drug intramuscularly.

61. Answer: A - 96567 is the correct code for each phototherapy


exposure session for external application of light to destroy
premalignant and/or malignant lesions of the skin and
adjacent mucosa. Codes 96570 and 96571 are both add-on
codes for photodynamic therapy using endoscopic application
of light. Code 96900 is incorrect because this is the code for
actinotherapy.

62. Answer: B - 43200 is the correct code for transoral flexible


esophagoscopy. Code 96567 is the correct add-on code for
photodynamic therapy by endoscopic application of light to
ablate abnormal tissue via activation of photosensitive drugs.
Modifier -52 is correct because less than 23 minutes was spent
doing this procedure. Code 96567 is incorrect because this
code is not involving an endoscopy. Add-on code 96571 is
incorrect because this code is for each additional 15 minutes
past the initial 30 minutes, and this patient’s procedure took less
than 30 minutes.

63. Answer: B - 31622 is the correct code for bronchoscopy. Code


96570 is a correct add-on code for the first 30 minutes of
photodynamic therapy by endoscopic application of light to
ablate abnormal tissue via activation of photosensitive drugs.
Code 96571 is also a correct add-on code for each additional 15
minutes. Code 31623 is incorrect because this is for a
bronchoscopy with brushings.

64. Answer: B - 96902 is the correct code for microscopic


examination of hairs plucked or clipped by the examiner to
determine telogen and anagen counts or structural hair shaft
abnormalities. Code 96900 is incorrect because this code is for
actinotherapy. Code 96904 is incorrect because this code is for
whole body integumentary photography for monitoring of high-
risk patients with dysplastic nevus syndrome. Code 96910 is
incorrect because this code is for Photochemotherapy, including
tar and ultraviolet B, or petrolatum and ultraviolet B.

65. Answer: C - 96921 is the correct code for 250-500 sq. cm


laser treatment for inflammatory skin diseases such as
psoriasis. Code 96913 is incorrect because this code is used
for
photochemotherapy for severe photoresponsive dermatoses
requiring at least 4-8 hours of care under direct supervision of
the physician. Code 96920 is incorrect because this code is for
laser treatments for a total area less than 250 sq. cm. Code
96922 is incorrect because this code is used for areas larger
than 500 sq. cm.

66. Answer: A - 96900 is the correct code for actinotherapy.


Code 96902 is incorrect because this code is for microscopic
examination of hairs plucked to determine telogen and anagen
counts. Codes 96910 and 96913 are both incorrect because
these codes are used for photochemotherapy.

67. Answer: A - 97001 is the correct code for initial physical


therapy evaluation. Code 97002 is incorrect because this code
is for re-evaluation for physical therapy. Codes 97003 and 97004
are both codes for occupational therapy evaluation.

68. Answer: D - 97006 is the correct code for athletic training re-
evaluation. Codes 97001 and 97002 are both incorrect because
these codes refer to physical therapy evaluations. Code 97005
is incorrect because this code is for the initial athletic training
evaluation.

69. Answer: D - Z51.89 "Encounter for other specified aftercare"


is the correct code for all occupational therapy and physical
therapy visits. Codes for diagnoses and reasons for care are
specified separately. Code Z51.5 is for palliative care and code
Z51.81 is for therapeutic drug level monitoring, both of which
do not apply. Code Z51.8 is a general code for other specified
aftercare, so code Z51.89 is more accurate.

70. Answer: A - 97802 is the correct code for 15 minute of


medical nutrition therapy for initial assessment and
intervention for a single patient. Code 97803 is incorrect
because this code is for re-assessment and intervention for an
individual patient. Code 97804 is incorrect because this code
is for group therapy and 97810 is incorrect because this code
is for acupuncture.

71. Answer: C - 97804 is the correct code for group (2+)


medical nutrition therapy lasting 30 minutes. Codes 97802 and
97803 are both incorrect codes because these codes are for
individual therapy and not for group therapy. Code 97810 is
incorrect because this code is for acupuncture.

72. Answer: B - 97803 is the correct code for medical


nutrition therapy re-assessment and intervention for each
15 minutes. Code 97802 is incorrect because this code is
for initial assessment. Code 97804 is incorrect because this
code is for group therapy, and 97810 is incorrect because
this code is for acupuncture.

73. Answer: A - 98925 is the correct code for OMT of 1-2 body
regions. Code 98926 is incorrect because this code is for 3-4
body regions. Code 98927 is incorrect because this code is for
5-6 body regions and 98928 is incorrect because this code is
for 7-8 body regions.

74. Answer: C - 98927 is the correct code for OMT of 5-6 body
regions.
75. Answer: C - 98928 is the correct code for OMT for 7-8 body
regions. Code 98926 is incorrect because this code involves 3-4
body regions. Code 98927 is incorrect because this code is for
5-6 body regions and code 98929 is incorrect because this
involves 9-10 body regions.

76. Answer: B - 98941 is the correct code for chiropractic


manipulative treatment of 3-4 regions of the spine. Code 98940
is incorrect because this code is only for 1-2 regions of
manipulation of the spine. Code 98942 is incorrect because this
code is for all five regions of the spine and code 98943 is
incorrect because this is for extraspinal regions.

77. Answer: D - 98943 is the correct code for extraspinal


chiropractic manipulative treatment of one or more regions.
Codes 98940, 98941, and 98942 are all incorrect because these
codes all refer to spinal manipulation and not the extraspinal
region of the head.

78. Answer: A - 98940 is the correct code for chiropractic


manipulative treatment of the spine for 1- 2 regions. Codes
98941 and 98942 are incorrect codes because, although these
are codes for manipulation of the spine, these codes involve
more than just one region of manipulation. Code 98943 is
incorrect because this code involves the extraspinal regions.

79. Answer: C - 98962 is the correct code for patient self-


management education and training for a group of 5-8 patients.
Code 98960 is incorrect because this code is for an individual
patient. Code 98961 is incorrect because this code is for group
training of 2-4 patients. Code 98966 is incorrect because this
code is for telephone assessment and management services.
80. Answer: A - 98960 is the correct code for individual patient
training to promote self- management, which may or may not
include family members or caregivers. Codes 98961 and
98962 are incorrect codes because these both refer to group
therapy for self-management education and training. Code
98966 is incorrect because this code is for telephone services.

81. Answer: B - 98961 is the correct code for education and


training for a group of 3-4 patients to promote self-management
of a particular disease. Code 98960 is incorrect because this
code is for individual patient training/education. Code 98962 is
incorrect because this code is for group training of 5-8 patients.
Code 98966 is incorrect because this code refers to telephone
services and not face-to-face training.

82. Answer: A - 98966 is the correct code for a 5-10 minute


telephone contact for a nonphysician healthcare professional. If
the patient would have followed up within 24 hours or at the
soonest available appointment, then this code would not be able
to be used because this telephone service would be considered
part of the patient’s workup for that office visit. Codes 98967 and
98968 are codes for 11+ minutes of telephone consultation
services. Code 98969 is incorrect because this code is for online
assessment and management services, not telephone services.

83. Answer: A - 99143 is the correct code for moderate sedation


services for a patient younger than five years of age for the first
30 minutes of intra-service time. Code 99144 is incorrect
because
this code is for ages five years and older. Code 99145 is
incorrect because this is an add-on code for each additional 15
minutes of sedation time and code 99148 is incorrect because
this code is used when a second physician is needed.

84. Answer: B - 99144 is the correct code for moderate sedation


of a patient age five years and older for the first 30 minutes of
the procedure. Codes 99143 and 99148 are incorrect because
these codes are for patients younger than five years of age.
Code 99145 is incorrect because this is an add-on code for each
additional 15 minutes after the first 30.

85. Answer: D - 99144, 99145 is the correct code sequence for


the moderate sedation of a patient older than five years of age.
Code 99144 is for the first 30 minutes and 99145 is the add-on
code for each additional 15 minutes after the first 30 minutes.
Code 99143 is incorrect because this code is for moderate
sedation for patients under the age of five. Single codes 99144
and 99145 are incorrect and must be used together to indicate
the appropriate length of time for the moderate sedation service.
RADIOLOGY (23 QUESTIONS) – ANSWER KEY &
RATIONALE

1. Answer: B - The correct code for this visit is 96020


(Neurofunctional Testing Selection and Administration During
Non-Invasive Imaging Functional Brain Mapping, with Test
Administered Entirely by a Physician, with Review or Test
Results and Report) and 70555 (Magnetic Resonance Imaging,
Brain, Functional MRI; Requiring Physician or Psychologist
Administration of Entire Neurofunctional Testing).The physician
administered the entire tests, therefore the correct code is
70555 not code 70554. In parenthesis, under the code, there is
a note that states that code 96020 can only be used with code
70555.

2. Answer: B - The correct code for a complete chest x-ray is 71030


(Radiologic Examination, Chest, Complete, Minimum of 4
Views).71034is the only code that needs to be included because
the procedure description does not mention fluoroscopic
guidance. Code 71020 only allows for 2 views of the chest,
while codes 71021, 71022, and 71023 include other
specifications that are not included in the procedure description.

3. Answer: A - The only code that the radiologist should report


would be 72114 (Radiologic Examination, Spine, Lumbosacral;
Complete, including Bending Views, Minimum of 6 Views) with
modifier -TC appended. Modifier -TC indicates that the
procedure consisted of taking the x- rays only and not the
interpretation of the x-rays. The radiologist sent the x-rays to the
patient’s PCP for interpretation; therefore the only part of this
service that was provided was the technical portion.

4. Answer: C - The correct code for the procedure is 72198


(Magnetic Resonance Angiography, Pelvis, With or Without
Contrast Materials). All of the other codes listed in the question
are used for a magnetic resonance imaging (MRI) not a
magnetic resonance angiography (MRA).

5. Answer: B - The radiology code that should be reported is


74178 (Computed Tomography, Abdomen and Pelvis; Without
Contrast Material in One or Both Body Regions, Followed by
Contrast Materials and Further Sections in One or Both Body
Regions).74178 is the only code that needs to be reported.

6. Answer: A - The appropriate code for this service is 75625


(Aortography, Abdominal, by Serialography, Radiological
Supervision and Interpretation). Code 75625 does not need to
be appended with the modifier -26 because modifier -26
indicates radiological supervision and interpretation. Code
75625 already states that the service was just for the
supervision and interpretation. To append modifier -26 would
be repetitive.

7. Answer: C - Radiation oncology codes consists of a specialized


portion of radiology codes that are used to report radiological
services that are used to deliver radiation treatments to tumors.
Radiation oncology services are provided by a multi-disciplinary
group of physicians, physicists, computer scientists,
radiotherapy technologists, nutritionists and social workers.
These professionals work together to create and deliver
radiation treatments for tumors.
8. Answer: C - The correct code for this service is
77261(Therapeutic Radiology Treatment Planning; Simple).
This procedure is considered simple because the planning
focused on a single treatment area and encompassed only a
single port and no blocking. Other levels of planning may
consist of more extensive treatments such as treatments for
more aggressive forms of cancer.

9. Answer: A - The correct codes for the service are: 77300 (Basic
Radiation Dosimetry Calculations) and 77332, mod-51 (Design
and Construction of the Treatment Device). Modifier - 51 needs
to be included on the claim to indicate that the procedure was a
multiple procedure performed on the same date of service.

10. Answer: B - 76830 is the correct code for nonobstetrical


transvaginal ultrasound. Code 76801 is incorrect because this
code is for an obstetrical ultrasound. Codes 76856 and 76857
are incorrect because these codes are not for a transvaginal
ultrasound; they are for a pelvic ultrasound.

11. Answer: B - 76825 is the correct code for fetal


echocardiography in real time with image documentation (2D)
with M-mode recording. Code 76820 is incorrect because this
code is for fetal Doppler velocimetry. Code 76826 is incorrect
because this code is for a follow-up or repeat fetal
echocardiogram and code 76827 is incorrect because this code
is for fetal Doppler echocardiography with pulsed wave and/or
continuous wave with spectral display.

12. Answer: B - 77003 is the correct code for fluoroscopic


guidance and localization of needle or catheter tip for spine or
paraspinous diagnostic or therapeutic injection procedures
(epidural or subarachnoid). Code 77002 is incorrect because
this code is for fluoroscopic guidance for needle placement
(biopsy, aspiration, injection, localization device). Code 77011 is
incorrect because this code is for computed tomography
guidance for stereotactic localization. Code 77012 is incorrect
because this code is for computed tomography guidance for
needle placement, radiological supervision and interpretation.

13. Answer: C - 77021 is the correct code for magnetic


resonance guidance for needle placement (e.g., for biopsy,
needle aspiration, injection, or placement of localization
device) radiological supervision and interpretation. Codes
77011 and 77012 are incorrect because these codes are for
computed tomography guidance procedures. Code 77022 is
incorrect because this code is for magnetic resonance
guidance for, and monitoring of, parenchymal tissue ablation.

14. Answer: B - 77002 is the correct code for fluoroscopic


guidance for needle placement (e.g., biopsy, aspiration,
injection, localization device). Code 77001 is incorrect as this is
an add-on code for fluoroscopic guidance for central venous
access device placement. Code 77003 is incorrect because this
code is for fluoroscopic guidance and localization of needle or
catheter tip for spine or paraspinous diagnostic or therapeutic
injection procedures. Code 77011 is incorrect because this code
is for computed tomography guidance for stereotactic
localization.
15. Answer: D - 77057, 77052 is the correct code sequence for
this procedure. 77057 is the correct code for a screening
mammography, bilateral with 2-view film study of each breast
and 77052 is the correct code for computer-aided detection
during a screening mammography. Code 77054 is incorrect
because this code is for a mammary ductogram or galactogram.
Code 77056 is incorrect because this is for a bilateral
mammography, but not for a screening mammography. Code
77057 is correct, but alone does not include computer-aided
detection and requires the additional code to indicate this further
testing.

16. Answer: B - 77056 is the correct code for bilateral


mammography. Code 77055 is incorrect because this code is for
a unilateral mammography. Code 77057 is incorrect because
this code is for a bilateral screening mammography. Code
77058 is incorrect because this code is for unilateral magnetic
resonance imaging of the breast with/without contrast material.

17. Answer: B - 77061 is the correct code for a unilateral digital


breast tomosynthesis. Code 77058 is incorrect because this
code is for unilateral magnetic resonance imaging. Code 77062
is incorrect because this code is for bilateral digital breast
tomosynthesis. Code 77063 is incorrect because this code is for
a bilateral screening digital breast tomosynthesis.

18. Answer: A - 77072 is the correct code for bone age studies.
Code 77073 is incorrect because this code is for bone length
studies. Code 77074 is incorrect because this is for radiologic
examination, osseous survey. Code 77076 is incorrect because
this is the code for radiologic examination, osseous survey for
an infant.

19. Answer: B - 77078 is the correct code for computed


tomography, bone mineral density study, one or more sites of
the axial skeleton. Code 77077 is incorrect because this code
is for a joint survey. Code 77080 is incorrect because this code
is for a dual-energy X-ray absorptiometry (DXA), bone density
study. Code 77086 is incorrect because this code is for
vertebral fracture assessment via DXA.

20. Answer: C - 77086 is the correct code for vertebral fracture


assessment via dual-energy X-ray absorptiometry (DXA). Code
77078 is incorrect because this code is for computed
tomography. Code 77080 is incorrect because this code is for a
DXA but is for a bone density study of one or more sites of the
axial skeleton, not specifically to assess for vertebral fracture.
Code 77084 is incorrect because this code is for a magnetic
resonance imaging for bone marrow blood supply.

21. Answer: D - 78202 is the correct code for static liver imaging
with vascular flow. Code 78195 is incorrect because this code is
for lymphatics and lymph nodes imaging. Code 78199 is
incorrect because this code is for unlisted hematopoietic,
reticuloendothelial and lymphatic procedure, diagnostic nuclear
medicine. Code 78201 is incorrect because this code is for
static liver imaging without vascular flow.

22. Answer: C - 78740 is the correct code for ureteral reflux


study (radiopharmaceutical voiding cystogram). Code 78700 is
incorrect because this code is for kidney imaging morphology.
Code 78725 is incorrect because this code is for kidney
function study, non-imaging radioisotopic
study. Code 78799 is incorrect because this code is for an
unlisted genitourinary procedure and there is a specific code
listed for this procedure.

23. Answer: C - 78761 is the correct code for testicular imaging


with vascular flow. Code 78700 is incorrect because this code is
for kidney imaging morphology. Code 78740 is incorrect
because this code is for ureteral reflux study. Code 78799 is
incorrect because this is the code for an unlisted genitourinary
procedure.

You might also like